Zahlentheorie - Blu7 Homepage

Werbung
Zahlentheorie
Wintersemester 2006/2007
Prof.Dr. Jürgen Sander
Dieses Skript wurde erstellt von Alexander Seifert.
Anregungen und Anmerkungen schickt bitte an [email protected].
Meine Homepage: www.blu7.com.
Stand des Inhalts: 02.01.2007
Version 2007.01.05a
Inhaltsverzeichnis
Kapitel 0: Grundlagen........................................................................................................ 3
Kapitel 1: Teilbarkeit ......................................................................................................... 4
Kapitel 2: Zahlentheoretische Funktionen .......................................................................... 9
Kapitel 3: Kongruenzen ................................................................................................... 19
Kapitel 4: Quadratische Reste ......................................................................................... 28
Kapitel 5: Quadratische Formen ...................................................................................... 35
Kapitel 6: Primzahlverteilung .......................................................................................... 42
Kapitel 7: Diophantische Approximation und Kettenbrüche ........................................... 47
Kapitel 8: Diophantische Gleichungen.............................................................................. 56
Kapitel 9: p-adische Zahlen.............................................................................................. 62
Anmerkung des Erstellers (diesen Skripts): Ich verwende zwei verschiedene Nummerierungen von Sätzen,
Lemmata etc. – meine und in Klammern dahinter die von Prof.Dr. Sander. Alle folgenden Bezüge auf Sätze,
Lemmata etc. gelten für meine Nummern.
Mit log x bezeichnet man in der Zahlentheorie den natürlichen Logarithmus. Dies gilt auch für dieses Skript.
Wegen Zeitmangel konnte Prof.Dr. Sander seine Theorie nicht komplett in diesem Semester präsentieren.
Somit habe ich den Rest hinten angefügt, den ich aus einem PDF-Dokument (WS 2001/2002) entnommen,
das ich von ihm bekommen habe. Zusätzlich hat er nicht alle Beweise geschafft und hat deswegen einige
unwichtige übersprungen und ein Korollar weggelassen. Alles das und fehlende interessante Zwischenbemerkungen habe ich zur Vervollständigung ebenfalls hinzugenommen.
Dieses Dokument ist eine Erweiterung der Vorlesung aus dem Wintersemester 2004/2005 mit den oben
beschriebenen Ergänzungen und denen von Martin Schmidt, dem ich hier danken möchte für die Anmerkungen und Korrekturvorschläge, die er im Zusammenhang mit der Vorlesung im Wintersemester
2006/2007 erarbeitet hat.
2
3
Kapitel 0
Grundlagen
N = { 1,2, 3,...} setzt im Grunde die Peano-Axiome voraus.
Satz 0.1 (0.1)
Sei M ⊆ N . Dann gilt:
(i) Ist 1 ∈ M und gilt für jedes m ∈ M , dass auch m + 1 ∈ M , so ist M = N (vollständige Induktion).
(ii) Ist M ≠ ∅ , so enthält M ein eindeutiges kleinstes Element (Wohlordnung).
Auf N lassen sich Addition und Multiplikation definieren. Diese Verknüpfungen sind assoziativ, kommutativ
und distributiv. N lässt sich so anordnen, dass für alle m, n ∈ N entweder m = n , m > n oder m < n gilt.
Wir bezeichnen wie üblich mit Z := { ..., −3, −2, −1, 0,1, 2, 3,...} den Ring der ganzen Zahlen und mit
Q := { ab | a ∈ Z, b ∈ N } den Körper der rationalen Zahlen und mit R bzw. C die Körper der reellen bzw.
komplexen Zahlen.
Satz 0.2 (0.2)
Sei M ⊆ Z . Dann gilt:
(i) Ist a ∈ M und gilt für alle m ∈ M mit m ≥ a , dass auch m + 1 ∈ M , so ist
{m ≥ a | m ∈ M } = {m ≥ a | m ∈ Z } .
(ii) Ist M ≠ ∅ und ist M nach unten beschränkt, so besitzt M ein eindeutiges kleinstes Element. Entsprechend hat eine nach oben beschränkte Menge ein eindeutiges größtes Element.
4
Kapitel 1
Teilbarkeit
Definition 1.1
Seien a, b ∈ N . Wir sagen a teilt b , geschrieben a|b , genau dann, wenn ein c ∈ N existiert mit b = a ⋅ c .
Dabei heißt a Teiler von b und b heißt Vielfaches von a , c heißt Komplementärteiler von b bezüglich a .
Satz 1.2 (1.1)
Seien a, b, c ∈ N . Dann gilt:
(i) Die Relation a|b ist reflexiv ( a|a ) und transitiv (aus a|b und b|c folgt a|c ).
(ii) Die Relation a|b ist nicht symmetrisch. Es gilt: aus a|b und b|a folgt a = b .
(iii) Aus a|b folgt a ≤ b . Insbesondere hat jede natürliche Zahl nur endlich viele Teiler.
Beweis
Übung.
Definition 1.3
Seien a, b ∈ Z , a ≠ 0 . Wir sagen a teilt b genau dann, wenn ein c ∈ Z existiert mit b = a ⋅ c .
Bemerkung 1.4
Satz 1.2 gilt weitestgehend auch in Z . Zu ändern sind:
(ii) aus a|b und b|a folgt a = b ,
(iii) aus a|b folgt a ≤ b .
Satz 1.5 (1.2) (Division mit Rest)
Seien a, b ∈ Z , b > 0 . Dann existieren q, r ∈ Z derart, dass a = q ⋅ b + r mit 0 ≤ r < b gilt.
Beweis
Wähle q maximal so, dass b ⋅ q ≤ a gilt, d.h. q := max { p ∈ Z | b ⋅ p ≤ a } . Es ist r := a − b ⋅ q ≥ 0 nach
Definition von q . Nach Konstruktion von q ist b ( q + 1 ) > a , also
r = a − b ⋅ q = a − b (q + 1 ) + b < a − a + b = b .
Bemerkung 1.6
Satz 1.5 gilt analog für jedes b ∈ Z , b ≠ 0 , allerdings mit 0 ≤ r < b .
Lemma 1.7 (1.3) (Existenz eines größten gemeinsamen Teilers)
Seien a, b ∈ N . Dann existiert ein eindeutiges d ∈ N mit folgenden Eigenschaften:
(i) d|a und d|b ( d heißt gemeinsamer Teiler von a und b ),
(ii) für jeden gemeinsamen Teiler t von a und b gilt t|d .
Beweis
Existenz: Seien M := {ax + by | x , y ∈ Z } ⊆ Z und M ′ := M ∩ N ⊆ N . Offenbar ist M ′ ≠ ∅ , z.B. a ∈ M ′ .
Nach Satz 0.1 besitzt M ′ ein eindeutiges Minimum d . Zu zeigen: d erfüllt (i) und (ii). Nach Konstruktion
von d gibt es Zahlen x 0 , y 0 ∈ Z mit d = ax 0 + by 0 . Ist also t ein gemeinsamer Teiler von a und b , d.h.
t|a und t|b , so folgt t|ax 0 und t|by 0 und damit t|ax 0 + by 0 = d . Nach Satz 1.5 existieren q, r ∈ Z mit
0 ≤ r < d und a = q ⋅ d + r . Also folgt:
r = a − q ⋅ d = a − q ( ax 0 + by 0 ) = a ( 1 − qx 0 ) − bqy0 ∈ M .
1. Fall: Aus r > 0 folgt r ∈ M ′ . Dies steht im Widerspruch zu d = min M ′ .
2. Fall: Aus r = 0 folgt a = q ⋅ d + 0 = q ⋅ d , d.h. d|a . Analog: d|b , d.h. (i) gilt.
Eindeutigkeit: Nehmen wir an, es gibt d und d ′ mit (i) und (ii). Aus (ii) folgt:
Kapitel 1
Teilbarkeit
t := d ′ ⇒ d ′|d und t := d ⇒ d|d ′ .
Also gilt d = d ′ .
Definition 1.8
Seien a, b ∈ N . Die nach Lemma 1.7 eindeutige Zahl d ∈ N heißt größter gemeinsamer Teiler von a und b
( ggT ). Schreibweisen: ggT ( a, b ) oder (a, b ) . Ist ( a, b ) = 1 , so heißen a und b teilerfremd.
Bemerkung 1.9
(i) Lemma 1.7 lässt sich auf Zahlen a1, a2 ,..., an ∈ N übertragen. Die eindeutige Zahl d = ( a1,..., an ) heißt
analog ggT von a1,..., an . Ist d = 1 , so heißen a1,..., an teilerfremd. Gilt sogar ( ai , a j ) = 1 für alle
1 ≤ i < j ≤ n , so heißen a1,..., an paarweise teilerfremd.
(ii) Die Definition des ggT lässt sich auf Z übertragen: Für a1,..., an ∈ Z \ { 0 } setzen wir
( a1,..., an ) := ( a1 ,..., an ) .
Da 0 durch jede natürliche Zahl teilbar ist, liegt nahe:
( a1,..., an , 0 ) := ( a1,..., an ) .
Sind alle ai = 0 , i = 1,..., n , so setzen wir ( a1,..., an ) = 0 .
Beispiel 1.10
Seien a = 15 und b = 6 . Dann haben wir:
M = {15x + 6y | x , y ∈ Z } = { 0, ±15, ±6, ±3, ±9, ±12,... } = 3Z .
Dann folgt:
M ′ := M ∩ N = 3N , d = ( 15, 6 ) = 3 .
In der Zahlentheorie geht es häufig um das Lösen von Gleichungen in ganzen Zahlen. Solche Gleichungen
nennt man diophantische Gleichungen.
Satz 1.11 (1.4) (Lösbarkeit linearer diophantischer Gleichungen)
Seien a1,..., an ∈ Z \ { 0 } , b ∈ Z . Dann ist die diophantische Gleichung
a1x1 + ... + an x n = b
genau dann lösbar, wenn ( a1,..., an )|b .
Beweis
„ ⇒ “ Sei d := ( a1,..., an ) . Klar ist d|a j für 1 ≤ j ≤ n , also auch d|a j x j und damit d|a1x1 + ... + an x n = b .
„ ⇐ “ Nach Definition des ggT ist d = ( a1,..., an ) eine ganzzahlige Linearkombination von a1,..., an , d.h., es
gibt x1,..., x n ∈ Z mit d = a1x1 + ... + an x n . Da b / d ∈ Z ist, gilt:
b
b
b = a1 x1 + ... + an x n .
d
d
∈Z
∈Z
Das eleganteste Verfahren zur Bestimmung des ggT ist der euklidische Algorithmus: Gegeben sind a, b ∈ N ,
gesucht wird (a, b ) . Das Verfahren ist die wiederholte Division mit Rest.
Es gibt q1, r1 ∈ Z mit a = q1 ⋅ b + r1 , 0 ≤ r1 < b . Ist r1 = 0 , sind wir fertig. Ist r1 > 0 , so gibt es q2 , r2 ∈ Z
mit b = q2 ⋅ r1 + r2 , 0 ≤ r2 < r1 . Falls r2 = 0 , sind wir fertig. Ansonsten gibt es q 3 , r3 ∈ Z mit
r1 = q 3 ⋅ r2 + r3 … Der Algorithmus bricht nach spätestens b Schritten ab, denn b > r1 > r2 > r3 > ... ≥ 0 .
Satz 1.12 (1.5)
Mit obigen Bezeichnungen gilt: Ist k + 1 der kleinste Index mit rk +1 = 0 , so ist rk = ( a, b ) .
Beweis
Aus a = q1 ⋅ b + r1 , b = q2 ⋅ r1 + r2 , r1 = q 3 ⋅ r2 + r3 , …, rk −2 = qk ⋅ rk −1 + rk und rk −1 = qk +1 ⋅ rk folgt, dass
aus d = ( a, b )|a und d|b d|q1b folgt und daraus d|( a − q1b ) = r1 folgt und daraus d|(b − q2r1 ) = r2 folgt und
daraus d|r3 folgt … und daraus d|rk folgt. Umgekehrt folgt aus rk |rk −1 , dass rk |( qk rk −1 + rk ) = rk −2 und daraus rk |rk −3 … und daraus rk |b und daraus rk |a .
Also ist rk gemeinsamer Teiler von a und b und da wegen dieser Eigenschaft rk |( a, b ) = d , gilt rk = d . 5
Kapitel 1
Teilbarkeit
Beispiel 1.13
Gesucht wird ( 37, 26 ) . Es gilt:
37 = 1 ⋅ 26 + 11 , 26 = 2 ⋅ 11 + 4 , 11 = 2 ⋅ 4 + 3 , 4 = 1 ⋅ 3 + 1 , 3 = 3 ⋅ 1 .
Also ist der ggT hier 1.
Zurück zum Lösen linearer diophantischer Gleichungen:
37x + 26y = 1 .
Aus der Berechnung des ggT mit dem euklidischen Algorithmus findet man:
37 ⋅ ( −7 ) + 26 ⋅ 10 = 1 .
Definition 1.14
Eine natürliche Zahl n > 1 heißt Primzahl, falls n nur die so genannten trivialen Teiler 1 und n besitzt.
Die Menge aller Primzahlen bezeichnen wir mit P = { 2, 3, 5, 7,11,... } .
Satz 1.15 (1.6) (Euklid)
Es gibt unendlich viele Primzahlen.
Beweis
Behauptung: Der kleinste Teiler d > 1 von n > 1 ist eine Primzahl. Denn: Wäre d ∉ P , so gäbe es d1, d2 ∈ N>1
mit d = d1 ⋅ d2 , wobei d1, d2 < d gilt. Es ist aber d1 > 1 und d1|d . Da d|n , folgt der Widerspruch aus d1|n ,
da d der kleinste Faktor von n ist.
Annahme: P < ∞ . D.h. P = { p1,..., pk } für ein k ∈ N . Setze n = p1p2 ⋅ ... ⋅ pk + 1 , dann ist n > 1 . Sei
nun d der kleinste Teiler größer als 1 von n . Wir wissen, dass d ∈ P gilt. Also ist d = p j für ein passendes
j . Da p j |n und p j |p1p2 ⋅ ... ⋅ pk = n − 1 folgt der Widerspruch aus p j |( n, n − 1 ) = 1 .
Satz 1.16 (1.7) (Fundamentalsatz der Zahlentheorie)
Jede natürliche Zahl n > 1 besitzt eine bis auf die Reihenfolge der Faktoren eindeutige Darstellung als Produkt von Primzahlen.
Beweis
Existenz: Wegen n > 1 hat n einen Primfaktor p1 ∈ P . Ist n = p1 , sind wir fertig, ansonsten sei
n
n1 :=
∈ N>1 .
p1
Es folgt, dass n1 einen Primfaktor p2 ∈ P besitzt. Ist n1 = p2 , sind wir fertig, ansonsten sei …
Das Verfahren bricht nach höchstens n Schritten ab, denn es gilt:
n
n
n>
>
> ... ≥ 1 .
p1
p1p2
Eindeutigkeit: Behauptung: Ist p ∈ P mit p|mn für m, n ∈ N , so gilt: p|m oder p|n . Denn: Nehmen wir an,
p ? m , also ( p, m ) = 1 , so gibt es x , y ∈ Z mit px + my = 1 . Somit ist pnx + mny = n , wobei p|pn und
p|mn und damit p|n . Induktiv folgt aus p|m1m2 ⋅ ... ⋅ mk die Existenz eines 1 ≤ j ≤ k mit p|m j .
Nehmen wir jetzt an, n = p1p2 ⋅ ... ⋅ pk = q1q2 ⋅ ... ⋅ ql für pi , q j ∈ P , 1 ≤ i ≤ k , 1 ≤ j ≤ l . Wir führen Induktion nach n . Für n = 2 ist die Voraussetzung erfüllt. Betrachten wir nun n − 1
n . Mit p1|n = q1 ⋅ ... ⋅ ql
gibt es ein 1 ≤ j ≤ l mit p1|q j . Da q j ∈ P , ist p1 = q j . Betrachten wir
n
= p2 ⋅ ... ⋅ pk = q1 ⋅ ... ⋅ q j −1 ⋅ q j +1 ⋅ ... ⋅ ql ,
p1
dann stimmen p2 ⋅ ... ⋅ pk und q1 ⋅ ... ⋅ q j −1 ⋅ q j +1 ⋅ ... ⋅ ql nach Induktionsvoraussetzung bis auf die Reihenfolge
der Faktoren überein.
Definition 1.17
Sei n ∈ N>1 . Die nach Satz 1.16 eindeutige Darstellung n = p1e1 ⋅ ... ⋅ pkek mit Primzahlen p1 < ... < pk und
e j ∈ N , 1 ≤ j ≤ k , heißt kanonische Primfaktorzerlegung von n . Für ein festes p ∈ P setzen wir
⎧⎪ 0, falls p j ? n,
ep j ( n ) := ⎪⎨
⎪⎪⎩e j , falls p j |n.
Also formal:
n = ∏ pep n .
(
p ∈P
6
)
Kapitel 1
Teilbarkeit
Dies ist ein endliches Produkt. Dabei heißt ep ( n ) die Ordnung von p in n .
Satz 1.18 (1.8)
Seien a1,..., an ∈ Z nicht alle gleich 0. Dann gilt:
( a1,..., an ) =
∏ p min
1≤i ≤n e p ( ai )
,
p ∈P
wobei das Minimum nur über die ai ≠ 0 erstreckt wird. Das Produkt ist endlich.
Beweis
Das obige Produkt erfüllt offenbar die Bedingungen (i) und (ii) aus Lemma 1.7.
Beispiel 1.19
Gegeben sei
102x + 33y = 6 .
Diese Gleichung ist genau dann ganzzahlig lösbar, wenn ( 102, 33 )|6 . Lösung:
102 ⋅ 2 − 6 ⋅ 33 = 6 .
Definition 1.20
Seien a1,..., an ∈ Z \ { 0 } . Die natürliche Zahl
[ a1,..., an ] :=
∏ p max
1≤i ≤n e p ( ai )
p ∈P
heißt kleinstes gemeinsames Vielfaches ( kgV ) von a1,..., an . Ist eines der ai gleich 0, so setzen wir
[ a1,..., an ] = 0 .
Aus Satz 1.18 und obiger Definition folgt:
Satz 1.21 (1.9)
Für a1,..., an , t ∈ Z gilt:
(i) ( a1,..., an ) = ( ( a1,..., an −1 ), an ) und [ a1,..., an ] = [ [ a1,..., an −1 ], an ] ,
(ii) ( ta1,..., tan ) = t ( a1,..., an ) ,
(iii) ( a1,..., an )|( a1,..., an −1 ) und [ a1,..., an −1 ]|[ a1,..., an ] ,
(iv) ( a1, a2 )[ a1, a2 ] = a1a2 .
Beweis
Übung.
Lemma 1.22 (1.10)
Seien a, b, c ∈ Z mit a|bc und ( a, b ) = 1 . Dann gilt a|c .
Beweis
Wegen ( a, b ) = 1 gilt nach Satz 1.21 (ii) (ac, bc ) = c ( a, b ) = c und wegen a|bc existiert ein g ∈ Z mit
ag = bc . Dann folgt
c = ( ac, bc ) = ( ac, ag ) = a ( c, g )
und damit a|c .
Bemerkung 1.23
Ohne die Bedingung (a, b ) = 1 ist die Aussage falsch, da z.B. 6|2 ⋅ 3 , aber 6 ? 2 und 6 ? 3 .
Nach Satz 1.11 wissen wir, dass die diophantische Gleichung ax + by = c genau dann lösbar ist, wenn
( a, b )|c . Mit dem euklidischen Algorithmus lässt sich eine Lösung finden. Alle Lösungen beschreibt ...
Satz 1.24 (1.11)
Die diophantische Gleichung ax + by = c besitzt die Lösung x 0 , y 0 ∈ Z . Alle Lösungen sind angegeben durch
b
a
x = x0 −
t und y = y 0 +
t für t ∈ Z .
( a, b )
( a, b )
Beweis
Die angegebenen Zahlen repräsentieren eine Lösung, denn:
7
Kapitel 1
Teilbarkeit
(
ax + by = a x 0 −
b
( a, b )
) (
t + b y0 +
a
( a, b )
)
t = ax 0 + by 0 = c .
Es gibt keine anderen Lösungen, denn: Sei x1, y1 eine beliebige Lösung, d.h. es gibt
ax 0 + by 0 = c = ax1 + by1 ,
also a ( x 0 − x1 ) = b ( y1 − y0 ) . Sei d = ( a, b ) . Setze
b
a
a ′ = und b ′ = ,
d
d
so dass ( a ′, b ′ ) = 1 ist. Es gilt dann a ′ ( x 0 − x1 ) = b ′ ( y1 − y 0 ) . Aus dem Lemma 1.22 folgt a ′|y1 − y 0 , d.h. es
gibt t ∈ Z mit a ′t = y1 − y 0 . Einsetzen liefert x 0 − x1 = b ′t . Insgesamt folgt:
b
a
x1 = x 0 − b ′t = x 0 −
t und y1 = y 0 + a ′t = y 0 +
t.
( a, b )
( a, b )
8
9
Kapitel 2
Zahlentheoretische Funktionen
Definition 2.1
Für x ∈ R bezeichne
[
x ] := max { m ∈ Z | m ≤ x } .
Wir nennen [ x ] die Gaußklammer, größtes Ganzes oder Ganzteil von x . Weiter sei
{ x } := x − [ x ] ,
genannt der Bruchteil von x .
Lemma 2.2 (2.1)
Seien x , y ∈ R . Dann gilt:
(i) x − 1 < [ x ] ≤ x und 0 ≤ { x } < 1 ,
(ii) für n ∈ Z ist [ x + n ] = [ x ] + n ,
(iii) [ x ] + [ y ] ≤ [ x + y ] und { x + y } ≤ { x } + { y } ,
(iv) für n ∈ N ist
⎡ x ⎤ = ⎡ [ x ]⎤ .
⎢⎣ n ⎥⎦
⎣⎢ n ⎦⎥
Beweis
(i) [ x ] ≤ x ist klar nach Definition und damit gilt x − 1 < [ x ] . Es folgt:
0 = x − x ≤ {x } = x − [ x ] < x − ( x − 1) = 1 .
(ii) Wieder klar nach Definition.
(iii) Nach (ii) gilt:
[
x ] + [y ] = [ x + [y ]] ≤ [ x + y ]
und damit
{ x + y } = ( x + y ) − [ x + y ] ≤ ( x + y ) − ( [ x ] + [ y ]) = ( x − [ x ]) + ( y − [ y ] ) = { x } + { y } .
(iv) Sei [ x ] = qn + r für passende q, r ∈ Z mit 0 ≤ r < n . Dann folgt:
⎡ [ x ] ⎤ = ⎡ qn + r ⎤ = ⎡ q + r ⎤ = q .
⎢⎣ n ⎥⎦
n ⎦⎥
⎣⎢ n ⎦⎥
⎣⎢
Außerdem gilt
⎡ [ x ] + { x } ⎤ = ⎡ qn + r + { x } ⎤ = ⎡ q + r + { x } ⎤ = q .
⎡x ⎤
⎥⎦
⎢⎣
⎥⎦
⎢⎣
⎥⎦
⎢⎣ n ⎥⎦ = ⎢⎣
n
n
n
Satz 2.3 (2.2)
Seien n ∈ N und p ∈ P . Dann gilt:
∞
ep ( n ! ) =
n
∑ ⎡⎢⎣ p j ⎤⎥⎦ .
j =1
Beweis
Für beliebige m ∈ N gilt, dass genau [ mn ] Zahlen aus der Menge { 1,...,n } durch m teilbar sind. Also sind
unter den Zahlen 1,...,n genau ⎡ pnj ⎤ teilbar durch p j und es gilt:
⎣ ⎦
n
n ∞
∞ n
∞
n
(
ep n ! ) = ∑ ep ( k ) = ∑ ∑ 1 = ∑ ∑ 1 = ∑ ⎡⎢ j ⎤⎥ .
k =1
k =1 j =1
j =1 k =1
j =1 ⎣ p ⎦
p j |k
Konvention: Ab jetzt schreiben wir p für p ∈ P .
p j |k
Kapitel 2
Zahlentheoretische Funktionen
Beispiel 2.4
Wenden wir die obige Formel auf p = 3 und n = 11 an, so erhalten wir:
⎡ 11 ⎤ ⎡ 11 ⎤ ⎡ 11 ⎤
e3 ( 11! ) = ⎢ ⎥ + ⎢ ⎥ + ⎢ ⎥ + ... = 3 + 1 + 0 + ... = 4 .
⎣ 3 ⎦ ⎣ 9 ⎦ ⎣ 27 ⎦
Korollar 2.5 (2.3)
(i) Es gilt:
n ⎤
ep ( n ! ) ≤ ⎡⎢
⎥.
⎣ p − 1⎦
(ii) Für 1 ≤ n ≤ m ist der Binomialkoeffizient
( mn ) := n ! mm−! n
!
eine natürliche Zahl. Für m = 0 definieren wir den Binomialkoeffizienten durch 0.
Beweis
(i) Es gilt
∞
∞
∞
⎛ ∞ 1 j
⎞
n
n
1 j
n
⎛ 1
⎞⎟
ep ( n ! ) = ∑ ⎢⎡ j ⎥⎤ ≤ ∑ j = n ∑
= n ⎜⎜⎜ ∑ j = 0
− 1 ⎟⎟⎟ = n ⎜⎜
⎟ = p −1
1 − 1⎟
⎝
⎠
p
p
1
−
p
p
⎝
⎠
p
⎦
j =1 ⎣
j =1
j =1
(
( )
und da ep ( n ! ) ∈ N 0 ist, folgt
)
( )
n ⎤
ep ( n ! ) ≤ ⎡⎢
⎥.
p
⎣ − 1⎦
(ii) Für festes p ∈ P gilt
m
m!
ep
= ep
= ep ( m ! ) − ep ( n ! ) − ep ( ( m − n ) ! ) =
n
n !(m − n )!
(( ) )
(
)
∞
m
n
∑ ( ⎡⎢⎣ p j ⎤⎥⎦ − ⎡⎢⎣ p j ⎤⎥⎦ − ⎡⎢⎣
j =1
m −n⎤
≥ 0,
p j ⎥⎦
)
da wir das Argument der Summe als [ x + y ] − [ x ] − [ y ] schreiben können und nach Lemma 2.2 (iii) die
ses Argument nichtnegativ ist.
Eine zahlentheoretische Funktion (auch arithmetische Funktion) ist eine Funktion f mit Definitionsbereich
N , z.B. f : N → R , f : N → C , d.h. reelle bzw. komplexe Folge.
Definition 2.6
Eine zahlentheoretische Funktion heißt multiplikativ, falls für alle m, n ∈ N mit ( m, n ) = 1 gilt:
f (m ⋅ n ) = f (m ) ⋅ f (n ) .
Sie heißt total multiplikativ, falls die Bedingung für alle m, n ∈ N erfüllt ist.
Lemma 2.7 (2.4)
Sei f multiplikativ und nicht die Nullfunktion. Dann gilt:
(i) f ( 1 ) = 1 ,
(ii) f ( p1e1 ⋅ ... ⋅ pkek ) = f ( p1e1 ) ⋅ ... ⋅ f ( pkek ) .
Beweis
(i) Es gilt: f ( 1 ) = f ( 1 ⋅ 1 ) = f ( 1 ) ⋅ f ( 1 ) . Dann haben wir zwei Fälle:
1. Fall: f ( 1 ) ≠ 0 . Dann folgt offensichtlich f ( 1 ) = 1 .
2. Fall: f ( 1 ) = 0 . Dann folgt:
f (n ) = f (n ⋅ 1) = f (n ) ⋅ f (1) = f (n ) ⋅ 0 = 0 ,
ist also die Nullfunktion, was unserer Voraussetzung widerspricht.
(ii) Wegen pi ≠ p j für i ≠ j gilt:
f ( p1e1 ⋅ ... ⋅ pkek ) = f ( p1e1 ( p2e2 ⋅ ... ⋅ pkek ) ) = f ( p1e1 ) ⋅ f ( p2e2 ⋅ ... ⋅ pkek ) = ... = f ( p1e1 ) ⋅ ... ⋅ f ( pkek ) .
Definition 2.8
Zu einer zahlentheoretischen Funktion f heißt
g ( n ) :=
∑ f (d )
d ∈N
d |n
die zugehörige summatorische Funktion.
10
Kapitel 2
Zahlentheoretische Funktionen
In Zukunft bedeutet die Summation über d|n nur die Summation über die positiven Teiler von n .
Lemma 2.9 (2.5)
Mit f ist auch die summatorische Funktion von f multiplikativ.
Beweis
Aus ( m, n ) = 1 und damit m = p1e1 ⋅ ... ⋅ pkek und n = q1r1 ⋅ ... ⋅ qlrl mit pi ≠ q j für alle i und j folgt also, dass
d die Gestalt
d = p1a1 ⋅ ... ⋅ pkak ⋅ q1b1 ⋅ ... ⋅ qlbl mit 0 ≤ ai ≤ ei und 0 ≤ bi ≤ ri
=d1
=d2
hat und somit d = d1 ⋅ d2 gilt, wobei d1|m und d2 |n . Also gilt:
g ( m ⋅ n ) = ∑ ∑ f ( d1 ⋅ d2 ) = ∑ ∑ f ( d1 ) ⋅ f ( d2 ) =
d1 |n d2 |n
=
( ∑d
1
d1 |m d2 |n
f ( d1 )
|m
)( ∑d
2
∑ ( f (d1 ) ⋅ ∑d |n f ( d2 ) )
2
d1 |m
)
f ( d2 ) = g ( m ) ⋅ g ( n ) .
|n
Beispiel 2.10
Multiplikative zahlentheoretische Funktionen sind: f ( n ) = n , f ( n ) = 1 , f ( n ) = n α , nicht aber f ( n ) = 2n ,
da f ( 1 ) ≠ 1 . Die drei Funktionen sind sogar total multiplikativ.
Definition 2.11
Die Funktion
∑
ϕ ( n ) :=
1
1≤m ≤n
( m ,n )=1
heißt Eulersche ϕ -Funktion.
Beispiel 2.12
Hier stehen die ersten 18 Werte der ϕ -Funktion:
n
1
2
3
4
5
6
7
8
9
10
11
12
13
14
15
16
17
18
ϕ (n )
1
1
2
2
4
2
6
4
6
4
10
4
12
6
8
8
16
6
Es ist offensichtlich, dass ϕ ( p ) = p − 1 gilt.
Lemma 2.13 (2.6)
Für p ∈ P und k ∈ N gilt:
ϕ ( pk ) = pk − pk −1 .
Beweis
Es gilt:
ϕ ( pk ) =
∑
1−
1≤m ≤ pk
∑
1 = pk −
1≤m ≤ pk
( m,pk )>1
⎡ pk
1 = pk − ⎢
⎢⎣ p
1≤m ≤ pk
∑
⎤
⎥ = pk − pk −1 .
⎥⎦
p| m
Zum Beweis der Multiplikativität von ϕ gehen wir einen Umweg: Wir zeigen die Gültigkeit der Formel, die
im Falle der Multiplikativität von ϕ gelten würde:
ϕ ( n ) = ϕ ( p1e1 ) ⋅ ... ⋅ ϕ ( pkek ) = ( p1e1 − p1e1 −1 ) ⋅ ... ⋅ ( pkek − pkek −1 )
1
1
1
1
1
= p1e1 1 −
⋅ ... ⋅ pkek 1 −
= n 1−
⋅ ... ⋅ 1 −
= n∏ 1 − .
p1
pk
p1
pk
p
p |n
(
)
(
) (
) (
)
(
)
Gilt diese Formel, so sind wir fertig. Also müssen wir nur noch den nächsten Satz beweisen.
Satz 2.14 (2.7)
Für n ∈ N gilt:
(
ϕ ( n ) = n∏ 1 −
p| n
)
1
.
p
Beweis
Seien p1,..., pk alle Primfaktoren von n . Durch Ausmultiplizieren des Produkts erhalten wir
11
Kapitel 2
Zahlentheoretische Funktionen
(
P := n ∏ 1 −
p ∈P
p |n
k
)
(
k
)
1
1
n
n
n
= n∏ 1 −
=n−∑
+ ∑
− ... + ( −1 )k
.
⋅
⋅ pk
p
p
p
p
p
p
...
1
i
i =1
1≤r <s ≤k r s
r =1 r
Nun ist pnr die Anzahl der m ≤ n , die durch pr teilbar sind,
teilbar sind, usw. Also folgt:
n
P =
∑1−
m =1
n
=
∑
n
∑1+
1≤r ≤k m =1
pr |m
n
∑1− ∑ ∑
m =1
1+
m =1 1≤r ≤k
pr |m
n
pr ps
n
∑ ∑
1≤r <s ≤k m =1
pr ps |m
n
∑ ∑
n
∑
1 − ... + ( −1 )k
1≤r1 <...<rk ≤k
n
1 − ... + ( −1 )k
m =1 1≤r <s ≤k
pr ps |m
∑
∑
1
m =1
pr1 ⋅...⋅ prk |m
∑
1
m =1 1≤r1 <...<rk ≤k
pr1 ⋅...⋅ prk |m
n
=
die Anzahl der m ≤ n , die durch pr ps
∑ ( 1 − ∑1≤p r|m≤k 1 + ∑1≤prp<s|m≤k 1 − ... + ( −1 )k ∑1≤pr ⋅<......⋅p<r|m≤k 1 ).
m =1
r
1
r1
r s
k
rk
Da die Summen im Argument je die Anzahlen der p1,..., pk ( l ( m ) ) bzw. der pr ps Paare (
stellen, die m teilen, gilt:
n
l (m )
l (m )
l (m )
l (m )
.
−
+
− ... + ( −1 )k
P = ∑ 1 − l (m ) +
2
3
4
k
m =1
(
(
) (
) (
)
(
( l (m2 ) ) ) usw. dar-
))
1. Fall: l ( m ) = 0 : Das Argument der Summe ist dann offensichtlich 1.
2. Fall: l ( m ) > 0 : Nach dem Binomischen Lehrsatz gilt für das Argument der Summe:
1 − l (m ) +
(
)
(
)
l (m )
l (m )
− ... + ( −1 )k
=
k
2
Also gilt l ( m ) = 0 für ( m, n ) = 1 und damit:
P =
∑
k
∑(
r =0
)
l ( m ) ( )r k −r
−1 ⋅ 1
= ( 1 − 1 )k = 0 .
r
1 = ϕ (n ) .
1≤m ≤n
( m ,n )=1
Korollar 2.15 (2.8)
Es gilt:
(i) ϕ ist multiplikativ,
(ii) für n ∈ N gilt:
∑ ϕ (d ) = n .
d |n
Beweis
(i) Seien m, n ∈ N mit ( m, n ) = 1 . Dann gilt mit Satz 2.14:
1
1
1
1−
ϕ ( mn ) = mn ∏ 1 −
= mn ∏
= mn ∏ 1 −
p
p
p
p|mn
p|m oder p|n
p |m
1
1
= m ∏ p| m 1 −
n ∏ p |n 1 −
= ϕ ( m ) ϕ ( n ).
p
p
(
(
)
(
(
))(
)
(
))
(
)∏(1 − p1 )
p| n
(ii) Wir wissen, dass nach Lemma 2.9 mit ϕ auch die summatorische Funktion von ϕ multiplikativ ist, d.h.
g ( n ) = ∑ ϕ (d )
d |n
ist multiplikativ. Also folgt
g ( n ) = g ( p1e1 ⋅ ... ⋅ pkek ) = g ( p1e1 ) ⋅ ... ⋅ g ( pkek ) ,
wobei für g ( pe ) gilt:
g ( pe ) =
∑ ϕ (d ) =
e
d |p
e
∑ ϕ ( p j ) = 1 + ( p − 1) + ( p2 − p ) + ... + ( pe − pe −1 ) = pe .
j =0
Also erhalten wir:
k
g ( n ) = ∏ piei = n .
i =1
Definition 2.16
Wir sagen zu einer natürlichen Zahl n , sie sei quadratfrei, falls aus p|n stets p 2 ? n folgt. Eine weitere wichtige zahlentheoretische Funktion ist die Möbiussche μ -Funktion:
12
Kapitel 2
Zahlentheoretische Funktionen
1,
für n = 1,
⎧
⎪
⎪
⎪
k
⎪
μ ( n ) = ⎨( −1 ) , für n = p1 ⋅ ... ⋅ pk quadratfrei,
⎪
⎪
0,
falls n nicht quadratfrei.
⎪
⎪
⎩
Beispiel 2.17
Hier die ersten 16 Werte der μ -Funktion:
n
μ (n )
1
1
2
−1
3
−1
4
0
5
−1
6
1
7
−1
8
0
9
0
10
1
11
−1
12
0
13
−1
14
1
15
1
16
0
Satz 2.18 (2.9)
(i) μ ist multiplikativ,
(ii) für die summatorische Funktion
ε (n ) =
∑ μ (d )
d |n
von μ gilt:
⎧⎪1,
ε ( n ) = ⎪⎨
⎪⎪⎩ 0,
für n = 1,
sonst.
Beweis
(i) Seien m, n ∈ N mit ( m, n ) = 1 . Ist m oder n nicht quadratfrei, so ist auch mn nicht quadratfrei. Also ist μ ( mn ) = μ ( m ) μ ( n ) = 0 . Sind m und n quadratfrei, so haben wir m = p1 ⋅ ... ⋅ pk mit pi ∈ P
paarweise verschieden und n = q1 ⋅ ... ⋅ ql mit q j ∈ P paarweise verschieden. Wegen ( m, n ) = 1 gilt
{ p1,..., pk } ∩ {q1,..., ql } = ∅ . Also gilt:
μ ( mn ) = μ ( p1 ⋅ ... ⋅ pkq1 ⋅ ... ⋅ ql ) = ( −1 )k +l = ( −1 )k ( −1 )l = μ ( m ) μ ( n ) .
(ii) Für n = 1 ist ε ( 1 ) = μ ( 1 ) = 1 . Für n > 1 gilt wegen der Multiplikativität von ε :
k
∏ ε ( pie
ε ( n ) = ε ( p1e1 ⋅ ... ⋅ pkek ) =
i
) = 0,
i =1
wobei
ε ( pe ) =
∑ μ (d ) = μ ( 1 ) + μ ( p ) + μ ( p2 ) + ... + μ ( pe ) = 1 + ( −1 ) + 0 + ... + 0 = 0 .
d |pe
Satz 2.19 (2.10) (1. Möbius-Umkehrung)
Sei f eine zahlentheoretische Funktion und
g (n ) =
∑ f (d )
d |n
die zugehörige summatorische Funktion. Dann gilt
f (n ) =
n
∑ μ (d ) g ( d ) .
d |n
Ist umgekehrt f durch diese Identität gegeben, so folgt
g ( n ) = ∑ f (d ) .
d |n
Beweis
Zunächst sei
g (n ) =
∑ f (d ) .
d |n
Nach Satz 2.18 gilt:
n
∑ μ ( d ) g ( d ) = ∑ ( μ ( d ) ∑d ′ |
d |n
d |n
=
∑
( f ( d ′ ) ∑d |
′
d |n
da ε ( dn′ ) = 0 für
n
d′
≠ 1 . Sei nun umgekehrt
f (n ) =
n
d
n
d′
) dd∑′ |n μ
f (d ′ ) =
(d ) f
(d ′ )
n
f (d ′ ) ε ( ) = f
) d∑
′
d
′ |n
μ (d ) =
n
n
∑ μ (d ) g ( d ) = ∑ μ ( d ′ ) g (d ′ ) .
d ′|n
d |n
Dann folgt nach Satz 2.18:
13
(
n ) ε ( 1 ) = f ( n ),
Kapitel 2
Zahlentheoretische Funktionen
n
n
μ(
g ( d ′ ) = ∑ ( g ( d ′ ) ∑d |
∑ f (d ) = ∑ f ( d ) = ∑ ∑
dd ′ )
′
′
d |n
d |n
=
∑
(
′
d |n d
g (d ′ )
d |n
da ε ( dn′ ) = 0 für
n
d′
∑d |
n
d′
|nd
n
d′
d |n
μ (d )
) = d∑′|n
g (d ′ ) ε
( dn′ ) = g
(
μ
( ddn ′ ))
n ) ε ( 1 ) = g ( n ),
≠ 1.
Satz 2.20 (2.11)
Für alle n ∈ N gilt:
ϕ (n ) = n ∑
d |n
μ (d )
.
d
Beweis
Nach Korollar 2.15 gilt:
∑ ϕ (d ) = n .
d |n
Setze f n = ϕ n , also g n = n . Aus der 1. Möbius-Umkehrung folgt:
n
n
μ (d )
ϕ ( n ) = ∑ μ (d ) g
= ∑ μ (d ) = n ∑
.
d
d
d
d |n
d |n
d |n
(
)
(
)
(
)
( )
Satz 2.21 (2.12) (2. Möbius-Umkehrung)
Seien f und g für n ∈ N reelle Funktionen mit
g (x ) =
x
∑ f (n ) .
n ≤x
Dann ist
f (x ) =
∑μ
(
n )g
n ≤x
( nx )
und es gilt die Umkehrung analog zur 1. Möbius-Umkehrung (2.19).
Beweis
Übung.
Definition 2.22
Weitere interessante zahlentheoretische Funktionen sind
(i) die Teileranzahl
τ (n ) =
∑1 .
d |n
Übliche Bezeichnung (insbesondere im Englischen): d ( n ) .
(ii) die Teilersumme
σ (n ) = ∑ d .
d |n
Satz 2.23 (2.13)
Die Funktionen τ und σ sind multiplikativ und es gilt für alle n ∈ N :
(i) τ ( n ) = ∏ (ep ( n ) + 1 ) ,
p|n
(ii) σ ( n ) = ∏
p| n
pep
n )+1
−1
.
p −1
(
Beweis
Da f ( n ) := 1 und f ( n ) := n multiplikativ sind, sind auch deren summatorische Funktionen τ ( n ) und
σ ( n ) multiplikativ. Also bleibt zu berechnen:
τ ( pe ) = ∑ 1 = # {1, p,..., pe } = e + 1 ,
d |pe
σ ( pe ) =
∑ d = 1 + p + ... + pe
d |pe
14
=
1 − pe +1
pe +1 − 1
=
.
p −1
1− p
Kapitel 2
Zahlentheoretische Funktionen
Beispiel 2.24
Es gilt:
τ ( 60 ) = τ ( 22 ⋅ 3 ⋅ 5 ) = τ ( 4 ) τ ( 3 ) τ ( 5 ) = 3 ⋅ 2 ⋅ 2 = 12 .
Definition 2.25
Seit der griechischen Antike heißt eine Zahl n vollkommen oder perfekt, falls σ ( n ) = 2n gilt.
D.h. die Summe aller Teiler von n ohne n selbst ist gleich n . Beispiele:
6 = 1 + 2 + 3 , 28 = 1 + 2 + 4 + 7 + 14 , 496 , 8128 .
Diese vier vollkommenen Zahlen waren den Griechen bekannt.
Satz 2.26 (2.14) (Satz von Euklid-Euler)
Eine gerade natürliche Zahl n ist genau dann vollkommen, wenn gilt:
n = 2p −1 ( 2p − 1 ) für p ∈ P und 2p − 1 ∈ P
2p − 1 wird die mersennesche Primzahl genannt.
Beweis
„ ⇒ “ Sei n gerade und vollkommen, d.h. n = 2k m mit k ∈ N , 2 ? m und σ ( n ) = 2n . Wegen der Multiplikativität von σ gilt mit Satz 2.23:
σ ( n ) = σ ( 2k m ) = σ ( 2k ) σ ( m ) = ( 2k +1 − 1 ) σ ( m ) = 2n = 2k +1 m .
Offenbar ist ( 2k +1 − 1, 2k +1 ) = 1 . Mit Lemma 1.22 folgt 2k +1|σ ( m ) , d.h. es gibt l ∈ N mit σ ( m ) = l 2k +1 .
Einsetzen liefert:
( 2k +1 − 1 )l 2k +1 = 2k +1 m ,
also
m = ( 2k +1 − 1 )l .
Behauptung: l = 1 . Denn nehmen wir an, l > 1 . Dann hat m die paarweise verschiedenen Teiler 1 , l und
m , da 2k +1 − 1 > 1 . Also gilt:
σ ( m ) ≥ 1 + l + m = 1 + l + ( 2k +1 − 1 )l = l 2k +1 + 1 = σ ( m ) + 1 .
Dies ist ein Widerspruch, aus dem die Behauptung folgt. Aus σ ( m ) = 2k +1 = m + 1 folgt m ∈ P . Also ist
m = 2k +1 − 1 . Es ist leicht zu zeigen, dass aus 2t − 1 ∈ P folgt: t ∈ P . Also muss k + 1 ∈ P sein und damit:
n = 2p −1 ( 2p − 1 ) .
„ ⇐ “ Sei n = 2p −1 ( 2p − 1 ) mit q := 2p − 1 ∈ P . Dann folgt:
σ ( n ) = σ ( 2p −1q ) = σ ( 2p −1 ) σ ( q ) = ( 2p − 1 ) ( q + 1 ) = q 2p = 2 ( 2p −1q ) = 2n .
Bemerkung 2.27
Falls es unendlich viele mersennesche Primzahlen gibt, so gibt es auch unendlich viele gerade vollkommene
Zahlen.
Definition 2.28
Weitere klassische Begriffe sind:
(i) eine Zahl n ∈ N heißt abundant, falls σ ( n ) > 2n ,
(ii) eine Zahl n ∈ N heißt defizient, falls σ ( n ) < 2n ,
(iii) zwei Zahlen m, n ∈ N mit m ≠ n heißen befreundet, falls σ ( m ) = σ ( n ) = m + n .
Man kann zeigen, dass es unendlich viele abundante bzw. defiziente Zahlen gibt. Es ist jedoch unbekannt, ob
es unendlich viele Paare befreundeter Zahlen gibt. Beispiel für befreundetes Paar: 220 und 284.
Satz 2.29 (2.15) (Schranken für einige zahlentheoretische Funktionen)
(i) Für alle n ≥ 2 , jedes ε > 0 und eine nur von ε abhängige Konstante c ( ε ) > 0 gilt:
2 ≤ τ (n ) ≤ c ( ε )n ε .
(ii) Für alle n ≥ 2 gilt:
n + 1 ≤ σ ( n ) ≤ n ( 1 + log ( n ) ) .
15
Kapitel 2
Zahlentheoretische Funktionen
(iii) Für alle n ≥ 2 gilt:
1 n
≤ ϕ (n ) ≤ n − 1 .
4 log ( n )
Beweis
(i) Sei ε > 0 fest gegeben. Die zahlentheoretische Funktion f mit
τ (n )
f (n ) =
nε
ist multiplikativ (als Quotient zweier multiplikativer Funktionen). Zu ε existiert ein k ( ε ) > 0 derart,
dass p ε ≥ 2 für alle p ∈ P mit p ≥ k ( ε ) ist. Wegen k + 1 ≤ 2k für alle k ∈ N gilt:
k +1
2k
2 k
≤
=
≤ 1 für alle p ≥ k ( ε ) und alle k ∈ N .
pε
p εk
p εk
Für jedes p ∈ P mit p < k ( ε ) gibt es daher höchstens endlich viele k ∈ N derart, dass kp+εk1 > 1 , denn
k +1
k +1
lim
= lim ε k = 0 .
k →∞ p εk
k →∞ ( p )
( )
Somit existieren insgesamt höchstens endlich viele Paare pi ∈ P , ki ∈ N , i = 1,..., i ( ε ) ∈ N , für die
ki +1
> 1 ist. Also folgt:
piεki
ep ( n ) + 1 i ε ki + 1
τ ( pep n )
≤ ∏ εki =: c ( ε ) .
f ( n ) = f ( ∏ p|n pep n ) = ∏ f ( pep n ) = ∏ εep n = ∏ εep n
p
p| n
p| n p
p| n
i =1 pi
(
(
)
(
(
)
)
)
)
(
(
)
(ii) Es gilt:
σ (n ) =
n
∑d = ∑ d
d |n
d |n
= n∑
d |n
1
1
≤ n∑ .
d
d ≤n d
und wegen
n1
n 1
1
≤1+ ∫
dx = 1 + [ log x ]1n = 1 + log n
∫1 x dx ≤ d∑
1 x
d
≤n
gilt
n log n ≤ σ ( n ) ≤ n ( 1 + log n )
für n ≥ 2 .
(iii) Die Funktion
σ (n )ϕ (n )
n2
ist multiplikativ als Produkt und Quotient multiplikativer Funktionen. Nach Satz 2.23 und Lemma 2.13
gilt für p ∈ P und k ∈ N
σ ( pk ) ϕ ( pk )
1 pk +1 − 1 k
1
pk +1 − 1
1
1
=
−
=
= 1 − k +1 .
f ( pk ) =
p
k
2k
2k
+
1
p −1
p
p
p
p
p
Also folgt:
∞
1
1 ⎞
1
1
⎛
⎞
⎛
f ( n ) = ∏ f ( pep n ) = ∏ ⎜⎜ 1 − ep n +1 ⎟⎟ ≥ ∏ ⎜⎜ 1 − 2 ⎟⎟ ≥ ∏ 1 − 2 = ,
⎟
⎝
⎠
2
⎝
⎠
p
m
p
p |n
p |n
p |n
m =2
f ( n ) :=
(
(
)
)
(
)
(
)
denn
M
∏(
m =2
1−
)
1
=
m2
M
∏(
m =2
1−
)
M
(
) (
1
1
1 2
M −1
1+
⋅ ... ⋅
⋅
=
m m∏
m
3
M
2
=2
)( 23 43 ⋅ ... ⋅ MM+ 1 ) = 21 MM+ 1 → 21
für M → ∞ . Mit (ii) folgt für n ≥ 3
1 2
n2 f (n )
n
n
2n
ϕ (n ) =
≥
=
≥
,
n ( 1 + log n ) 2 ( 1 + log n ) 4 log n
σ (n )
da dann 1 + log n ≤ 2 log n gilt. Für n = 2 ist die Behauptung trivialerweise erfüllt.
Da viele zahlentheoretische Funktionen stark schwanken, versuchen wir nun ihren Mittelwert zu berechnen:
Ist f ( n ) eine zahlentheoretische Funktion, so ist der Mittelwert
1
f (n ) = H ( x ) + F ( x )
x n∑
≤x
mit einem dominanten Hauptterm H ( x ) und einem vergleichsweise kleinen Fehlerterm F ( x ) . Dazu definieren wir die so genannten Landau-Symbole O ( ⋅ ) und o ( ⋅ ) :
16
Kapitel 2
Zahlentheoretische Funktionen
Für zwei reelle (oder komplexe) Funktionen f ( x ) , g ( x ) sagen wir f ( x ) = O ( g ( x ) ) bzw. f ( x )
nau dann, wenn f ( x ) ≤ cg ( x ) für eine feste positive Konstante c und alle x ∈ R gilt.
Wir schreiben f ( x ) = o ( g ( x ) ) genau dann, wenn für x → ∞ gilt:
f (x )
→ 0.
g (x )
g ( x ) ge-
Satz 2.30 (2.16)
Für x → ∞ gilt:
(i)
∑τ
(
n ) = x log x + O ( x ) .
n ≤x
Dabei wächst x log x schneller als O ( x ) . Äquivalent gilt:
1
τ ( n ) = log x + O ( 1 ) .
x n∑
≤x
(ii)
∑σ
(
n) =
π2 2
x + O ( x log x ) .
12
∑ϕ
(
n) =
3 2
x + O ( x log x ) .
π2
n ≤x
(iii)
n ≤x
Beweis
(i) Mit Lemma 2.2 (i) gilt:
∑τ
(
n) =
n ≤x
x
x
x
1
∑ ∑ 1 = ∑ ∑ 1 = ∑ ⎡⎢⎣ d ⎤⎥⎦ = ∑ ( d + O ( 1 )) = ∑ d + ∑ O ( 1 ) = x ∑ d + O ( ∑d ≤x 1 )
n ≤x d |n
d ≤x n ≤x
d |n
d ≤x
d ≤x
d ≤x
d ≤x
d ≤x
= x ( log x + O ( 1 ) ) + O ( x ) = x log x + O ( x ) + O ( x ) = x log x + O ( x ) ,
wobei wir x log x ≤ x ∑d ≤x d1 ≤ x ( 1 + log x ) verwendet haben.
(ii) Es gilt, wobei wir m := dn nach dem vierten Gleichzeichen und
M
1
∑ m = 2 M 2 + O (M )
m =1
verwenden:
∑σ
(
n) =
n ≤x
n
∑ ∑d = ∑ ∑ d
n ≤x d |n
=
n ≤x d |n
n
∑∑d
=
d ≤x n ≤x
d |n
dm
=∑ ∑ m=
d ≤x m ≤ x d
d ≤x m ≤[ x ]
∑∑
d
d
∑ ( 2 ⎡⎢⎣ d ⎤⎥⎦
1 x
2
d ≤x
2
1
x
1
x
⎡ x ⎤2 + O
= ∑
+ O ( 1 ) + O ( x log x )
∑
∑
⎢
⎥
d
≤
x
2 d ≤x ⎣ d ⎦
d
2 d ≤x d
2
⎛
⎞
1
x
x
= ∑ ⎜⎜⎝ 2 + 2 O ( 1 ) + O ( 1 )2 ⎠⎟⎟ + O ( x log x )
2 d ≤x d
d
2
x
x
1
= ∑ 2 + O ∑d ≤x
+ O ( ∑d ≤x 1 ) + O ( x log x )
d
2 d ≤x d
1
1
1
1
= x 2 ∑ 2 + O ( x log x ) + O ( x ) + O ( x log x ) = x 2 ∑ 2 + O ( x log x ),
2 d ≤x d
2 d ≤x d
(
=
)
(
(
)
)
wobei
∞
( )
1
1
1
1
π2
1
∑ d2 = ∑ d2 = ∑ d2 − ∑ d2 = 6 + O x .
d ≤x
d≤ x
d =1
d> x
[
]
[
Dabei ist
∑
d >[ x ]
Also folgt:
∑σ
n ≤x
(
n) =
1
≈
d2
(
∞
∫x
[
]
]
( )
1
1
1
⎡ 1 ⎤∞
=
− ⎥ =
=O
dt
.
⎢
2
[
]
⎣ t⎦x
x
x
t
[
]
( )) + O ( x log x ) = π12 x
1 2 π2
1
x
+O
2
6
x
(iii) Es gilt nach Satz 2.20, wobei wir wieder m :=
n
d
2
verwenden:
17
2
+ O ( x log x ) .
+O
( dx ) )
Kapitel 2
Zahlentheoretische Funktionen
∑ϕ
(
n) =
n ≤x
∑ n∑
n ≤x
=
d |n
μ (d )
=
d
n
∑ μ (d ) ∑ d
d ≤x
1 2
μ (d )
x
2 +O
2 d∑
≤x d
=
n ≤x
d |n
(∑
d ≤x
∑ μ (d ) ∑ m = ... = ∑ μ (d )( 2 ⎡⎢⎣ d ⎤⎥⎦
1 x
m ≤dx
d ≤x
2
+O
d ≤x
μ (d )
x
.
d
μ (d )
x
x
1
≤ ∑ = x ∑ = O ( x log x ) .
d d ≤x d
d ≤x d
( dx ))
)
Für den Fehlerterm gilt:
x
∑ μ (d ) d
∑
≤
d ≤x
d ≤x
Für den Hauptterm gilt:
∞
μ (d )
μ (d )
μ (d )
∑ d2 = ∑ d2 − ∑ d2 ,
d ≤x
d =1
d >x
wobei
μ (d )
=O
2
d >x d
∑
(∑
d >x
)
1
=O
d2
(∫
∞
x
)
( )
1
1
2 dt = O x .
t
Wir haben Glück und können auch die erste Summe des Hauptterms berechnen. Es gilt:
∞ ∞
∞ ∞
∞
∞ ∞
∞
μ ( d1 )
μ ( d1 )
μ ( d1 )
1⎞
∞
⎛ ∞ μ ( d1 ) ⎞⎛
⎟
⎟
⎜∑
⎜
=
=
=
∑
∑
∑
∑
∑
∑
∑
∑
∑
⎟
2
2
⎟ ⎜ d2 =1 d22 ⎠⎟⎟
⎜⎝ d1 =1 d12 ⎠⎝
n2
d1 =1 d2 =1 ( d1d2 )
n =1 d1 =1 d2 =1 ( d1d2 )
n =1 d1 =1 d2 =1
∞
=
∞
1
∑ n2 ∑ ∑
n =1
∞
=
d1d2 =n
∞
∞
μ ( d1 ) =
d1 =1 d2 =1
d1d2 =n
1
∞
n
1
n =1
1=
d2 =1
d1d2 =n
d1 |n
n
∑ n2 ∑ ∑
n =1
∑ n 2 ∑ μ (d1 ) ∑
d1d2 =n
n
μ ( d1 )
d1 =1 d2 =1
d1d2 =n
1
n
∞
d2 =1
d2 = dn1
n =1
1
∑ n 2 ∑ μ (d1 ) ∑ 1 = ∑ n 2 ε
n =1
d1 |n
(
n ) = 1,
wobei das Umordnen der Reihen erlaubt ist, da die beiden Summen am Anfang absolut konvergent sind.
Die vorletzte Summe ergibt 1, da ε ( n ) die summatorische Funktion von μ ( n ) ist, die 0 für n > 1 und
1 für n = 1 liefert. Also folgt:
( )
∞
μ (d )
π2
∑ d2 = 6
d =1
Insgesamt folgt:
∑ϕ
(
n) =
n ≤x
(
−1
=
6
.
π2
( )) + O ( x log x ) = π3 x
1 2 6
1
x
2 +O x
2
π
2
2
+ O ( x log x ) .
Korollar 2.31 (2.17)
Die Wahrscheinlichkeit dafür, dass zwei zufällig gewählte Zahlen m, n ∈ N teilerfremd sind, ist
6
.
π2
Beweis
Seien 1 ≤ m ≤ n ≤ x für ein hinreichend großes x . Wir bezeichnen die Anzahl der möglichen Fälle mit
x
n
x
1
1
# mF := ∑ 1 = ∑ ∑ 1 = ∑ n = [ x ]( [ x ] + 1 ) = x 2 + O ( x )
2
2
1≤m ≤n ≤x
n =1 m =1
n =1
und die Anzahl der günstigen Fälle mit
# gF :=
∑
x
1=
1≤m ≤n ≤x
( m ,n )=1
n
∑ ∑
n =1
x
1=
m =1
∑ϕ
(
n) =
n =1
( m ,n )=1
3 2
x + O ( x log x ) .
π2
Es gilt:
# gF
=
# mF
3
π2
x 2 + O ( x log x )
1 2
2x
+ O (x )
für x → ∞ .
→
3 x2
π2
1 2
2x
=
6
π2
18
19
Kapitel 3
Kongruenzen
Definition 3.1
Seien a, b ∈ Z , m ∈ N . Wir sagen: a ist kongruent zu b modulo m (geschrieben: a ≡ b mod m ), falls
m|b − a , d.h. a und b haben bei Division mit m denselben Rest. Wir nennen m Modul der Kongruenz
a ≡ b mod m .
Es ist ≡ mod m eine Äquivalenzrelation auf Z . Denn es gilt:
a ≡ a mod m , aus a ≡ b mod m folgt b ≡ a mod m , aus a ≡ b mod m und b ≡ c mod m folgt a ≡ c mod m .
Letzteres gilt, da aus m|b − a und m|c − b folgt: m|( c − b ) + (b − a ) = c − a . Wir nennen die Äquivalenzklassen Restklassen mod m , die die folgenden Mengen repräsentieren:
[ 0 ]m := { km | k ∈ Z } , [ 1 ]m := { km + 1 | k ∈ Z } , …, [ m − 1 ]m := { km + ( m − 1 ) | k ∈ Z } .
Wir schreiben häufig 0,1,..., m − 1 mod m . Unter einem vollen Restsystem mod m verstehen wir eine Menge
von m ganzen Zahlen mit einer aus jeder Restklasse mod m , also a1,..., am Reste mod m mit ai ≡ a j (inkongruent) für i ≠ j .
Beispiel 3.2
Volles Restsystem mod 7 :
0,1, 2,..., 6 oder 0, −6, 9, 3, 4,19, −1 .
Satz 3.3 (3.1)
Sei m ∈ N . Die Menge der Restklassen mod m bildet mit den Verknüpfungen [ a ]m ± [ b ]m := [ a ± b ]m und
[ a ]m ⋅ [ b ]m := [ a ⋅ b ]m einen Ring, den so genannten Restklassenring mod m .
Beweis
Die Ringeigenschaften ergeben sich direkt aus den Ringeigenschaften von Z , z.B. das Distributivgesetz:
( [ a ]m + [ b ]m ) [ c ]m = [ a + b ]m [ c ]m = [ ( a + b ) c ]m = [ ac + bc ]m = [ ac ]m + [ bc ]m = [ a ]m [ c ]m + [ b ]m [ c ]m .
Es bleibt ein Problem:
[ 3 ]7 [ 5 ]7
= [ 3 ⋅ 5 ]7 = [ 15 ]7 = [ 1 ]7 = [ −4 ]7 [ 12 ]7 = [ −48 ]7 = [ 1 ]7 ,
die Wohldefiniertheit (d.h. Rechnungen mod m hängen nicht von den gewählten Repräsentanten ab):
Seien [ a ]m = [ a ′ ]m und [ b ]m = [ b ′ ]m , d.h. m|a − a ′ und m|b − b ′ . Somit folgt:
m|( a − a ′ ) ± (b − b ′ ) = ( a ± b ) − ( a ′ ± b ′ ) ,
d.h. [ a ± b ]m = [ a ′ ± b ′ ]m und m|( a − a ′ )b + a ′ (b − b ′ ) = ab − a ′b ′ und damit [ ab ]m = [ a ′b ′ ]m .
Korollar 3.4 (3.2)
Ist f ∈ Z [ x ] , so folgt aus a ≡ a ′ mod m , dass auch f ( a ) ≡ f ( a ′ ) mod m gilt.
Beweis
Da [ a ]m = [ a ′ ]m und f ( a ) bzw. f ( a ′ ) nur durch wiederholte Anwendung der Operationen + , − und ⋅
entstehen, erhalten wir am Ende dieselben Restklassen und damit folgt die Behauptung.
Lemma 3.5 (3.3)
Seien m, k ∈ N , a, a ′, a1,..., am ∈ Z . Dann gilt:
m
(i) Es ist a ≡ a ′ mod
genau dann, wenn ka ≡ ka ′ mod m .
( k, m )
(ii) Für ( k, m ) = 1 ist a ≡ a ′ mod m genau dann, wenn ka ≡ ka ′ mod m .
(iii) Für ( k, m ) = 1 ist mit a1,..., am auch ka1,..., kam ein volles Restsystem mod m .
Kapitel 3
Kongruenzen
Beweis
(i) „ ⇒ “ Sei
a ≡ a ′ mod
Dann gilt:
m
( k, m )
Wegen ( k, m )|k folgt
m=
m
( k, m )
m
( k, m )
.
|a − a ′ .
( k , m )|( a − a ′ ) k ,
d.h. ka ≡ ka ′ mod m .
„ ⇐ “ Sei ka ≡ ka ′ mod m . Dann gilt m|( ka − ka ′ ) = k ( a − a ′ ) . Also folgt:
m k (a − a ′ )
|
.
( k, m ) ( k, m )
Wegen
m
k
,
=1
( k, m ) ( k, m )
gilt nach Lemma 1.22
m
|a − a ′ ,
( k, m )
d.h.
m
a ≡ a ′ mod
.
( k, m )
(ii) Folgt sofort aus (i).
(iii) Sei a1,..., am ein volles Restsystem mod m . Dann folgt ai ≡ a j mod m für i ≠ j . Wegen ( k, m ) = 1
folgt aus (ii) kai ≡ ka j mod m für i ≠ j . Also sind die m Zahlen ka1,..., kam paarweise inkongruent
mod m . Und daraus folgt die Behauptung.
(
)
Beispiel 3.6
Ein volles Restsystem mod 6 ist z.B. 0, −5, 8, 3, −2,17 . Für k = 9 gilt 0, −45, 72,27, −18,153 ≡ 0, 3, 0, 3, 0, 3
und für k = 5 erhalten wir ein volles Restsystem 0, −25, 40,15, −10, 85 ≡ 0, 5, 4, 3,2,1 .
Definition 3.7
Die Kongruenz ax ≡ b mod m nennen wir die lineare Kongruenz. Wir sagen, es gilt ax ≡ b mod m , wenn es
ein x 0 ∈ Z gibt, so dass m |ax 0 − b , d.h. die lineare Kongruenz ist lösbar. Ansonsten sagen wir ax ≡ b mod m ,
d.h. sie ist nicht lösbar.
Satz 3.8 (3.4)
Seien m ∈ N und a, b ∈ Z . Die lineare Kongruenz ax ≡ b mod m ist genau dann lösbar, wenn (a, m )|b . In
diesem Fall existieren genau (a, m ) verschiedene Lösungen mod m .
Beweis
O.B.d.A. sei a ∈ N (sonst multipliziere die Kongruenz mit −1 ( a < 0 ) oder die Aussage ist trivial ( a = 0 )).
„ ⇒ “ Sei x eine Lösung von ax ≡ b mod m . Dies ist äquivalent zu m|ax − b , d.h. es gibt c ∈ Z mit
cm = ax − b . Also gilt b = ax − cm . Aus dem Satz über die Lösbarkeit linearer diophantischer Gleichungen
(Satz 1.11) folgt nun die Behauptung.
„ ⇐ “ Sei d = ( a, m ) und gelte d|b . Wir setzen a ′ = da , m ′ = md , b ′ = db . Die zu lösende Kongruenz, d.h. die
zu lösende diophantische Gleichung ax − b = mc ist äquivalent zu a ′x − b ′ = m ′c . Dies ist wiederum äquivalent zu a ′x ≡ b ′ mod m ′ . Wegen ( a ′, m ′ ) = 1 durchläuft mit x nach Lemma 3.5 (ii) auch a ′x ein volles Restsystem mod m ′ . Also existiert (genau) eine Restklasse x 0 mod m ′ mit a ′x 0 ≡ b ′ mod m ′ . Dieses x 0 löst auch
die ursprüngliche Kongruenz. Wir wählen o.B.d.A. 0 ≤ x 0 < m ′ .
Für k ∈ Z sind auch alle Zahlen der Form x 0 + m ′k Lösungen der reduzierten Kongruenz (trivialerweise).
Bezüglich des Moduls m können diese Zahlen aber inkongruent sein. Wir erhalten 0 ≤ x 0 + m ′k < m . Das
erfüllen genau die Zahlen k ∈ Z mit
m
0≤k <
.
m′
Das sind genau mm′ zulässige Werte für k , d.h. es gibt genau mm′ = d = ( a, m ) Lösungen mod m .
20
Kapitel 3
Kongruenzen
Beispiel 3.9
Für a = 3 , b = 6 und m = 9 haben wir:
3x ≡ 6 mod 9 mit ( a, m ) = 3 ⇐⇒ x ≡ 2 mod 3 mit a ′ = 1 , b ′ = 2 , m ′ = 3 .
Für die Ausgangskongruenz haben wir x = 2 + 3k mit k ∈ Z . Es folgt 0 ≤ 2 + 3k < 9 und damit k = 0,1,2 .
Also lösen die Restklassen 2, 5, 8 mod 9 die Ausgangskongruenz.
Bemerkung 3.10
Der Restklassenring mod m , d.h. Zm := ( {[ 0 ]m ,..., [ m − 1 ]m } , +, ⋅ ) , ist im Allgemeinen kein Körper, denn
Zm enthält i.A. Nullteiler. Für alle m ∉ P mit m ≠ 1 existieren offenbar Nullteiler. Also ist Zm kein Körper. Demgegenüber gilt: Z p ist ein Körper für alle p ∈ P . Denn es existieren alle multiplikativen Inversen.
Behauptung: Für alle a ≡ 0 mod p gilt ax ≡ 1mod p .
Nach Satz 3.8 ist die lineare Kongruenz genau dann lösbar, wenn ( a, p )|1 . Da dies der Fall ist, sind also alle
Fp := Z p endliche Körper mit p Elementen.
Satz 3.11 (3.5) (Der chinesische Restsatz)
Seien m1,..., mk ∈ N paarweise teilerfremd, d.h. ( mi , m j ) = 1 für alle i ≠ j . Sind c1,..., ck ∈ Z beliebig, so
besitzt das (lineare) Kongruenzsystem
x ≡ c1 mod m1
x ≡ ck mod mk
eine eindeutige Lösung mod m mit m := m1 ⋅ ... ⋅ mk .
Beweis
Existenz: Wir setzen m j′ = mmj für 1 ≤ j ≤ k . Da die m j paarweise teilerfremd sind, gilt ( m j , m j′ ) = 1 . Nach
Satz 3.8 existieren Zahlen x j ∈ Z mit
m j′x j ≡ c j mod m j für 1 ≤ j ≤ k .
Wir setzen x = m1′x1 + ... + mk′ x k . In der Tat gilt nach Konstruktion (da mi |m j′ für i ≠ j ):
x = m1′x1 + ... + m j′x j + ... + mk′ x k ≡ m j′x j ≡ c j mod m j für 1 ≤ j ≤ k .
Eindeutigkeit: Sind x und y zwei Lösungen von , so gilt x ≡ c j ≡ y mod m j für 1 ≤ j ≤ k . Damit folgt:
Für alle j ∈ N gilt m j |x − y und wegen ( mi , m j ) = 1 für i ≠ j folgt m = m1 ⋅ ... ⋅ mk |x − y . Also gilt
x ≡ y mod m .
Korollar 3.12 (3.6)
Seien m1,..., mk paarweise teilerfremd, a j , bj ∈ Z für 1 ≤ j ≤ k . Das Kongruenzsystem
a1x ≡ b1 mod m1
ak x ≡ bk mod mk
ist genau dann lösbar, wenn jede Einzelkongruenz lösbar ist, d.h. ( a j , m j )|bj für 1 ≤ j ≤ k .
Beweis
„ ⇒ “ Sei lösbar. Dann ist jede Einzelkongruenz lösbar und nach Satz 3.8 gilt ( a j , m j )|bj für 1 ≤ j ≤ k .
„ ⇐ “ Es gelte ( a j , m j )|bj für 1 ≤ j ≤ k . Für jedes j existiert nach Satz 3.8 ein c j ∈ Z mit
a jc j ≡ bj mod m j . Nach dem Chinesischen Restsatz (3.11) gibt es zu diesen c j ein x ( mod m1 ⋅ ... ⋅ mk ) mit
x ≡ c j mod m j für 1 ≤ j ≤ k . In der Tat gilt a j x ≡ a jc j ≡ bj mod m j für 1 ≤ j ≤ k .
Beispiel 3.13
Sei gegeben:
6x ≡ 10 mod 4
2x ≡ 4 mod 3
3x ≡ 5 mod 7
Dann gilt: 4|6x − 10 ⇐⇒ 2|3x − 5 ⇐⇒ 3x ≡ 5 mod 2 . Wir erhalten also:
21
Kapitel 3
Kongruenzen
3x ≡ 5 mod 2
2x ≡ 4 mod 3
3x ≡ 5 mod 7
Nach einer Umformung erhalten wir für m1 := 2 , m2 := 3 , m3 := 7 :
x ≡ 5 mod m1
x ≡ 8 mod m2
x ≡ 25 mod m3
′
′
Seien m1 = m2m3 = 3 ⋅ 7 = 21 , m2 = m1m3 = 2 ⋅ 7 = 14 und m3′ = m1m2 = 2 ⋅ 3 = 6 . Es gilt:
21x1 ≡ 5 mod 2
x1 = 1
14x 2 ≡ 8 mod 3 ⇒ x 2 = 1 ⇒ x = 21 ⋅ 1 + 14 ⋅ 1 + 6 ⋅ 3 = 53 .
x3 = 3
6x 3 ≡ 25 mod 7
Definition 3.14
Neben dem so genannten vollen Restsystem mod m spielt das so genannte prime Restsystem mod m eine
wesentliche Rolle: Ein primes Restsystem mod m ist eine Menge von ϕ ( m ) Zahlen, die paarweise inkongruent mod m und teilerfremd zu m sind.
Beispiel 3.15
Sei m = 12 . Dann gilt für das prime Restsystem:
1, 2, 3, 4, 5, 6, 7, 8, 9, 10,11, 12 oder auch 13, −7,19, −1 .
Wie man sieht, sind es ϕ ( 12 ) = ϕ ( 3 ) ϕ ( 4 ) = 2 ⋅ 2 = 4 Zahlen. Insbesondere bilden die Zahlen 1 ≤ a ≤ m
mit ( a, m ) = 1 ein primes Restsystem mod m .
Lemma 3.16 (3.7)
Seien m, n ∈ N mit ( m, n ) = 1 . Durchlaufen a und b jeweils ein primes Restsystem mod m bzw. mod n , so
durchläuft an + bm ein primes Restsystem mod mn .
Beweis
Wegen ( a, m ) = (b, n ) = 1 für alle a, b folgt ( an + bm, n ) = (bm, n ) = 1 und ( an + bm, m ) = ( an, m ) = 1
und damit ( an + bm, nm ) = 1 , d.h. an + bm ist eine prime Restklasse mod mn . Nun folgt aus
an + bm ≡ a ′n + b ′m mod mn , dass a ≡ a ′ mod m und b ≡ b ′ mod n , denn aus
mn|an + bm − ( a ′n + b ′m ) = ( a − a ′ ) n + (b − b ′ ) m
folgt m|( a − a ′ ) n + (b − b ′ ) m und damit m|( a − a ′ ) n . Aus Lemma 1.22 folgt nun m|a − a ′ , d.h. es gilt
a ≡ a ′ mod m . Analog folgt aus n|( a − a ′ ) n + (b − b ′ ) m , dass b ≡ b ′ mod n ist. Da a ein primes Restsystem mod m und b ein primes Restsystem mod n durchlaufen, ist a ≡ a ′ mod m und b ≡ b ′ mod n . Also sind
die Zahlen an + bm paarweise inkongruent mod mn .
Bleibt zu zeigen, dass es alle Restklassen sind. Sei also d eine gegebene prime Restklasse mod mn . Da
( m, n ) = 1 , existieren nach Satz 1.11 Zahlen x , y mit mx + ny = 1 . Offenbar gilt dabei ( x , n ) = ( y, m ) = 1 .
Wegen ( d, n ) = ( d, m ) = 1 folgt ( xd, n ) = ( yd, m ) = 1 . Also können wir a, b wählen mit a :≡ dy mod m und
b :≡ dx mod n . Es folgt
an + bm = dyn + dxm = d ( yn + xm ) = d mod mn .
Beispiel 3.17
Seien m = 12 und n = 5 . Dann gilt:
{ 1, 5, 7,11 } ist primes Restsystem mod12 und { 1,2, 3, 4 } ist primes Restsystem mod 5 .
Weiter gilt ϕ ( 60 ) = ϕ ( 4 ⋅ 3 ⋅ 5 ) = ϕ ( 4 ) ϕ ( 3 ) ϕ ( 5 ) = 2 ⋅ 2 ⋅ 4 = 16 und:
5a + 12b : 5 ⋅ 1 + 12 ⋅ 1 = 17 , 5 ⋅ 5 + 12 ⋅ 1 = 37 , 5 ⋅ 7 + 12 ⋅ 1 = 47 , 5 ⋅ 11 + 12 ⋅ 1 =
a
a
12
12
1 5 7 11
1 5 7
5\
5\
1
17 37 47 67
1
17 37 47
und
mod
60
:
2
29 49 59 79
2
29 49 59
b
b
3
41 61 71 91
3
41 61 11
4
53 73 83 103
4
53 13 23
22
67 , ... Also folgt:
11
7
19
31
43
Kapitel 3
Kongruenzen
Satz 3.18 (3.8) (Der kleine Satz von Fermat)
Seien m ∈ N und a ∈ Z mit ( a, m ) = 1 . Dann gilt:
a ϕ m ≡ 1 mod m .
(
)
Insbesondere gilt: Ist p ∈ P und a ∈ Z mit p ? a , so gilt a p −1 ≡ 1 mod p .
Beweis
Behauptung: Mit x durchläuft auch ax ein primes Restsystem mod m , sofern ( a, m ) = 1 , denn die ax sind
paarweise inkongruent mod m , da aus ax ≡ ax ′ mod m sofort m|a ( x − x ′ ) und dann nach Lemma 1.22
m|x − x ′ und damit x ≡ x ′ mod m folgt. Also sind die ax eine prime Restklasse und es folgt die Behauptung. Damit gilt:
∏ ax ≡ ∏ x mod m .
x mod m
x mod m
( x ,m )=1
( x ,m )=1
Durch umschreiben erhalten wir
∏
Dann folgt aber
aϕ
(
∏
a
x mod m
( x ,m )=1
m)
x =
x mod m
( x ,m )=1
∏
∏
x mod m .
x mod m
( x ,m )=1
x =
x mod m
( x ,m )=1
∏
x mod m
x mod m
( x ,m )=1
und damit
aϕ
(
m)
≡ 1 mod m .
Die allgemeine Form des Satzes von Euler wurde bewiesen. Fermat formulierte seinen Satz nur für m = p .
Satz 3.19 (3.9) (Satz von Wilson)
Für eine natürliche Zahl n > 1 gilt: Es ist n ∈ P genau dann, wenn ( n − 1 ) ! ≡ −1 mod n ist.
Beweis
„ ⇒ “ Sei n = p ∈ P . Für p = 2 gilt die Behauptung. Sei also p ungerade.
Behauptung: Zu jedem a mit 0 < a < p existiert ein eindeutiges a , 0 < a < p , mit aa ≡ 1 mod p . Denn:
ax ≡ 1 mod p ist nach Satz 3.6 genau dann lösbar, wenn ( a, p )|1 . Es gibt sogar genau ( a, p ) = 1 Lösung
mod p . Dabei ist a das (eindeutige) multiplikative Inverse von a mod p .
1. Fall: a = a . Also gilt a 2 ≡ 1 mod p , d.h. p|a 2 − 1 = ( a − 1 )( a + 1 ) und damit p|a − 1 oder p|a + 1 , d.h.
a ≡ 1 mod p oder a ≡ −1 mod p . Es folgt also a = 1 oder a = p − 1 wegen 0 < a < p .
2. Fall: a ≠ a . Diese Zahlen treten also paarweise auf. Beispiel:
p = 17 : 1, 2, 3, 4, 5, 6, 7, 8, 9,10,11,12,13,14,15, 16 .
Das Produkt jedes Paares ist 1 mod17 (ohne 1 und 16 ).
Demnach können wir die Zahlen 2,..., p − 2 zu Paaren ai , ai mit ai ai ≡ 1 mod p für 1 ≤ i ≤
fassen. Also gilt:
( p − 1)! = 1 ⋅
p −3
2
zusammen-
( ∏i =1ai ai ) ⋅ ( p − 1 ) ≡ p − 1 ≡ −1mod p .
p −3
2
„ ⇐ “ Wir führen den Beweis durch Widerspruch. Sei n ∉ P . Also ist n = mk für 1 < m ≤ n − 1 und ein
k ∈ N , da n > 1 . Damit folgt m|( n − 1 ) ! und somit m ? ( n − 1 ) ! + 1 , d.h. ( n − 1 ) ! ≡ − 1mod m . Es gilt also
erst recht ( n − 1 ) ! ≡ − 1mod n .
Korollar 3.20 (3.10)
Sei p ∈ P , p > 2 . Dann ist die quadratische Kongruenz x 2 ≡ −1mod p genau dann lösbar, wenn p ≡ 1mod 4
gilt. In diesem Fall sind
p −1
x =±
!
2
Lösungen.
Beweis
„ ⇒ “ Wir führen einen Beweis durch Widerspruch. Setzen wir zuerst
p −1
r =
∈N.
2
Ist p ≡ 1 mod 4 , so gilt p ≡ 3 mod 4 . Dann gilt
(
23
)
Kapitel 3
Kongruenzen
r
x p −1 = ( x 2 ) ≡ ( −1 )r ≡ −1mod p ,
da
p − 1 ( 4k + 3 ) − 1 4k + 2
=
=
= 2k + 1 für k ∈ N .
2
2
2
Aber nach dem kleinen Satz von Fermat (3.18) gilt für p ? x : x p −1 ≡ 1mod p . Also gilt:
r =
−1 ≡ x p −1 ≡ 1mod p
und damit p = 2 , woraus ein Widerspruch folgt.
„ ⇐ “ Nach Satz von Wilson (3.19) gilt wegen 2|r für p ≡ 1mod 4
−1 ≡ ( p − 1 ) ! = 1 ⋅ ... ⋅ r ( r + 1 ) ⋅ ... ⋅ ( r + r ) mod p
≡ r ! ( −r ) ( −r + 1 ) ⋅ ... ⋅ ( −1 ) mod p
≡ r ! ( −1 )r r ! ≡ ( r ! )2 ( −1 )r mod p ≡ ( r ! )2 mod p.
Also ist r ! und damit auch −r ! Lösung.
Wir haben in Satz 3.8 die Lösbarkeit linearer Kongruenzen vollständig geklärt. Für Polynome höheren Grades ist allgemein wenig bekannt.
Satz 3.21 (3.11) (Lagrange)
Sei p ∈ P und f ∈ Z [ x ] mit führendem Koeffizienten ungleich [ 0 ]p (d.h. nicht durch p teilbar). Ist
deg f = n , so hat die Kongruenz f ( x ) ≡ 0 mod p höchstens n Lösungen mod p .
Beweis
Wir führen Induktion nach n . Sei n = 1 . Dann ist f ( x ) = ax + b mit p ? a . Nach Satz 3.8 hat die Kongruenz f ( x ) ≡ 0 mod p eine Lösung, d.h. ax ≡ −b mod p , falls ( a, p )| − b . Es existiert genau ( a, p ) = 1
n , d.h. deg f = n . Für jedes a ∈ Z gilt f ( x ) − f ( a ) = ( x − a ) g ( x ) für
Lösung mod p . Sei nun n − 1
[
]
ein Polynom g ∈ Z x mit deg g ≤ n − 1 und führendem Koeffizienten nicht durch p teilbar. Denn: Ist
f ( x ) = an x n + ... + a 0 , so gilt
f ( x ) − f ( a ) = an ( x n − a n ) + ... + a1 ( x − a )
= ( x − a ) ( an ( x n −1 + ax n −2 + ... + a n −2x + a n −1 ) + ... + a1 ) .
=:g ( x )
Ist a eine Nullstelle von f ( x ) , so gilt f ( x ) ≡ 0 mod p genau dann, wenn x ≡ a mod p (mit einer Nullstelle)
oder g ( x ) ≡ 0 mod p (mit ≤ n − 1 Nullstellen) gilt.
Bemerkung 3.22
Der Satz von Lagrange (3.21) gilt allgemein nur für Primzahlmoduln. Sogar für Primzahlpotenzen existieren
Gegenbeispiele: z.B. f ( x ) = x 2 − 1 hat 4 Nullstellen mod 8 , denn: 12 ≡ 32 ≡ 52 ≡ 72 ≡ 1 mod 8 .
Korollar 3.23 (3.12)
Sei p ∈ P und gelte d|p − 1 . Dann hat die Kongruenz x d ≡ 1mod p genau d Lösungen. (in C ist es das
Kreisteilungspolynom.)
Beweis
Die Lösungsanzahl ist ≤ d nach Lagrange. Zu zeigen bleibt also ≥ d . Zunächst hat die Kongruenz
x p −1 ≡ 1mod p
die p − 1 Lösungen 1,..., p − 1 mod p nach dem kleinen Satz von Fermat (3.18). Wegen d|p − 1 gilt:
x p −1 − 1 = ( x d − 1 ) ( x p −1−d + x p −1−2d + ... + x d + 1 ) .
Die linke Seite hat p − 1 Nullstellen und der rechte Term der rechten Seite ≤ p − 1 − d Nullstellen nach
Lagrange (3.21). Also hat x d − 1 ≥ d Nullstellen.
Bemerkung 3.24
Da Z p = Fp = {[ 0 ]p ,..., [ p − 1 ]p } ein Körper ist, entspricht das Rechnen mit Polynomen mod p dem Rechnen mit diesen Polynomen, wobei die Koeffizienten mod p abgeändert werden dürfen:
3x 2 − 17x + 15 ≡ 0 mod 7 ⇐⇒ 3x 2 + 4x + 1 = 0 in F7 .
24
Kapitel 3
Kongruenzen
Definition 3.25
Sind m ∈ N und a ∈ Z mit ( a, m ) = 1 , so heißt ordm ( a ) := min {d ∈ N | a d ≡ 1mod m } Ordnung von a
mod m . Manchmal schreiben wir auch om (a ) .
Nach dem kleinen Satz von Fermat (3.18) ist a ϕ
(
m)
≡ 1mod m , also ordm (a ) ≤ ϕ ( m ) .
Lemma 3.26 (3.13)
Ist k ∈ N mit a k ≡ 1mod m für ( a, m ) = 1 , so gilt ordm (a ) |k . Insbesondere gilt ordm ( a )|ϕ ( m ) .
Beweis
Nach der Division mit Rest existieren q, r mit 0 ≤ r < ordm ( a ) und k = q ⋅ ordm ( a ) + r . Also gilt:
q
a r ≡ a r ( a ordm a ) ≡ a q ⋅ordm a +r ≡ a k ≡ 1 mod m .
(
)
(
)
Es folgt r = 0 wegen der Minimalität von ordm ( a ) und damit folgt ordm (a )|k .
Definition 3.27
Sei m ∈ N . Eine natürliche Zahl a heißt Primitivwurzel mod m , falls ordm ( a ) = ϕ ( m ) .
Beispiel 3.28
Sei p = 11 . Dann gilt
1
2
3
4
5
für die Potenzen mod p :
4
9
5
3
8
5
9
4
5
4
3
9
6
7
8
9
10
10 9 7 3 6 1
1
1
1
3
5
9
4
1
7
2
6
3
9
3
4
5
10 5 8 4 2 1
10 4 6 9 8 1
10 3 2 5 7 1
1
Also erhalten wir ord11 ( 1 ) = 1 , ord11 ( a ) = 5 für a = 3, 4, 5, 9 , ord11 ( a ) = 10 für a = 2, 6, 7, 8 und
ord11 ( 10 ) = 2 .
Satz 3.29 (3.14)
Sei p ∈ P mit p > 2 . Dann gibt es genau ϕ ( p − 1 ) = ϕ ( ϕ ( p ) ) Primitivwurzeln mod p .
Beweis
Jede der Zahlen 1,..., p − 1 besitzt eine Ordnung mod p , die nach Lemma 3.26 ein Teiler von ϕ ( p ) = p − 1
ist. Sei ψ ( d ) := # {a | 1 ≤ a ≤ p − 1, ordp (a ) = d } für alle d|p − 1 . Es gilt offenbar:
∑ ψ (d ) = p − 1 .
d |p −1
=
Behauptung:
≠ 0 , also
Sei ψ ( d ) ≠ 0 . Dann existiert ein a mit ordp (a ) = d . Die Zahlen a, a 2 ,..., a d sind allesamt Lösungen der
Kongruenz x d ≡ 1 mod p . Denn:
ψ (d )
ψ (d )
ϕ (d ) .
( a j )d = ( a d )j ≡ 1j ≡ 1 mod p für 1 ≤ j ≤ d .
Außerdem sind die a j für 1 ≤ j ≤ d paarweise inkongruent mod p , da sonst aus a j ≡ a i mod p für
1 ≤ i < j ≤ d stets a j −i ≡ 1 mod p gelten würde, woraus dann wegen j − i < d der Widerspruch zur Ordnung von a folgen würde. Nach dem Satz von Lagrange hat die Kongruenz x d ≡ 1 mod p höchstens d Lösungen, d.h. genau die Lösungen a, a 2 ,..., a d . Unter diesen befinden sich also alle Elemente der Ordnung d .
Zwischenbehauptung: Es gilt ordp ( a m ) = d genau dann, wenn ( m, d ) = 1 ist.
„ ⇒ “ Annahme: ( m, d ) > 1 . Dann gilt
d
m
m
≡ (a d )(m ,d ) ≡ 1(m ,d ) ≡ 1 mod p
und es folgt ein Widerspruch, da ( md,d ) < d ist.
′
„ ⇐ “ Annahme: 1 ≡ ( a m )d ≡ a md ′ mod p . Wegen ordp ( a ) = d gilt d|md ′ und aus Lemma 1.22 folgt d|d ′ .
m
Also ist ordp ( a ) = d . Damit gilt die Zwischenbehauptung und somit die Behauptung.
Nach Korollar 2.15 (ii) gilt:
∑ ϕ (d ) = p − 1 = ∑ ψ (d ) .
( a m )( m ,d )
d |p −1
d |p −1
Damit gilt:
0=
∑ ( ϕ (d ) − ψ (d ) ) = ∑
d |p −1
d |p −1
ψ(d )= 0
25
ϕ (d ) ,
Kapitel 3
Kongruenzen
da
falls ψ (d ) ≥ 1,
⎧
⎪ 0,
ϕ ( d ) − ψ ( d ) = ⎪⎨ ( )
ϕ d , falls ψ ( d ) = 0.
⎪
⎪
⎩
Die Summe rechts muss leer sein, d.h. für alle d|p − 1 gilt ψ ( d ) ≠ 0 .
Im Beispiel 3.28 oben ist 7 eine Primitivwurzel mod11 . Aus dem Satz folgt, dass alle 7m Primitivwurzeln
sind, für die ( m,10 ) = 1 gilt. Die einfach unterstrichenen Zahlen sind Primitivwurzeln.
Bemerkung 3.30
(i) Für p ∈ P und alle d|p − 1 gilt:
ψ ( d ) := # {a | 1 ≤ a ≤ p − 1, ordp (a ) = d } = ϕ ( d ) .
(ii) Ist a Primitivwurzel mod p , so ist
{a m | 1 ≤ m ≤ p − 1, ( m, p − 1 ) = 1}
die Menge aller Primitivwurzeln mod p .
Satz 3.31 (3.15)
Seien p ∈ P mit p > 2 und k ∈ N . Dann existiert eine Primitivwurzel mod pk , d.h. ein Element der Ordnung ϕ ( pk ) = pk − pk −1 .
Beweis
Sei g eine Primitivwurzel mod p .
Behauptung: Es existiert ein gx := g + px für ein x ∈ N , das eine Primitivwurzel mod pk für alle k ist. Es
gilt g p −1 ≡ 1 mod p , d.h. g p −1 = 1 + py für ein passendes y ∈ Z . Binomischer Lehrsatz impliziert nun:
p − 1 p −3
gxp −1 = ( g + px )p −1 = g p −1 + ( p − 1 ) g p −2 ( px ) +
g
( px )2 + ... + ( px )p −1
2
≡ 1 + ( y + ( p − 1 ) g p −2x ) p mod p 2 ,
d.h. gxp −1 = 1 + pz , wobei z ≡ y + ( p − 1 ) g p −2x mod p ist. Wegen p ? ( p − 1 ) g p −2 durchläuft ( p − 1 ) g p −2x
mit x ein volles Restsystem mod p . Insbesondere können wir x so wählen, dass ( p − 1 ) g p −2x ≡ − y mod p
ist. Dann folgt p ? z . Also ist gx Primitivwurzel mod pk für alle k . Sei nun d := ordpk ( gx ) . Nach Lemma
3.26 gilt: d|ϕ ( pk ) = pk −1 ( p − 1 ) . Wegen gx ≡ g mod p ist gx nach Voraussetzung Primitivwurzel mod p .
Also gilt ordp ( gx ) = p − 1 , also p − 1|d und damit wegen d|pk −1 ( p − 1 ) erhalten wir d = ( p − 1 ) p j für ein
j mit 1 ≤ j ≤ k − 1 . Noch ein Mal liefert der Binomische Lehrsatz:
(
pj
( 1 + pz )
= 1+ p
pj
z +∑
j +1
t =2
( )
pj
t
( pz )t
= 1+ p
)
pj
p j ( p j − 1 ) ⋅ ... ⋅ ( p j − t + 1 )
j +1
zj
( pz )t = 1 + p
t!
t =2
z +∑
j +1
mit einem passenden z j , wobei p ? z j wegen p ? z . Nach Definition von d ist gxd ≡ 1mod pk . D.h. es gilt:
j
j
1 ≡ gxd = gx( p −1 )p = ( 1 + pz )p = 1 + p j +1z j mod pk .
Dann folgt p k |p j +1z j , also j ≥ k − 1 . Da andererseits j ≤ k − 1 ist, folgt j = k − 1 .
Korollar 3.32 (3.16)
Sei m ∈ N . Es existieren Primitivwurzeln mod m genau dann, wenn m = 1, 2, 4 oder m = p k oder
m = 2pk für p ∈ P mit p > 2 und k ∈ N gilt.
Beweis
„ ⇐ “ Zu m = 1,2 ist 1 Primitivwurzel, zu m = 4 ist 3 Primitivwurzel. Zu m = p k existieren Primitivwurzeln nach dem vorigen Satz.
Sei g Primitivwurzel mod pk . Sei g ′ das ungerade Element der Menge { g, g + pk } . Dann ist g ′ Primitivwurzel mod 2pk . Denn:
ϕ ( 2p k ) = ϕ ( 2 ) ϕ ( p k ) = ϕ ( p k ) .
d
d
k
Aus ( g ′ ) ≡ 1mod 2p folgt also ( g ′ ) ≡ 1mod p k und damit ϕ ( pk )|d .
Bleibt zu zeigen, dass für m ≠ 1,2, 4, pk , 2pk keine Primitivwurzeln mod m existieren.
„ ⇒ “ Da m ∉ {1, 2, 4, pk , 2pk | p ∈ P, p > 2, k ∈ N } ist m = 2k für k ≥ 3 oder m = m1m2 mit ( m1, m2 ) = 1
und m1, m2 ≥ 3 (mit etwas Überlegung).
1. Fall: m = 2k für k ≥ 3 .
k −2
Behauptung: a 2 ≡ 1mod 2k für alle ungeraden a und k ≥ 3 . Wir führen Induktion nach k . Sei also
k = 3 . Dann gilt a 2 ≡ 1mod 8 . Sei nun k
k + 1 . Dann gilt:
26
Kapitel 3
Kongruenzen
k +1−2
a2
k −2
da a 2
k −1
− 1 = a2
k −2
− 1 ≡ 0 mod 2k ist und a 2
− 1 = (a 2
k −2
− 1)(a 2
k −2
+ 1 ) ≡ 0 mod 2k +1 ,
+ 1 ≡ 0 mod 2 nach Induktionsvoraussetzung. Aus
1
( k)
k −2
a 2 ϕ 2 = a 2 ≡ 1mod 2k
folgt nun, dass es keine Primitivwurzeln mod 2k gibt.
2. Fall: m = m1m2 mit ( m1, m2 ) = 1 , m1, m2 ≥ 3 . Dann enthalten beide mi einen ungeraden Primfaktor pi
oder einen Faktor 2ki mit ki ≥ 2 . Wegen ϕ ( pili ) = pili − pili −1 bzw. ϕ ( 2ki ) = 2ki −1 folgt 2|ϕ ( pili ) bzw.
2|ϕ ( 2ki ) . Also gilt für jedes a mit ( a, m ) = 1 :
1
a 2ϕ
(
m)
1
= ( a ϕ( m1 ) )2 ϕ( m2 ) ≡ 12 ϕ( m2 ) ≡ 1mod m1 ,
1
da a ϕ( m1 ) ≡ 1mod m1 nach dem kleinen Satz von Fermat (3.18) gilt. Analog gilt:
1
a 2ϕ
1
Also erhalten wir a 2 ϕ
(
m)
(
m)
1
= ( a ϕ( m2 ) )2 ϕ( m1 ) ≡ 12 ϕ( m1 ) ≡ 1mod m2 .
1
≡ 1mod m .
27
28
Kapitel 4
Quadratische Reste
Bisher haben wir nur lineare Kongruenzen gelöst, d.h.
ax ≡ b mod m lösbar ⇐⇒ ( a, m )|b .
Jetzt untersuchen wir, ob ax 2 + bx + c ≡ 0 mod m lösbar ist. Mit y := 2ax + b , d := b 2 − 4ac wird daraus
y 2 ≡ d mod 4am (quadratische Ergänzung).
Es genügt also rein quadratische Kongruenzen zu betrachten.
Definition 4.1
Sei m ∈ N und a ∈ Z mit ( a, m ) = 1 . Dann heißt a quadratischer Rest mod m , falls x 2 ≡ a mod m lösbar
ist, andernfalls heißt a quadratischer Nichtrest mod m . Für Primzahlmoduln m = p ∈ P und beliebiges
a ∈ Z heißt
⎧
0,
falls p|a,
⎪
⎪
a
⎪
:= ⎪
1,
falls a quadratischer Rest mod p,
⎨
p
⎪
⎪
falls a quadratischer Nichtrest mod p
−1,
⎪
⎪
⎩
das Legendre-Symbol (gelesen „ a über p “).
( )
Beispiel 4.2
Sei p = 11 . Für x 2 ≡ a mod11 gilt:
.a .
a
11
( )
0
1
2
3
4
5
6
7
8
9
10
0
1
−1
1
1
1
−1
−1
−1
1
−1
a
⎛a′ ⎞
Lemma 4.3 (4.1)
Sei p ∈ P . Dann gilt:
(i) Aus a ≡ a ′ mod p folgt
(ii) Es gibt genau
( p ) = ⎜⎜⎝ p ⎠⎟⎟⎟ .
p −1
p −1
quadratische Reste und
quadratische Nichtreste mod p für p > 2 .
2
2
Beweis
(i) Klar.
1
2 2
2
(ii) Wir setzen r = p −
2 . Offenbar sind 1 , 2 ,..., r quadratische Reste mod p und paarweise inkongruent
mod p , denn aus 1 ≤ i < j ≤ r mit i 2 ≡ j 2 mod p folgt j 2 − i 2 ≡ ( j − i )( j + i ) ≡ 0 mod p , d.h.
p|( j − i )( j + i ) und nach Lemma 1.22 gilt p|j − i oder p|j + i . Da 1 ≤ j − i < r und 1 ≤ j + i < p ,
folgt der Widerspruch. Außerdem sind ( r + 1 )2 ,..., ( r + r )2 nochmal die selben Reste r 2 ,...,12 , denn:
j 2 ≡ ( p − j )2 ≡ p 2 − 2pj + j 2 ≡ j 2 mod p für 1 ≤ j ≤ r .
Insgesamt haben wir also die 0 , genau
p −1
2
quadratische Reste und
Satz 4.4 (4.2) (Eulersches Kriterium)
Sei p ∈ P mit p > 2 , a ∈ Z . Dann gilt:
Beweis
1
Sei r := p −
2 .
1. Fall: p|a . Trivial.
a
(p) ≡ a
p −1
2
mod p .
p −1
2
quadratische Nichtreste.
Kapitel 4
Quadratische Reste
2. Fall: Sei a quadratischer Rest. Also existiert ein x ∈ N mit x 2 ≡ a mod p . Nach dem kleinen Satz von
Fermat gilt:
a
r
= 1 ≡ x p −1 ≡ ( x 2 ) ≡ a r mod p .
p
3. Fall: Sei a quadratischer Nichtrest. Nach Satz von Lagrange (3.21) hat die Kongruenz x r ≡ 1 mod p
höchstens r Lösungen mod p . Nach Lemma 4.3 (ii) sind dies genau die r quadratischen Reste mod p , denn
es gilt y 2r ≡ y p −1 ≡ 1mod p nach dem kleinen Satz von Fermat (3.18). D.h. dies sind alle Lösungen von
x r ≡ 1 mod p . Folglich ist für alle quadratischen Nichtreste a
a r ≡ 1 mod p .
( )
Aus dem kleinen Satz von Fermat (3.18) folgt:
a 2r − 1 = ( a r − 1 ) ( a r + 1 ) ≡ 0 mod p .
Hier sehen wir aber: p ? a r − 1 , also muss p|a r + 1 gelten, d.h.
a
mod p .
a r ≡ −1 ≡
p
( )
Korollar 4.5 (4.3)
Für p ∈ P und a, b ∈ Z gilt
( abp ) = ( ap )( bp ) .
Beweis
Man rechne die Gültigkeit für p = 2 nach. Sei also p > 2 . Aus Satz 4.4 folgt
p −1 p −1
p −1
ab
a b
≡ ( ab ) 2 = a 2 b 2 ≡
mod p .
p
p p
Also gilt:
ab
a b
,
p|
−
p
p p
also
ab
a b
−
= 0.
p
p p
( )
( )( )
( ) ( )( )
( ) ( )( )
Satz 4.6 (4.4) (1. Ergänzungsgesetz)
Für p ∈ P mit p > 2 gilt:
Beweis
Nach dem eulerschen Kriterium gilt:
( −p1 ) =
( −p1 ) ≡
Also folgt:
p −1
2
( −1 )
p −1
2
( −1 )
( −p1 ) =
.
mod p .
p −1
2
( −1 )
.
Bemerkung 4.7
Das 1. Ergänzungsgesetz besagt: −1 ist genau dann quadratischer Rest mod p , wenn p ≡ 1mod 4 . Das ist
1
aber uns schon aus Korollar 3.20 bekannt. Für p ≡ 1mod 4 ist x 2 ≡ −1 mod p lösbar mit x = ± ( p −
2 )! .
Definition 4.8
Für einen Modul m ∈ N und ein a ∈ Z definieren wir den betragskleinsten Rest a ′ mod m als diejenige
Zahl a ′ , für die gilt a ′ ≡ a mod m und − m2 < a ′ ≤ m2 .
Lemma 4.9 (4.5) (Lemma von Gauß)
Sei p ∈ P mit p > 2 , a ∈ Z , p ? a . Es bezeichne
1
mod p für 1 ≤ k ≤ p −
mit ( ak )′ < 0 . Dann gilt:
2
29
p
(
a ) die Anzahl der betragskleinsten Reste (ak )′
Kapitel 4
Quadratische Reste
a
(p) =
p (a )
( −1 )
.
Beweis
1
(
)′
Seien r := p −
2 und ak := ak .
Behauptung:
{ ak | 1 ≤ k ≤ r } = { 1,..., r } .
Denn wegen p ? a ist für alle k offenbar 1 ≤ ak ≤ r . Es genügt also zu zeigen: ak ≠ a j für k ≠ j .
1. Fall: ak = a j . Dann ist ( ak )′ = ( aj )′ , d.h. ak ≡ aj mod p und damit p|a ( k − j ) . Aus Lemma 1.22 folgt
nun: k ≡ j mod p . Aus 1 ≤ k, j ≤ r folgt jetzt k = j .
2. Fall: ak = −a j . Dann ist (ak )′ = − ( aj )′ , d.h. ak ≡ −aj mod p und damit p|a ( k + j ) . Aus Lemma 1.22
folgt wieder: k + j ≡ 0 mod p . Aber aus 2 ≤ k + j ≤ r + r < p folgt nun der Widerspruch.
Es folgt:
r
r
⎛ r
⎞
∏ ak = a r r ! ≡ ∏ (ak )′ = ⎜⎜⎜⎝ ∏ | (ak )′ |⎠⎟⎟⎟⎟( −1 ) p a = r ! ( −1 ) p a mod p .
(
k =1
j =1
)
(
)
j =1
Aus ( r !, p ) = 1 folgt:
( −1 )
p (a )
≡ ar ≡
a
( p ) mod p
nach dem eulerschen Kriterium (4.4).
Beispiel 4.10
Sei p = 17 , a = 3 . Dann haben wir:
.k .
1
2
3
4
5
6
7
8
. ak .
3
6
9
12
15
18
21
24
. ( ak )′ .
.3 .
.6 .
. −8 .
. −5 .
. −2 .
.1 .
.4 .
.7 .
Also ist
17
(3)
= 3 und x 2 ≡ 3 mod17 unlösbar, denn es gilt:
3
= ( −1 )3 = −1 .
17
( )
Korollar 4.11 (4.6) (2. Ergänzungsgesetz)
Für p ∈ P mit p > 2 gilt
( p2 ) =
( −1 )
p2 −1
8
.
Beweis
Mit der Terminologie der betragskleinsten Reste gilt:
p
p
p −1
2k = 2, 4, 6,..., 2 ⎡⎢ ⎤⎥ , 2 ⎡⎢ ⎤⎥ + 1 ,..., p − 3, p − 1 für k = 1,...,
.
⎣4⎦
⎣4⎦
2
(
>0
)
<0
1
p
( )
Es folgt, dass es genau [ p4 ] positive und genau p −
2 − [ 4 ] = p 2 negative betragskleinste Reste mod p gibt.
Aus dem Lemma von Gauß (4.9) folgt:
p −1
p
2
( )
= ( −1 ) p 2 = ( −1 ) 2 −[ 4 ] .
p
Zu zeigen ist also:
p −1 ⎡p⎤
p2 − 1
mod 2 .
−⎢ ⎥ ≡
⎣4⎦
2
8
Wir unterscheiden: p = 4m + r mit r ∈ { 1, 3 } . Dann gilt:
p −1 ⎡p⎤
r −1
r −1
,
− ⎢ ⎥ = 2m +
−m = m +
⎣4⎦
2
2
2
( )
= 0,1
2
2
p −1 1(
r −1
r2 − 1
r −1
mod 2 .
= 16m 2 + 8mr + r 2 − 1 ) = 2m 2 + mr +
≡m+
≡m+
8
8
8
8
2
= 0,1
Bemerkung 4.12
Es gilt:
30
Kapitel 4
Quadratische Reste
für p ≡ ±1 mod 8,
für p ≡ ±3 mod 8,
⎧1
( p2 ) = ⎪⎪⎨⎪⎪⎩−1
denn sei p ≡ ±1mod 8 , d.h. p = 8t ± 1 . Nun folgt: p 2 − 1 = 64t 2 ± 16t + 1 − 1 = 16 ( 4t 2 ± t ) . Also gilt:
p2 − 1
= 2 ( 4t 2 ± t ) ≡ 0 mod 2 .
8
Im anderen Fall ergibt sich wegen p = 8t ± 3 p 2 − 1 = 64t 2 ± 48t + 9 − 1 = 8 ( 8t 2 ± 6t + 1 ) . Also gilt:
p2 − 1
= 8t 2 ± 6t + 1 ≡ 1mod 2 .
8
Beispiel 4.13 (Anwendung der Gesetze für Legendre-Symbole)
Es ist x 2 ≡ −8 mod13 genau dann lösbar, wenn
Es gilt:
( −138 ) = 1 .
( −138 ) = ( −131 )( 132 )( 132 )( 132 ) =
13−1
2
( −1 )
=1
=1
( 132 ) = −1 ,
da 13 ≡ −3 mod 8 .
Das folgende Ergebnis wurde von Euler, Legendre und Gauß unabhängig voneinander gefunden, erstmals
aber von Gauß bewiesen.
Satz 4.14 (4.7) (Quadratisches Reziprozitätsgesetz)
Seien p, q ∈ P mit p, q > 2 und p ≠ q . Dann gilt:
⎧⎪ −1,
p −1 q −1
p q
= ( −1 ) 2 2 = ⎪⎨
⎪⎪⎩1,
q p
( )( )
Beweis
Nach dem Lemma von Gauß gilt:
p
(q ) =
( −1 )
falls p ≡ q ≡ 3 mod 4,
sonst.
q(p)
mit der Anzahl q ( p ) der betragskleinsten Reste ( px )′ mod q für 1 ≤ x ≤ q −2 1 und − q2 < ( px )′ < 0 . Wegen ( px )′ ≡ px mod q , d.h. ( px )′ = px − qy für passendes eindeutiges y ∈ Z , gilt
q −1
q
2
q ( p ) = # {( x , y ) ∈ Z | 1 ≤ x ≤ 2 , − 2 < px − qy < 0 } .
p
p +1
p −1
1
1
Dabei gilt offenbar 1 ≤ y < px
q + 2 < 2 + 2 = 2 , also 1 ≤ y ≤ 2 . Also folgt:
q
p
q
2
q ( p ) = # {( x , y ) ∈ Z | 0 < x < 2 , 0 < y < 2 , − 2 < px − qy < 0 } .
Analog gilt:
p (q )
= # {( x , y ) ∈ Z2 | 0 < x < p2 , 0 < y < q2 , − p2 < qx − py < 0 }
= # {( y, x ) ∈ Z2 | 0 < y < p2 , 0 < x < q2 , − p2 < qy − px < 0 }
= # {( x , y ) ∈ Z2 | 0 < x < q2 , 0 < y < p2 , − p2 < qy − px < 0 } .
Wir haben zu zeigen:
q (p) +
p (q )
≡
p − 1q − 1
mod 2 ,
2
2
also, dass
L ( p, q ) :=
p − 1q − 1
−
2
2
q (p)−
p (q )
gerade ist.
1 q −1
q
p
2
Offenbar ist p −
2
2 die Anzahl der Gitterpunkte in Z im Rechteck 0 < x < 2 , 0 < y < 2 . Also ist L ( p, q )
q
genau die Anzahl der Gitterpunkte dieses Rechtecks mit px − qy ≤ − 2 oder qy − px ≤ − p2 (d.h.
L ( p, q ) = # ( A ∪ B ) , wobei A ∪ B die Menge der roten Punkte in der Zeichnung unten ist). Setze
Δ1 := {( x , y ) ∈ R2 | 0 < x < q2 , 0 < y < p2 , px − qy ≤ − q2 } ,
Δ2 := {( x , y ) ∈ R2 | 0 < x < q2 , 0 < y < p2 , qy − px ≤ − p2 } .
31
Kapitel 4
Quadratische Reste
Offenbar gilt Δ1 ∩ Δ2 = ∅ und wir haben eine Bijektion zwischen den Gitterpunkten von Δ1 und denen
von Δ2 , nämlich
( q +2 1 − x, p +2 1 − y ) .
( x, y )
Also ist L ( p, q ) gerade und damit folgt die Behauptung.
Beispiel 4.15
Ist x 2 ≡ −15 mod 71 lösbar? Es gilt:
−15
−1
=
71
71
(
) ( )( 713 )( 715 ) = −1 ( 713 ) −1 ( 715 ) −1
2
1
= −1 ( ) −1 ( ) ⋅ 1 = − 1 − 1
−1 ⋅ 1 ⋅ 1 = −1,
3
5
(
da
)35
(
)
(
( 713 )( 713 ) =
)
(
( −1 )1⋅35
⇐⇒
)1⋅35
(
)(
32 −1
8
)
(
)2⋅35
)
(
( 713 ) = ( 713 ) −1
(
)1⋅35
und analog für 5 über 71 gilt. Also ist die Kongruenz unlösbar.
Zur Behandlung quadratischer Kongruenzen mit beliebigen Moduln verallgemeinern wir das Legendre-Symbol.
Definition 4.16
Sei n ∈ N , 2 ? n , also n = p1 ⋅ ... ⋅ pk , k ∈ N 0 , mit ungeraden (nicht notwendig verschiedenen) Primzahlen
p j . Ist a ∈ Z , so heißt
1,
für n = 1,
⎧
⎪
⎪
a
für (a, n ) > 1,
=⎪
⎪ 0,
⎨
n
⎪
a
a
⎪
⋅ ... ⋅
,
für n > 1 und ( a, n ) = 1
⎪
⎪
pk
⎪
⎩ p1
das Jacobi-Symbol.
( )
( )
( )
Satz 4.17 (4.8)
Seien n ∈ N , 2 ? n und ( a, n ) = 1 . Dann ist a quadratischer Rest mod n genau dann, wenn a quadratischer Rest mod p für alle p ∈ P mit p|n ist. Insbesondere folgt aus
a
= −1 ,
n
dass a quadratischer Nichtrest mod n ist. Aus
a
=1
n
folgt aber im Allgemeinen nicht, dass a quadratischer Rest mod n ist.
Beweis
„ ⇒ “ Sei n = p1e1 ⋅ ... ⋅ pkek . Ist a quadratischer Rest mod n , so existiert ein x ∈ Z mit x 2 ≡ a mod n . Also
ist für alle 1 ≤ j ≤ k x 2 ≡ a mod p j , d.h. a ist quadratischer Rest mod p j für 1 ≤ j ≤ k .
„ ⇐ “ Nach Voraussetzung existiert ein x i ∈ Z für 1 ≤ i ≤ k mit x i2 ≡ a mod pi .
Behauptung: Ist x 2 ≡ a mod p lösbar, dann ist auch x 2 ≡ a mod pr lösbar für jedes r ∈ N . Wir führen Inr + 1 . Sei also x 02 ≡ a mod pr .
duktion nach r . Die Voraussetzung für r = 1 ist offensichtlich erfüllt. Sei r
Wegen ( a, n ) = 1 und 2 ? n gilt ( 2x 0 , p ) = 1 . Also besitzt die Kongruenz
( )
( )
32
Kapitel 4
Quadratische Reste
x 02 − a
mod p
pr
eine ganze Zahl nach Induktionsvoraussetzung ist. Diese Kongruenz ist genau
2x 0y ≡ −
eine Lösung y , wobei
dann lösbar, wenn
x 02 −a
pr
1 = ( 2x 0 , p ) |
x 02 − a
.
pr
Also gilt:
x 02 − a
mod p
pr
und damit 2x 0ypr ≡ − ( x 02 − a ) mod pr +1 . Wir definieren x := x 0 + pr y und erhalten
2x 0y ≡ −
x 2 = ( x 0 + pr y )2 = x 02 + 2x 0 p r y + p 2r y 2 ≡ x 02 + 2x 0 pr y ≡ x 02 − ( x 02 − a ) ≡ a mod pr +1
und damit folgt die Behauptung.
Demnach existiert ein yi ∈ Z für 1 ≤ i ≤ k mit yi2 ≡ a mod piei . Mit dem Chinesischen Restsatz (3.11) konstruieren wir daraus ein y ≡ yi mod piei für 1 ≤ i ≤ k . Dieses erfüllt
y 2 ≡ yi2 ≡ a mod piei
und somit auch y 2 ≡ a mod p1e1 ⋅ ... ⋅ pkek = n .
Sei nun
Also gibt es p ∈ P mit p|n , so dass
( na ) = −1 .
a
( p ) = −1
d.h. a ist quadratischer Nichtrest mod p und damit auch mod n . Gilt
a
= 1,
n
so kann für n = p1p2
a
a
=
= −1
p1
p2
gelten, also ist a quadratischer Nichtrest mod n .
( )
( ) ( )
Beispiel 4.18
Es gilt:
( 23 ) = ( 25 ) = −1 ,
aber
( 152 ) = ( 23 )( 25 ) = 1 .
Vollkommen analog zu den bewiesenen Aussagen für Legendre-Symbole lässt sich folgender Satz zeigen.
Satz 4.19 (4.9)
Seien m, n ∈ N mit 2 ? mn und a, a ′, b ∈ Z . Dann gilt:
(i) Aus a ≡ a ′ mod m folgt
a
a′
=
.
m
m
(ii) Es gilt:
ab
a b
a
a a
,
.
=
=
m
m m
mn
m n
(iii) Es gilt:
n −1
−1
= ( −1 ) 2 (1. Ergänzungsgesetz),
n
n 2 −1
2
= ( −1 ) 8 (2. Ergänzungsgesetz).
n
(iv) Ist ( m, n ) = 1 , so gilt
m −1 n −1
m n
= ( −1 ) 2 2 (Reziprozitätsgesetz).
n m
( ) ( )
( ) ( )( ) ( ) ( )( )
( )
( )
( )( )
33
Kapitel 4
Quadratische Reste
Beispiel 4.20
Es ist 2999 ∈ P . Also ist
335
( 2999
)
ein Legendre-Symbol. Weiter gilt:
335
5
67
( 2999
) = ( 2999
) = ...
)( 2999
Aus dem Reziprozitätsgesetz für Jacobi-Symbole folgt:
335
2999 ( )167⋅1499
−16 ( )
−1
=
−1
=
−1 =
2999
335
335
335
Also ist x 2 ≡ 335 mod 2999 lösbar, da
335
2999
ein Legendre-Symbol ist.
(
) (
)
(
)
16
( )( 335
) −1
(
)
34
(
)
= ( −1 )167
4
( 335
)
2
( −1 )
= 1.
35
Kapitel 5
Quadratische Formen
Wir untersuchen binäre quadratische Formen über Z , d.h.
f ( x , y ) = ax 2 + bxy + cy 2 ∈ Z [ x , y ] .
Es stellt sich die Frage, welche Werte f ( x , y ) bei x , y ∈ Z annimmt. Wir haben
{ x 2 + y 2 | x , y ∈ Z } = { 0,1, 2, 4, 5, 8, 9,10,13,...} .
Definition 5.1
Zu f ( x , y ) heißt d f := b 2 − 4ac die Diskriminante von f . Offenbar ist
⎧⎪ 0 mod 4,
für 2|b,
d f ≡ ⎪⎨
für 2 ? b.
⎪⎪⎩1 mod 4,
Lemma 5.2 (5.1)
Ist d f < 0 , so nimmt f ( x , y ) nur positive Werte für a > 0 bzw. nur negative Werte für a < 0 an. Dann
heißt f positiv bzw. negativ definit. Für d f > 0 nimmt f ( x , y ) positive und negative Werte an und heißt
indefinit.
Beweis
Es gilt:
4af ( x , y ) = 4a 2x 2 + 4abxy + 4acy 2 = ( 2ax + by )2 − d f y 2 ,
woraus die Behauptung folgt.
Für das Folgende eignet sich die Matrixschreibweise quadratischer Formen:
⎛ a b2 ⎞⎟
x
⎟⎟ , x = ⎛⎜ ⎞⎟⎟ , x T = ( x , y ) .
f
F = ⎜⎜⎜ b
⎜
⎝ y ⎠⎟
⎜⎝ 2 c ⎠⎟
Wir erhalten:
x T Fx = f ( x , y ) .
Definition 5.3
Eine 2 × 2 -Matrix T über Z heißt unimodular, falls detT = 1 . Die zugehörige Variablensubstitution
x
Tx
heißt auch unimodular.
Definition 5.4
Zwei quadratische Formen f und g heißen äquivalent, falls die zugehörigen Matrizen F und G durch eine
unimodulare Transformation auseinander hervorgehen, d.h.
x TGx = (T x )T F (T x ) = ( x TT T ) F (T x ) = x T (T TFT ) x ,
also wenn gilt:
G = T T FT .
Offensichtlich ist diese Relation eine Äquivalenzrelation auf der Menge der binären quadratischen Formen
über Z .
Kapitel 5
Quadratische Formen
Lemma 5.5 (5.2)
Seien f und g äquivalente quadratische Formen. Dann gilt:
(i) f und g haben dieselbe Wertemenge für x , y ∈ Z .
(ii) f und g haben dieselbe Wertemenge für x , y ∈ Z mit ( x , y ) = 1 .
(iii) d f = dg .
Beweis
Sei T unimodulare Transformation zwischen f und g . Wegen detT = 1 ist T invertierbar und
1
= 1 = detT ⋅ detT −1
detT −1 =
detT
folgt aus
⎛p q ⎞
T = ⎜⎜ r s ⎟⎟⎟ ,
⎝
⎠
dass gilt:
1 ⎛ s −q ⎞⎟ ⎛ s −q ⎞⎟
⎜
,
=⎜
detT ⎝⎜ −r p ⎠⎟⎟ ⎝⎜ −r p ⎠⎟⎟
also ist auch T −1 ganzzahlig und unimodular.
(i) Es genügt daher zu zeigen: Ist g ( x , y ) = n ∈ Z , so existieren x ′, y ′ ∈ Z mit f ( x ′, y ′ ) = n . Sei also
n = g ( x , y ) = x TGx . Es folgt:
f (T x ) = (T x )T F (T x ) = x T (T T FT ) x = x TGx = n ,
T −1 =
d.h.
⎛ ′⎞
⎜⎜ x ⎟⎟ := T ⎜⎛ x ⎞⎟ .
⎟
⎝⎜ y ⎠⎟⎟
⎜⎝⎜ y ′ ⎠⎟
(ii) Sei ( x , y ) = 1 und g ( x , y ) = n . Wir wissen bereits nach (i), dass f ( x ′, y ′ ) = n . Also genügt zu zeigen:
( x ′, y ′ ) = 1 . Es gilt:
⎛ ′⎞
⎜⎜ x ⎟⎟ = T ⎛⎜ x ⎞⎟ = ⎛⎜ px + qy ⎞⎟ mit T = ⎛⎜ p q ⎞⎟ .
⎜⎜ y ′ ⎟⎟
⎜⎝ r s ⎠⎟⎟
⎜⎝ y ⎠⎟⎟ ⎜⎜⎝ rx + sy ⎠⎟⎟
⎝ ⎠
Behauptung: ( px + qy, rx + sy ) = 1 .
Sei ( px + qy, rx + sy ) = m . Dann gibt es ein m1 und ein m2 mit
prx + qry = mrm1 und prx + psy = mpm2 .
Es folgt:
y = ( ps − qr ) y = m ( pm2 − rm1 ) .
= detT =1
Also gilt: m|y . Analog folgt: m|x . Wegen ( x , y ) = 1 folgt nun m = 1 .
(iii) Es gilt:
dg = −4 detG = −4 detT T detG detT = −4 det (T TGT ) = −4 det F = d f .
Ab jetzt betrachten wir nur noch positiv definite quadratische Formen, d.h. wir setzen d f < 0 und a > 0
voraus. Also ist auch c > 0 , denn es soll d f = b 2 − 4ac < 0 gelten.
Satz 5.6 (5.3) (Reduktionssatz)
Jede positiv definite quadratische Form f ist äquivalent zu einer Form
f0 ( x , y ) = a 0x 2 + b0xy + c0y 2 mit −a 0 < b0 ≤ a 0 < c0 oder 0 ≤ b0 ≤ a 0 = c0 .
Beweis
Wir wenden nur die drei unimodularen Transformationen an mit den Matrizen
⎛ 0 1 ⎞⎟
⎛ 1 ±1 ⎞⎟
⎟ und T± := ⎜⎜
⎟.
T0 := ⎜⎜
⎜−
⎜⎝ 1 0 ⎠⎟⎟
⎜⎝ 0 1 ⎠⎟⎟
Ist f ( x , y ) = ax 2 + bxy + cy 2 , so erhalten wir unter Anwendung dieser Transformationen
⎛ ⎛ a b2 ⎞⎟ ⎞⎟
⎛ c − b2 ⎞⎟
2
2
⎟⎟T0 ⎟⎟ x = x T ⎜⎜
⎟
f (T0 x ) = x T ⎜⎜⎜T0 T ⎜⎜⎜ b
⎜⎜ − b a ⎟⎟ x = cx − bxy + ay ,
⎜⎝ ⎜⎝ 2 c ⎠⎟ ⎠⎟
⎝ 2
⎠
d.h. T0 vertauscht a und c bei unverändertem b . Außerdem ist
f (T+ x ) = ax 2 + (b + 2a ) xy + ( a + b + c ) y 2 und f (T− x ) = ax 2 + (b − 2a ) xy + ( a − b + c ) y 2 ,
36
Kapitel 5
Quadratische Formen
d.h. wir können mittels T± b durch b ± 2a ersetzen, d.h. wir können b mod 2a abändern, wobei a unverändert bleibt. Durch Mehrfachanwendung von T+ bzw. T− können wir bei festem a den mittleren Koeffizienten b zu b ′ abändern, wobei b ′ ≤ a (wegen der Rechnung mod 2a ). Zusammen mit T0 können wir f
derart zu einer äquivalenten Form f ′ ( x , y ) = a ′x 2 + b ′xy + c ′y 2 umformen, dass b ′ ≤ a ′ ≤ c ′ gilt (dies geschieht nach endlich vielen Schritten). Erhalten wir auf diese Weise −a ′ < b ′ ≤ a ′ < c ′ , so ist f reduziert. Es
bleiben also die beiden Fälle:
1. Fall: −a ′ = b ′ . Wir wenden erneut T+ an und erhalten aus f ′ ( x , y ) = a ′x 2 − a ′xy + c ′y 2 :
f ′′ ( x , y ) = a ′′x 2 + b ′′xy + c ′′y 2 mit a ′′ = a ′ , b ′′ = a ′ und c ′′ = a ′ + b ′ + c ′ = a ′ − a ′ + c ′ = c ′ ,
also −a ′′ < b ′′ ≤ a ′′ ≤ c ′′ .
2. Fall: a ′ = c ′ . Wir wenden erneut T0 an und erhalten aus f ′ ( x , y ) = a ′x 2 + b ′xy + a ′y 2 :
f ′′ ( x , y ) = a ′′x 2 + b ′′xy + c ′′y 2 mit a ′′ = c ′ = a ′ , b ′′ = −b ′ und c ′′ = a ′ .
Falls b ′ ≥ 0 ist, sind wir fertig, falls b ′ < 0 , ist b ′′ ≥ 0 .
Definition 5.7
Eine Form, deren Koeffizienten eine der beiden Relationsbedingungen aus Satz 5.6 erfüllen, heißt reduziert.
Beispiel 5.8
f ( x , y ) = 2x 2 + 9xy + 11y 2 ist nicht reduziert, da b > a und wird durch T− übergeführt in
2x 2 + 5xy + 4y 2
und durch nochmaliges Anwenden von T− erhalten wir 2x 2 + xy + y 2 . Anwendung von T0 liefert
x 2 − xy + 2y 2
und T+ führt zu der reduzierten quadratischen Form
f0 ( x , y ) := x 2 + xy + 2y 2 ( −a 0 < b0 ≤ a 0 < c0 ).
Es gilt: d f = 92 − 4 ⋅ 2 ⋅ 11 = 81 − 88 = −7 und d f0 = 1 − 4 ⋅ 2 ⋅ 1 = −7 , also d f = d f0 .
Lemma 5.9 (5.4)
Sei d < 0 . Ist f ( x , y ) = ax 2 + bxy + cy 2 positiv definit und reduziert mit d f = d , so gilt
d
d
d
d
d
− ≤ ac ≤ − und damit 0 < a ≤ − , b ≤ − , 0 < c ≤ − .
4
3
3
3
3
Insbesondere gibt es nur endlich viele positiv definite reduzierte quadratische Formen mit Diskriminante d
und eine unendliche Menge von positiv definiten Formen mit einer festen Diskriminante d .
Beweis
Nach Definition ist −d f = −d = 4ac − b 2 . Da f reduziert ist, gilt b 2 ≤ a 2 ≤ ac . Es folgt:
3ac ≤ 4ac − b 2 = −d ≤ 4ac .
Insbesondere gilt
0 ≤ b ≤a ≤c ≤−
woraus die Behauptung folgt.
d
d
≤− ,
3a
3
Definition 5.10
Sei d < 0 . Die nach Lemma 5.9 endliche Anzahl von positiv definiten reduzierten quadratischen Formen mit
Diskriminante d heißt Klassenzahl von d , geschrieben h ( d ) .
Beispiel 5.11
Wir berechnen h ( −4 ) . Nach Lemma 5.9 gilt:
d
d
4
− ≤ ac ≤ − ⇒ 1 ≤ ac ≤
⇒ a = c = 1,
4
3
3
da die Diskriminante positiv definit ist und damit a > 0 gilt. Wegen 0 ≤ b ≤ a = 1 bleibt nur b = 0 , denn
für b = 1 ist b 2 − 4ac = −3 ≠ −4 . Also ist h ( −4 ) = 1 ( x 2 + y 2 ist die einzige reduzierte Form mit der
Diskriminante −4 ).
Satz 5.12 (5.5)
Zwei verschiedene positiv definite reduzierte quadratische Formen sind nicht äquivalent. Also ist h ( d ) genau
die Anzahl der Äquivalenzklassen nicht äquivalenter Formen mit Diskriminante d .
37
Kapitel 5
Quadratische Formen
Beweis
Sei f ( x , y ) = ax 2 + bxy + cy 2 positiv definit und reduziert, also insbesondere a > 0 , c > 0 . Sind x , y ∈ Z \ { 0 }
mit x ≥ y , so gilt:
f ( x, y ) ≥ a x 2 − b x y + c y 2 = x (a x − b y ) + c y 2
≥ x (a x − b x ) + c y 2 = x 2 (a − b ) + c y 2
≥ 1 ⋅ ( a − b ) + c ⋅ 1 = a − b + c.
Analog folgt dies für x ≤ y . Insgesamt folgt für alle x , y ∈ Z \ { 0 } : f ( x , y ) ≥ a − b + c . Wir betrachten
nun die kleinsten Werte, die f ( x , y ) für ( x , y ) = 1 annehmen kann. Das sind auch die kleinsten Werte, die
f ( x , y ) überhaupt annehmen kann, denn sei f ( dx , dy ) = a ( dx )2 + b ( dx ) ( dy ) + c ( dy )2 . Für x ≠ 0 und
y ≠ 0 ist dies der Wert a − b + c , indem x = 1 und y = 1 gewählt werden. Für x = 1 und y = 0 ergibt
sich f ( 1, 0 ) = a , für x = 0 , y = 1 erhalten wir f ( 0,1 ) = c . Da f reduziert ist, sind die drei kleinsten positiven Werte, die von f angenommen werden, die Zahlen a ≤ c ≤ a − b + c . Ist f ′ ( x , y ) = a ′x 2 + b ′xy + c ′y 2
eine zu f äquivalente positiv definite reduzierte quadratische Form, so nimmt f ′ nach Lemma 5.5 (ii) für
( x , y ) = 1 dieselben Werte wie f an, allerdings unter Umständen mit anderen Argumenten x , y . Analog zu
obiger Überlegung folgt, dass f ′ die kleinsten positiven Werte a ′ ≤ c ′ ≤ a ′ − b ′ + c ′ annimmt. Also folgt
aus der Gleichheit der Wertemengen: a = a ′ liefert c = c ′ und dies wiederum b = b ′ . Es genügt zu zeigen,
dass aus b = −b ′ stets b = 0 folgt. Da f reduziert ist, wissen wir, dass −a < b ≤ a < c oder 0 ≤ b ≤ a = c
gilt. Entsprechendes gilt für f ′ , also insgesamt
−a ′ = −a < b ′ = −b ≤ a ′ = a < c ′ = c oder 0 ≤ b ′ = −b ≤ a ′ = a = c ′ = c .
Es folgt a < c , denn für a = c ergäbe sich sofort b ≥ 0 und −b ≥ 0 , woraus b = 0 folgt. Damit ergibt sich
b < a , also zusammen −a < b < a < c . Nach den eingangs gemachten Überlegungen folgt für x , y ∈ Z \ { 0 } :
f ( x, y ) ≥ a − b + c > c > a ,
>0
d.h. f ( x , y ) = a nur für x = ±1 und y = 0 . Sei
⎛p q ⎞
T = ⎜⎜ r s ⎟⎟⎟
⎝
⎠
die unimodulare Transformationsmatrix zwischen f und f ′ , also
⎛ ⎛ x ⎞⎞
f ′ ( x , y ) = f ⎜⎜T ⎜⎜ y ⎟⎟
⎟⎟
⎟⎟ .
⎝ ⎝ ⎠⎠
Also gilt wegen a ′ = a :
⎛ ⎛ 1 ⎞⎞
⎟⎟ = f ( p, r ) .
a = f ′ ( 1, 0 ) = f ⎜⎜T ⎜⎜ ⎟⎟
⎟⎟
⎜⎝ ⎜⎝ 0 ⎠⎠
⎟⎟
Hieraus folgt aber p = ±1 und r = 0 . Außerdem gilt:
1 = detT = ps − qr = ps = ±1 ⋅ s , also s = ±1 .
Analog folgt wegen c ′ = c :
⎛ ⎛ 0 ⎞⎞
⎟⎟
c = f ′ ( 0,1 ) = f ⎜⎜T ⎜⎜ ⎟⎟
⎟⎟ = f ( q, s ) .
⎜⎝ ⎜⎝ 1 ⎠⎠
⎟⎟
Also folgt wiederum q = 0 (und s = ±1 ). Also ist
⎛ ±1 0 ⎞⎟
⎟.
T = ⎜⎜
⎜⎝ 0 ±1 ⎠⎟⎟
Sind F bzw. F ′ die zu f bzw. f ′ gehörige Matrix, so folgt:
⎛ ±1 0 ⎞⎟ ⎛⎜ a b2 ⎞⎟ ⎛ ±1 0 ⎞⎟ ⎛⎜a b2 ⎞⎟
⎟⎜
⎟=⎜
F ′ = T T FT = ⎜⎜
⎟⎟ ⎜⎜
⎟⎟ = F ,
⎝⎜ 0 ±1 ⎠⎟⎟ ⎜⎝⎜ b2 c ⎠⎟ ⎝⎜ 0 ±1 ⎠⎟⎟ ⎝⎜⎜ b2 c ⎠⎟
also f ′ = f .
Definition 5.13
Eine Zahl n ∈ Z wird von einer binären Form f ( x , y ) eigentlich dargestellt, falls f ( x , y ) = n für gewisse
x , y ∈ Z mit ( x , y ) = 1 ist.
Satz 5.14 (5.6)
Seien d ∈ Z mit d < 0 , n ∈ Z gegeben. Es gibt eine binäre quadratische Form mit Diskriminante d , die n
genau dann eigentlich darstellt, wenn x 2 ≡ d mod 4n lösbar ist.
38
Kapitel 5
Quadratische Formen
Beweis
„ ⇒ “ Sei f quadratische Form mit d f = d und n = f ( p, r ) für geeignete
q, s ∈ Z mit ps − qr = 1 ( ggT als Linearkombination). Setze
⎛p q ⎞
T = ⎜⎜ r s ⎟⎟⎟ .
⎝
⎠
( p, r )
= 1 . Dann existieren
Es folgt unmittelbar:
⎛ a b2 ⎞⎟
⎛ ap 2 + bpr + cr 2 *1 ⎞⎟ ⎛ f ( p, r ) *1 ⎞
⎟⎟ ,
⎟⎟T = ⎜⎜
⎟⎟ = ⎜⎜
T T ⎜⎜⎜ b
⎟
⎜⎜
⎜
⎟
*
*
⎜
*
*
⎜⎝ 2 c ⎠⎟
1
2
1
2⎠
⎝
⎠⎟
⎝
d.h. f ist äquivalent zu f ′ ( x , y ) = f ( p, r ) x 2 + b ′xy + c ′y 2 . Nach Lemma 5.5 (iii) haben f und f ′ dieselbe
Diskriminante, also folgt aus
2
2
d = (b ′ ) − 4 f ( p, r )c ′ = (b ′ ) − 4nc ′ ,
2
dass (b ′ ) ≡ d mod 4n gilt.
„ ⇐ “ Sei b 2 ≡ d mod 4n . Wir setzen a = n und c =
kriminante d ,
b 2 −d
4n
∈ Z . Dann hat f ( x , y ) = ax 2 + bxy + cy 2 die Dis-
d f = b 2 − 4ac = b 2 − 4n
b2 − d
=d,
4n
und stellt n eigentlich dar, nämlich f ( 1, 0 ) = a = n .
Korollar 5.15 (5.7)
Sei h ( d ) = 1 für ein d < 0 und sei f eine beliebige positiv definite quadratische Form mit d f = d . Ein
geeignetes n ∈ N wird von f genau dann eigentlich dargestellt, wenn x 2 ≡ d mod 4n lösbar ist.
Beweis
Wegen h ( d ) = 1 sind alle Formen mit der Diskriminante d äquivalent, stellen also dieselbe Wertemenge dar.
Die Behauptung folgt aus Satz 5.14.
Betrachten wir nun
⎛ 1 0 ⎞⎟ ⎛ x ⎞
⎟⎜ ⎟ .
f ( x , y ) = x 2 + y 2 = ( x , y ) ⎜⎜
⎜⎝ 0 1 ⎠⎟⎟ ⎜⎝ y ⎠⎟⎟
Wir erhalten:
{n ∈ N | n = x 2 + y 2 } = { 0,1, 2, 4, 5, 8, 9,10,13,16,17,18,...} .
Charakterisierung dieser Menge: Betrachten wir p = x 2 + y 2 , so erhalten wir
{ p ∈ P | p = x 2 + y 2 } = { 2, 5,13,17, 29, 37, 41, 61,... } .
Nach näherer Betrachtung drängt sich die Vermutung auf, dass aus p ≡ 3 mod 4 stets p ≠ x 2 + y 2 gilt, denn
wegen x 2 ≡ 0,1mod 4 und y 2 ≡ 0,1 mod 4 haben wir x 2 + y 2 ≡ 0,1, 2 mod 4 (sogar für beliebige n ∈ N ).
Satz 5.16 (5.8) (Zwei-Quadrate-Satz)
Sei n ∈ N . Dann ist n Summe von zwei Quadraten ( n ∈ 2 := { x 2 + y 2 | x , y ∈ Z } ) genau dann, wenn für
alle p|n mit p ≡ 3 mod 4 gilt: 2|e p ( n ) .
Insbesondere gilt für p ∈ P : Es ist genau dann p ∈ 2 , wenn p = 2 oder p ≡ 1 mod 4 .
Beweis
Die Aussage für Primzahlen folgt sofort aus der allgemeinen Aussage:
„ ⇒ “ Gelte p|n für p ≡ 3 mod 4 . Nach Voraussetzung ist n = x 2 + y 2 . Also gilt x 2 ≡ −y 2 mod p , also
2⎞
⎛ 2⎞
⎛ 2⎞
p −1 ⎛
⎛ x 2 ⎞⎟
⎜⎜ ⎟ = 1, 0 , ⎜⎜ −y ⎟⎟ = −1 ⎜⎜ y ⎟⎟ = ( −1 ) 2 ⎜⎜ y ⎟⎟ = −1, 0 .
⎜⎝ p ⎠⎟
⎝⎜ p ⎠⎟
⎝⎜ p ⎠⎟
p ⎝⎜ p ⎠⎟
( )
Es folgt also:
⎛ x 2 ⎞⎟ ⎛ −y 2 ⎞⎟
⎜⎜ ⎟ = ⎜⎜
⎟=0
⎝⎜ p ⎠⎟ ⎝⎜ p ⎠⎟
und damit p|y und wegen n = x 2 + y 2 auch p|x . Insgesamt folgt aus p 2 |y 2 und p 2 |x 2 : p 2 |x 2 + y 2 , also
x 2 y2
n
+ 2 = 2 ∈ Z.
p2
p
p
Sei n ′ := pn2 ∈ 2 . Induktion liefert, dass für alle p |n ′ mit p ≡ 3 mod 4 gilt: 2|ep ( n ′ ) .
39
Kapitel 5
Quadratische Formen
„ ⇐ “ Sei n = m 2k mit k quadratfrei ( k heißt quadratfreier Kern von n ), also ist k = p1 ⋅ ... ⋅ ps mit
2
2
pi ≠ p j für i ≠ j nach Voraussetzung. Außerdem sei pi ∈ { 2 } ∪ { p ≡ 1 mod 4 } . Ist k = ( x ′ ) + ( y ′ ) , so
2
2
gilt n = ( mx ′ ) + ( my ′ ) . Die quadratische Form f ( x , y ) = x 2 + y 2 ist positiv definit mit der Diskriminante
d f = −4 . Nach früherem Beispiel 5.11 ist h ( −4 ) = 1 . Also ist nach Korollar 5.15 ausreichend zu zeigen,
dass x 2 ≡ −4 mod 4n lösbar ist. Setze x := 2x ′ . Also haben wir x ′2 ≡ −1mod n zu lösen. Zu zeigen ist, ob
x i2 ≡ −1mod pi für 1 ≤ i ≤ s
lösbar ist. Für pi = 2 gibt es trivialerweise eine Lösung, für alle anderen pi ist die Kongruenz nach dem 1.
Ergänzungssatz (Satz 4.6) lösbar. Nach dem chinesischen Restsatz gibt es also eine Lösung x ′ , woraus die
Behauptung folgt.
Bemerkung 5.17
(i) Es gilt die Identität
nm = ( x 2 + y 2 ) ( u 2 + v 2 ) = ( xu + yv )2 + ( xv − yu )2 .
Also ist jede zusammengesetzte Zahl nm ∈ 2 , sofern n ∈ 2 und m ∈ 2 . Also genügt es Satz 5.16 nur
für Primzahlen zu beweisen.
(ii) Der obige Beweis ist nicht konstruktiv. Ein konstruktives Verfahren geht zurück auf Fermat (siehe folN0 , 3 N0 .
genden Satz). Man kann leicht zeigen, dass gilt: 2
Satz 5.18 (5.9) (Vier-Quadrate-Satz)
Es gilt:
4 = N0 .
D.h. jede natürliche Zahl n lässt sich als Summe von vier Quadraten darstellen, d.h. es gibt x1, x 2, x 3, x 4 ∈ Z
mit n = x12 + x 22 + x 32 + x 42 .
Beweis
Wir verwenden Fermats Methode des Abstiegs.
Behauptung 1: P ⊆ 4 . Wegen 2 = 12 + 12 + 02 + 02 ∈ 4 können wir p > 2 annehmen. Nach dem Beweis zu
1
paarweise inkongruent mod p . Also gilt:
Lemma 4.3 sind die folgenden Zahlen x12 für 0 ≤ x1 ≤ p −
2
p −1
p +1
2
x1 mod p | 0 ≤ x1 ≤
=
2
2
und damit
p −1
p +1
−1 − x 22 mod p | 0 ≤ x 2 ≤
=
.
2
2
p −1
2
2
Folglich existiert unter den Paaren x1 , −1 − x 2 für 0 ≤ x1, x 2 ≤ 2 mindestens ein Paar mit
x12 ≡ −1 − x 22 mod p .
{
}
{
}
Also gilt
x12 + x 22 + 12 ≡ 0 mod p .
Es existiert somit ein m ∈ N mit mp = x12 + x 22 + 12 + 02 ∈ 4 . Dabei gilt:
p −1 2
p −1 2
1 ≤ mp = x12 + x 22 + 12 ≤
+
+ 1 < p2 .
2
2
Also gilt: 1 ≤ m < p . Setze l p = min {l ∈ N | lp ∈ 4 } . Offenbar ist m ∈ {l ∈ N | lp ∈ 4 } . Es folgt:
1 ≤ lp ≤ m < p .
(
) (
)
Behauptung: 2 ? l p . Angenommen 2|l p , dann sind 0, 2 oder 4 der Zahlen x1, x 2, x 3, x 4 mit
l p p = x 12 + x 22 + x 32 + x 42
ungerade. Nach Umsortieren sind also x1 + x 2 , x1 − x 2 , x 3 + x 4 , x 3 − x 4 gerade. Wegen
x1 + x 2 2
x − x2 2
x + x4 2
x − x4 2
x2 x2 x2 x2
1
+ 1
+ 3
+ 3
= 1 + 2 + 3 + 4 = lp p ∈ 4
2
2
2
2
2
2
2
2
2
folgt der Widerspruch zur Minimalität von l p .
Behauptung: l p = 1 . Angenommen l p > 1 . Sei also l p p = x12 + x 22 + x 32 + x 42 . Seien außerdem x1′, x 2′ , x 3′ , x 4′ die
2
2
2
2
betragskleinsten Reste der x i mod l p (also x i′ ≤ 12 l p ). Sei n := ( x1′ ) + ( x 2′ ) + ( x 3′ ) + ( x 4′ ) ∈ 4 . Offenbar
ist n ≡ 0 mod l p und n > 0 (sonst folgt aus x i′ = 0 für alle i , dass l p |x i gilt. Aus
x1 2
x 2
x 2
x 2
p
+ 2 + 3 + 4 =
lp
lp
lp
lp
lp
(
) (
) (
) (
)
( ) ( ) ( ) ( )
folgt, dass l p = 1 ist, da l p < p gilt.). Da 2 ? l p , folgt
40
Kapitel 5
Quadratische Formen
( )
lp 2
= l p2 ,
2
also gibt es n = kl p mit 1 ≤ k < l p . Es gilt die Identität
( x12 + x 22 + x 32 + x 42 ) ( ( x1′ )2 + ( x 2′ )2 + ( x 3′ )2 + ( x 4′ )2 )
1≤n <4
2
2
= ( x1x1′ + x 2x 2′ + x 3x 3′ + x 4x 4′ ) + ( x1x 2′ − x1′x 2 + x 3x 4′ − x 3′ x 4 )
(5.1)
2
2
+ ( x1x 3′ − x1′x 3 + x 2x 4′ − x 2′x 4 ) + ( x1x 4′ − x1′x 4 + x 2x 3′ − x 2′x 3 ) .
Also folgt für m, n ∈ 4 stets mn ∈ 4 . Weiter folgt:
( l p p ) n = ( l p p )( l pk ) = l p2kp ∈ 4 .
Wegen
x1x1′ + x 2x 2′ + x 3x 3′ + x 4x 4′ ≡ x12 + x 22 + x 32 + x 42 ≡ 0 mod l p
und
x i x j′ − x j x i′ ≡ x i x j − x j x i = 0 mod l p
sind die vier Quadratzahlen x1′′, x 2′′, x 3′′, x 4′′ ,
x1′′ := x1x1′ + x 2x 2′ + x 3x 3′ + x 4x 4′ , x 2′′ := x1x 2′ − x1′x 2 + x 3x 4′ − x 3′ x 4 ,
x 3′′ := x1x 3′ − x1′x 3 + x 2x 4′ − x 2′x 4 , x 2′′ := x1x 4′ − x1′x 4 + x 2x 3′ − x 2′x 3 ,
auf der rechten Seite von (5.1) durch lp2 teilbar und es folgt:
⎛ x ′′ ⎞2 ⎛ x ′′ ⎞2 ⎛ x ′′ ⎞2 ⎛ x ′′ ⎞2
kp = ⎜⎜ 1 ⎟⎟ + ⎜⎜ 2 ⎟⎟ + ⎜⎜ 3 ⎟⎟ + ⎜⎜ 4 ⎟⎟ ∈ 4 .
⎝⎜ l p ⎠⎟
⎝⎜ l p ⎠⎟
⎝⎜ l p ⎠⎟
⎝⎜ l p ⎠⎟
Also ist kp Summe von vier Quadraten, was aber wegen k < l p zum Widerspruch zur Minimalität von l p
führt, aus dem nun wegen m = l p = 1 die Behauptung 1 folgt.
Behauptung 2: N 0 ⊆ 4 . Die Fälle n = 0 und n = 1 sind trivial. Ist n = p1 ⋅ ... ⋅ ps (nicht notwendigerweise
verschiedene Faktoren), so folgt aus (5.1) induktiv die Behauptung 2.
Beispiel 5.19
Sei n = 34 = 2 ⋅ 17 ( 2 = 12 + 12 + 02 + 02 ), 17 ∈ 4 . Mittels Beweismethode gilt:
2
2
2
2
3 ⋅ 17 = 51 = 72 + 12 + 12 + 02 ≡ ( x1′ ) + ( x 2′ ) + ( x 3′ ) + ( x 4′ ) = 12 + 12 + 12 + 02 = 3 = n .
Aus der Reduktion mod l17 folgt:
( 3 ⋅ 17 ) n = ( 7 ⋅ 1 + 1 ⋅ 1 + 1 ⋅ 1 + 0 ⋅ 0 )2 + ( 7 ⋅ 1 − 1 ⋅ 1 + 1 ⋅ 1 − 0 ⋅ 1 )2 + ( 7 − 1 + 0 − 0 )2 + ( 0 − 0 + 1 − 1 )2
= 92 + 62 + 62 + 02.
Mit
17 =
( 93 ) + ( 63 ) + ( 63 ) + ( 03 )
2
2
2
2
= 32 + 22 + 22 + 02
erhalten wir
34 = 2 ⋅ 17 = ( 12 + 12 + 02 + 02 )( 32 + 22 + 22 + 02 ) = 52 + 12 + 22 + 22 .
Es gilt aber auch:
34 = 52 + 32 + 02 + 02 = 42 + 42 + 12 + 12 = 42 + 32 + 32 + 02 .
Bemerkung 5.20
(i) Wie das Beispiel 5.19 schon zeigt, ist die Darstellung einer Zahl als Summe von vier (wie auch von zwei)
Quadraten i.A. nicht eindeutig.
(ii) Der Vier-Quadrate-Satz ist bestmöglich, d.h. drei Quadrate reichen i.A. nicht aus. Alle Zahlen der Form
n ≡ 7 mod 8 lassen sich nicht als Summe von drei Quadraten darstellen, denn
x 2 ≡ 0,1, 4 mod 8 ⇒ x 2 + y 2 + z 2 ≡ 7 mod 8 .
Allgemeiner wollen wir für beliebiges k ∈ N wissen, ob ein s := s ( k ) derart existiert, dass für alle n ∈ N es
x1,..., xs ∈ Z gibt mit n = x1k + ... + xsk (Waring-Problem). Hilbert gelang es 1909, es zu lösen. Besser waren
da aber Hardy und Littlewood.
41
42
Kapitel 6
Primzahlverteilung
Wir wissen seit Euklid, dass P = ∞ ist. Wir definieren die Primzahlzählfunktion mit
π ( x ) := ∑ 1 .
p ≤x
Satz 6.1 (Primzahlsatz – PZS)
Es ist
π (x ) ∼
x
log x
„asymptotisch gleich“ oder es gilt äquivalent dazu
π (x )
→ 1 für x → ∞ .
x
log x
Vermutet von Legendre und Gauß um 1800, bewiesen von Hadamard und de la Vallée Poussin um 1896.
Riemann veröffentlichte zu diesem Problem 1859 eine Arbeit „Über die Anzahl der Primzahlen unter einer
gegebenen Größe“, wo er seine Ideen darstellte, wie man an das Problem herangehen könnte. Nebenbei war in
dieser Arbeit auch seine berühmte Vermutung zu finden.
Wir definieren
ϑ ( x ) := ∑ log p = log ( ∏ p ≤x p ) .
p ≤x
Der Zusammenhang zwischen ϑ x
(
und π x
)
(
liefert …
)
Lemma 6.2 (6.1)
Sei 0 < ε < 1 . Dann gilt für alle x ≥ 2
ϑ(x )
1 ϑ(x )
≤ π (x ) ≤
+ x 1−ε .
log x
1 − ε log x
Beweis
Klar ist
ϑ ( x ) = ∑ log p ≤ ∑ log x = log x ∑ 1 = log x ⋅ π ( x ) .
p ≤x
p ≤x
Außerdem ist
ϑ(x ) ≥
p ≤x
∑ log 2 = log 2 ⋅ π
(
x)
p ≤x
zu schlecht, also
ϑ (x ) ≥ ∑
x 1−ε < p ≤x
log p ≥
∑
log ( x 1−ε ) = ( 1 − ε ) log x
x 1−ε < p ≤x
1−ε
(
)
≥ ( 1 − ε ) log x ( π x − x
Dies ist die obere Abschätzung.
∑
1 = ( 1 − ε ) log x ( π ( x ) − π ( x 1−ε ) )
x 1−ε < p ≤x
).
Lemma 6.3 (6.2) (Sehr schwache Stirling-Formel)
Für n ∈ N>1 gilt:
n log n − n + 1 < log ( n ! ) < n log n .
Beweis
Wegen der strengen Monotonie der Logarithmus-Funktion gilt für jedes j ∈ N
∫j
j +1
log x dx = [ x log x − x ]jj +1 < 1 ⋅ log ( j + 1 ) .
Kapitel 6
Primzahlverteilung
Es folgt
n
∫1
n −1
log x dx =
∑ ( ∫j
j +1
n −1
)
log x dx <
j =1
∑ log ( j + 1) = log ( n ! )
j =1
und
n
∫1
log x dx = [ x log x − x ]1n = n log n − n + 1 .
Aus n ! = 1 ⋅ 2 ⋅ ... ⋅ n < n ⋅ n ⋅ ... ⋅ n = n n folgt log ( n ! ) < log ( n n ) = n log n .
Lemma 6.4 (6.3)
Für n ∈ N>1 gilt:
Pn :=
∏ p = eϑ n
)
(
< 4n .
p ≤n
D.h. die Primzahldichte nimmt ab.
Beweis (nach Paul Erdõs)
Wir führen Induktion nach n . Die Induktionsvoraussetzung ist für n = 2 erfüllt. Die Behauptung gelte für
alle k ≤ n .
1. Fall: n + 1 ist keine Primzahl. Dann gilt:
Pn +1 = ∏ p = ∏ p < 4n < 4n +1 .
p ≤n +1
p ≤n
2. Fall: n + 1 ist Primzahl, d.h. n + 1 = 2m + 1 ∈ P . Dann gilt:
Pn +1 = ∏ p
∏ p = Pm +1
p ≤m +1 m +1< p ≤2m +1
∏
p.
m +1< p ≤2m +1
Offensichtlich teilt jedes p mit m + 1 < p ≤ 2m + 1 den Zähler von
( 2m + 1 ) ⋅ 2m ⋅ ... ⋅ ( m + 2 )
2m + 1
=
,
m
m!
aber nicht den Nenner. Also gilt:
2m + 1
∏ p| m .
m +1< p ≤2m +1
(
)
(
Damit gilt
∏
p≤
m +1< p ≤2m +1
und nach Induktionsvoraussetzung gilt
Pn +1 ≤ 4m +1
)
( 2mm+ 1 )
( 2mm+ 1 ) .
Wegen
( 2mm+1 )
( 13 ) = 3 < 4 und für m > 1
gilt
( 2mm−−11 )
( 2mm+ 1 ) < 4
m
also erhalten wir
Pn +1 ≤ 4m +1
( 2mm+ 1 ) < 4
m +1
=
2m ( 2m + 1 )
<4
m (m + 1)
,
⋅ 4m = 42m +1 = 4n +1 .
Satz 6.5 (6.4) (Chebyshev, 1852)
Es gibt Konstanten 0 < C 1 < C 2 derart, dass für alle x ≥ 2 gilt
x
x
C1
≤ π ( x ) ≤ C2
.
log x
log x
Beweis (nach Erdõs & Shapiro)
Wir werden zeigen, dass es 0 < B1 < B2 gibt mit
B1x ≤ ϑ ( x ) ≤ B2x .
Dies genügt zum Beweis des Satzes, denn mit Lemma 6.2 folgt:
x
ϑ(x )
1 ϑ(x )
x
1
≤
≤ π (x ) ≤
+ x 1−ε ≤ ( 2B2 + 1 )
für ε = ,
B1
log x
log x
1 − ε log x
log x
2
43
(6.1)
Kapitel 6
Primzahlverteilung
denn es gilt:
1 ϑ(x )
x
x
x
+ x 1−ε ≤ 2B2
+ x ≤ 2B2
+
,
1 − ε log x
log x
log x log x
da log x ≤ x für x ≥ 2 ist, also
x
x ≤
.
log x
Setze C 1 = B1 und C 2 = 2B2 + 1 . Zu (6.1) folgt die obere Abschätzung sofort aus Lemma 6.4:
ϑ ( x ) = log ( ∏p ≤x p ) < log ( 4x ) = x ( log 4 ) .
=:B2
Es bleibt die untere Abschätzung. Mit Satz 2.3 folgt:
log ( n ! ) = log ( ∏p ≤n pep
(n !)
n
= ∑ ⎡⎢ ⎤⎥ log p + O
p ≤n ⎣ p ⎦
∞
n
∞
n
n
) = ∑ ep ( n ! ) log p = ∑ ∑ ⎡⎢⎣ p j ⎤⎥⎦ log p = ∑ ⎡⎢⎣ p ⎦⎤⎥ log p + ∑ ∑ ⎡⎢⎣ p j ⎤⎥⎦ log p
(
p ≤n
)
p ≤n j =1
p ≤n
n
∑ p ≤n ∑ j =2 p j log p .
∞
p ≤n j =2
Wegen
∞
1
∑ pj =
j =2
∞
1
∑ pj
−1−
j =0
( p + 1 )( p − 1 )
1
1
p +1
p2
1
2
=
−
=
−
=
<
1
p
p
p ( p − 1)
p ( p − 1)
p ( p − 1 ) p2
1− p
folgt
∞
∞
n
1
∑ ∑ p j log p = n ∑ log p ∑ p j
p ≤n j =2
Nach Lemma 6.3 folgt aus
p ≤n
j =2
∞
log p
log m
(
)
2 ≤ 2n ∑
2 =O n .
p
p ≤n
m =1 m
< 2n ∑
= n log n + O ( n )
n
∑ ⎡⎢ ⎤⎥ log p = log ( n ! ) + O ( n ) = n log n + O ( n ) .
p ≤n ⎣ p ⎦
log ( n ! )
Jetzt benutzen wir die obere Abschätzung für ϑ ( x ) zur Herleitung einer unteren Abschätzung:
log p
n
n
n
n
n
n∑
= ∑ log p = ∑ ⎡⎢ ⎤⎥ +
log p = ∑ ⎡⎢ ⎤⎥ log p + ∑
log p
p
p
p
p
p
p ≤n
p ≤n
p ≤n ⎣ ⎦
p ≤n ⎣ ⎦
p ≤n p
= n log n + O ( n ) + O ( ϑ ( n ) ) = n log n + O ( n ) .
Also folgt
log p
∑ p = log n + O ( 1 ) .
p ≤n
(
{ })
{ }
(6.2)
(Diese Aussage nennt man übrigens „Satz von Mertens“, die den nächsten Satz darstellt.)
Ist a ∈ R mit 0 < a < 1 , so folgt für n ≥ 2
log p
log p
log p
∑ p = ∑ p − ∑ p = log n + O ( 1 ) − ( log (an ) + O ( 1 ) ) = log n − log a − log n + O ( 1 )
an < p ≤n
p ≤n
p ≤an
= − log a + O ( 1 ) ≥ − log a − C
für eine passende Konstante C . Trivial ist
log p
log p
1
1
1
∑ p ≤ ∑ an = an ∑ log p ≤ an ∑ log p = an ϑ ( n ) .
an < p ≤n
an < p ≤n
an < p ≤n
p ≤n
Zusammengefasst ergibt sich
( )
1
1
1 ( )
ϑ n ≥ − log a − C
ϑ ( n ) ≥ log
−C .
an
an
a
Wähle a = e−(C +1 ) und es folgt
ϑ ( n ) ≥ an ( log (eC +1 ) − C ) = an ( (C + 1 ) log e − C ) = an ,
wobei wir B1 := a setzen. Im vorhergehenden Beweis wurde u.A. (6.2) gezeigt.
Im Laufe des oberen Beweises haben wir zusätzlich Folgendes gezeigt:
Satz 6.6 (6.5) (Satz von Mertens)
Für n ∈ N>1 gilt
log p
= log n + O ( 1 ) .
p ≤n p
∑
Analog zur partiellen Integration gilt:
44
Kapitel 6
Primzahlverteilung
Lemma 6.7 (6.6) (Abelsche Teilsummation)
Sei λ1 < λ2 < ... eine reelle Folge mit limn →∞ λn = ∞ . Seien an ∈ C für n ∈ N und x ∈ R mit x ≥ λ1 .
Wir setzen
A ( x ) = ∑ an .
λn ≤x
Sei f : R
≥λ1
→ C stetig differenzierbar. Dann gilt:
∑ an f ( λn ) = A
x
(
λn ≤x
x ) f ( x ) − ∫ A ( t ) f ′ ( t )dt .
λ1
Beweis
Wegen λn → ∞ sind die auftretenden Summanden alle endlich. Es gilt:
A(x ) f (x ) −
∑ an f ( λn ) = ∑ an f
λn ≤x
(
∑ an f ( λn ) = ∑ an ( f
x)−
λn ≤x
Wegen λ1 ≤ λn ≤ t ≤ x gilt:
x
∑ an ∫λ
λn ≤x
f ′ ( t ) dt =
n
λn ≤x
x
∫λ
1
(
x ) − f ( λn ) ) =
λn ≤x
f ′ (t )( ∑λ
n ≤t
an )dt =
x
∑ an ∫λ
λn ≤x
x
∫λ
f ′ ( t ) dt .
n
f ′ ( t ) A ( t ) dt .
1
Satz 6.8 (6.7)
Es existiert eine Konstante C 3 ∈ R , so dass für x ≥ 2 gilt:
1
1
∑ p = log log x + C 3 + O log x .
p ≤x
(
Beweis
( )
( )
Sei p n die n -te Primzahl ( p 1 = 2 , p 2 = 3 ). Dann gilt:
1
1
log p
∑p = ∑
n = ∑
n
p
n
n
p ≤x
p ≤x
p ≤x p
(
)
)
(
(
)
(
)
(
)
(
n)
)
1
log p
(
n)
.
Setze
λn := p
(
n)
, an :=
log p
pn
(
(
n)
)
und f ( t ) :=
1
.
log t
Nach Satz von Mertens (6.6) gilt
A(x ) =
log p
∑
n
n
p ≤x p
(
(
(
n)
)
=
)
log p
= log x + O ( 1 ) = log x + r ( x )
p ≤x p
∑
mit einer beschränkten und stückweise stetigen Funktion r ( x ) , wobei
log p
∑ p
p ≤x
Treppenfunktion ist. Abelsche Teilsummation (6.7) liefert:
x
x
1
1
dt
∑ p = A ( x ) f ( x ) − ∫p 1 A ( t ) f ′ ( t )dt = ( log x + r ( x ) ) log x − ∫2 ( log t + r ( t ) ) −t ( log t )2
p ≤x
x dt
x
1
r (t )
= 1 +O
+∫
+∫
dt
2 t log t
2 t ( log t )2
log x
x
1
r (t )
= 1 +O
+ [ log log x ]2x + ∫
dt.
2 t ( log t )2
log x
Integration durch Substitution liefert für v := log t
log x r (e v )
∞ r (e v )
∞ r (e v )
=
−
dv
dv
∫log 2 v 2
∫log 2 v 2
∫log x v 2 dv .
Wegen
∞ r (e v )
∞ r (e v )
∞ C
C
⎡ 1 ⎤∞
∫log 2 v 2 dv ≤ ∫log 2 v 2 dv ≤ ∫log 2 v 2 dv = C ⋅ ⎢⎣ − v ⎦⎥ log 2 = log 2 .
existiert ein B3 mit
∞ r (e v )
B3 := ∫
dv .
log 2 v 2
Analog gilt:
∞ r (e v )
∞ 1
1
⎛ ⎡ 1 ⎤∞ ⎞
∫log x v 2 dv = O ∫log x v 2 dv = O ⎜⎜⎝ ⎢⎣ − v ⎥⎦ log x ⎠⎟⎟ = O log x .
( )
(
(
)
)
(
)
45
(
)
Kapitel 6
Primzahlverteilung
Insgesamt erhalten wir
∑ p = log log x + 1 − log log 2 + B3 + O ( log x ) .
1
1
p ≤x
Also existiert ein C 3 := 1 − log log 2 + B3 .
Eine weitere zahlentheoretische Funktion ist
∑1 .
ω ( n ) :=
p |n
Sie ist additiv, d.h. für ( m, n ) = 1 gilt ω ( mn ) = ω ( m ) + ω ( n ) .
Korollar 6.9 (6.8)
Es existiert eine Konstante C 4 > 0 derart, dass für x ≥ 2 gilt
1 −1
−
= C 4 log x + O ( 1 ) .
1
∏
p
p ≤x
(
)
Beweis
Mit
T ( x ) :=
∏ (1 − p )
−1
1
p ≤x
und der Potenzreihe von log gilt:
logT ( x ) =
∞
∑ − log ( 1 − p ) = ∑ ∑ jp j
1
p ≤x
1
=
p ≤x j =1
∞
1
1
∑ p + ∑ ∑ jp j .
p ≤x
p ≤x j =2
Trivialerweise gilt:
∞
0 < B4 :=
1
∑ ∑ jp j
p ∈ P j =2
∞
<
∞
∑ 2n ( n − 1 ) = 2 ∑ ( n − 1 − n ) = 2 ,
1
1
n =2
1
1
1
n =2
also ist B4 wohldefiniert. Außerdem haben wir
∞
1
1
1
1
1
1
1
=
−
=
=O
.
0 < ∑∑ j < ∑
∑
)
(
[
]
x
2 n ≥ x +1 n − 1 n
2 x
p >x j =2 jp
n >x 2n n − 1
(
[
Wegen
(
)
( )
]
1
1
=O
x
log x
)
folgt mit Satz 6.8
logT ( x ) = log log x + C 3 + C 4 + O
( log1 x ) ,
also wegen
ey = 1 + y +
y2
+ ... mit C 4 := eC 3 +B4
2
gilt:
(
O 1
T ( x ) = C 4 log x ⋅ e ( log x ) = C 4 log x 1 + O
( log1 x )) = C
4
log x + O ( 1 ) .
Korollar 6.10 (6.9)
Mit C 3 wie im Beweis von Satz 6.8 gilt:
∑ω
(
n ) = x log log x + C 3x + O
n ≤x
x
( log x ) .
Beweis
Es gilt:
∑ω
n ≤x
(
n) =
x
x
1
∑ ∑ 1 = ∑ ∑ 1 = ∑ ⎢⎣⎡ p ⎤⎥⎦ = ∑ ( p + O ( 1 )) = x ∑ p + O
n ≤x p|n
p ≤x n ≤x
p| n
p ≤x
p ≤x
Nach dem Satz von Chebyshev (6.5) und Satz 6.8 gilt
1
1
x ∑ + O ( π ( x ) ) = x log log x + C 3 + O
p
log
x
p ≤x
(
(
46
(
π ( x )) .
p ≤x
)) + O ( logx x ) = x log log x + C x + O ( logx x ) .
3
47
Kapitel 7
Diophantische Approximation
und Kettenbrüche
Die diophantische Approximation beschäftigt sich mit der Lösbarkeit von diophantischen Ungleichungen, d.h.
Ungleichungen in ganzzahligen Variablen bzw. Unbekannten. Ein grundlegendes Resultat dieses Gebiets
stammt von Dirichlet (1842).
Im Folgenden sei p nicht mehr prim.
Satz 7.1 (7.1)
Seien α ∈ R und Q ∈ R>1 . Dann gibt es p, q ∈ Z mit ( p, q ) = 1 , 0 < q < Q und
p
1
1
α− <
bzw. αq − p < ,
q
qQ
Q
wobei α irrational ist.
Beweis
Ist die Ungleichung überhaupt lösbar, so erst recht mit ( p, q ) = 1 . Wir setzen zuerst Q ∈ N voraus. Betrachte die Q + 1 Zahlen 0, { α } , { 2α }, { 3α },..., {Q α } (wobei { α } = α − [ α ] ist), alle im Intervall [ 0,1 ] .
Unterteile [ 0,1 ] in Q Teilintervalle ⎡⎢⎣ Qi , i Q+1 ) , i = 0,...,Q − 2 , sowie ⎡⎣⎢ QQ−1 ,1 ⎤⎦⎥ (11 Tauben in 10 Taubenhäuschen). Nach Dirichlets Schubfachprinzip (Taubenschlagprinzip) liegen in mindestens einem Teilintervall zwei
der Q + 1 Zahlen. Die Differenz dieser beiden Zahlen (in der richtigen Reihenfolge) hat offenbar die Form
q α − p für ein 0 < q < Q und geeignetes p ( { k α } − {l α } = ( k − l ) α − ( [ k α ] − [ l α ]) =: q α − p ) und erfüllen außerdem
1
qα − p < .
Q
Für Q ∈ R>1 \ N beliebig liefert die obige Überlegung mit [Q ] + 1 anstelle von Q das Gewünschte, denn aus
0 < q < [Q ] + 1 folgt 0 < q < Q (da q ∈ Z ) und aus
p
1
α− <
q
q ( [Q ] + 1 )
folgt
p
1
α− <
.
q
qQ
Als Korollar ergibt sich ...
Satz 7.2 (7.2) (Dirichletscher Approximationssatz~)
Sei α ∈ R \ Q . Dann gibt es unendlich viele
p
q
∈ Q mit ( p, q ) = 1 , q > 0 und
p
1
α− < 2.
q
q
Beweis
Sei Q > 1 beliebig. Nach Satz 7.1 existieren p, q mit ( p, q ) = 1 , 0 < q < Q und
p
1
1
α− <
<
(1. Lösung).
q
qQ q 2
Wegen α ∉ Q ist α − qp > 0 . Hieraus folgt αq − p > 0 . Definiere
1
Q1 :=
> 1.
αq − p
Kapitel 7
Diophantische Approximation und Kettenbrüche
Nach Satz 7.1 existieren p1, q1 mit ( p1, q1 ) = 1 , 0 < q1 < Q1 und
p
1
p
α− 1 <
< α− ,
q1
q1Q1
q
also ist
p1
p
≠
(2. Lösung).
q1
q
Induktiv erhalten wir unendlich viele Lösungen.
Bemerkung 7.3
(i) Der dirichletsche Approximationssatz (7.2) garantiert Näherungen an eine gegebene Zahl α , die viel genauer sind als wir es z.B. von Dezimalzahlnäherungen erwarten können:
3141
5
1
π−
≈
=
.
1000
10000
2000
Nach Dirichlet existiert ein Näherungsbruch von π mit Nenner unterhalb 1000 derart, dass der Fehler
höchstens 10−6 ist.
(ii) Für α ∈ Q gilt Satz 7.2 nicht, wir haben sogar:
p
p
1
α ∈ Q ⇒ es gibt nur endlich viele Brüche ≠ α mit α − < 2 ,
q
q
q
denn sei α = ab . Für qp ≠ α folgt
p
aq − bp
p
1
1
0< α− =
, also α − ≥
≥ 2
q
bq
q
bq
q
für q ≥ b . Für q < b existieren aber nur endlich viele geeignete Zähler p .
Bemerkung 7.4
Satz 7.2 ist für rationales α falsch.
Wegen der Nichtkonstruktivität des Schubfachschlusses haben wir mit der obigen Methode keine Möglichkeit
die Dirichlet-Näherungen explizit zu konstruieren. Deshalb gibt es Kettenbrüche.
Definition 7.5
Ein endlicher Kettenbruch ist ein Ausdruck der Gestalt
1
a0 +
=: a 0 ; a1,..., an
1
a1 +
1
a2 +
1
... +
1
an −1 +
an
mit a 0 ∈ Z und ai ∈ N , 1 ≤ i ≤ n . Ist an ≠ 1 , so heißt der Kettenbruch normiert. Ein a j heißt j -ter Teilnehmer des Kettenbruchs.
Sei an = 1 . Dann erhalten wir a 0 ; a1,..., an −1 + 1 .
Satz 7.6 (7.3)
Jeder endliche Kettenbruch stellt eine rationale Zahl dar. Umgekehrt wird jede rationale Zahl von genau
einem normierten Kettenbruch dargestellt: Ist ab ∈ Q , so liefert der euklidische Algorithmus in der Form
a = b ⋅ a 0 + r1,
, 0 < r1 < b,
b = r1 ⋅ a1 + r2 ,
.0 < r2 < r1,
rn −1 = rn ⋅ an
die Darstellung
a
= a 0 ; a1,..., an .
b
Beweis
Die erste Aussage ist klar, z.B.
48
Kapitel 7
Diophantische Approximation und Kettenbrüche
1
2+
4+
=2+
1
1+
1
7
1
7
4+
8
=2+
8
86
=
.
39
39
Umkehrung:
a
b ⋅ a 0 + r1
r
1
1
1
1
=
= a0 + 1 = a0 + = a0 + r ⋅ a + r = a0 +
.
r2 = ... = a 0 +
1
b
1
1
2
b
b
b
a1 +
a
+
1
r1
1
r1
r1
... +
an
Eindeutigkeit: Annahme: α = a 0 ; a1,..., an = b0 ;b1,..., bm . Ist a 0 ≠ b0 , so folgt wegen [ a 0 ; a1,..., an ] = a 0
und [ b0 ;b1,..., bm ] = b0 , dass a 0 ; a1,..., an ≠ b0 ;b1,..., bm . Also ist a 0 = b0 , d.h.
1
1
1
1
α = a0 +
= a0 +
= b0 +
= b0 +
.
1
1
;
,...,
;
,...,
a
a
a
b
b
bm
n
1 2
1 2
a1 +
b1 +
1
1
... +
... +
an
bm
Damit folgt a1; a2 ,..., an = b1;b2 ,..., bn . Sei o.B.d.A. n ≤ m . Dann folgt induktiv an = an = bn ;bn +1,..., bm .
Also ist bn ;bn +1,..., bm ∈ N und aus
1
bn ;bn +1,..., bm = bn +
1
bn +1 +
1
... +
bm
n = m und damit bn = an , da m = n + 1 und bn +1 = 1 zu einem Widerspruch wegen Nichtnormiertheit
führt.
Irrationale Zahlen sind i.A. nicht eindeutig, z.B. gilt:
3
= 0.3 = 0.29 .
10
Lemma 7.7 (7.4)
Sei a 0 ∈ Z , ( an )n ∈N eine Folge über N . Wir setzen für n ≥ 0
p
a 0 ; a1,..., an = n für ( pn , qn ) = 1 .
qn
Dann gelten mit p−1 := 1 , p0 := a 0 , q−1 := 0 und q 0 := 1 die Rekursionsformeln
pn = an pn −1 + pn −2 und qn = anqn −1 + qn −2 für n ≥ 1 .
Beweis
Wir führen Induktion nach n . Sei n = 1 . Dann gilt
p1
1
a a +1
:= a 0 ; a1 = a 0 +
= 0 1
q1
a1
a1
n . Wir setzen
und damit ist die Induktionsvoraussetzung erfüllt. Sei also nun n − 1
p j′
a1; a2,..., a j +1 =
mit ( p j′ , q j′ ) = 1 für j ≥ 1 .
q j′
Nach Induktionsvoraussetzung gilt pn′ −1 = an pn′ −2 + pn′ −3 und qn′ −1 = anqn′ −2 + qn′ −3 . Offenbar ist
pj
q j′−1
1
.
= a0 +
= a0 +
qj
p j′−1
p j′ −1
q j′−1
Aus ( p j , q j ) = 1 folgt nun p j = a 0 p j′−1 + q j′ −1 und q j = p j′−1 . Dreimal einsetzen ( j := n, n − 1, n − 2 ) ergibt:
pn = a 0 (an pn′ −2 + pn′ −3 ) + ( anqn′ −2 + qn′ −3 ) = an ( a 0 pn′ −2 + qn′ −2 ) + ( a 0 pn′ −3 + qn′ −3 ) , qn = an pn′ −2 + pn′ −3 ,
pn −1 = a 0 pn′ −2 + qn′ −2 , qn −1 = pn′ −2 ,
pn −2 = a 0 pn′ −3 + qn′ −3 , qn −2 = pn′ −3 .
Korollar 7.8 (7.5)
Für n ≥ 0 gilt:
(i) pnqn −1 − pn −1qn = ( −1 )n +1 ,
(ii) ( pn , qn ) = 1 .
49
Kapitel 7
Diophantische Approximation und Kettenbrüche
Beweis
(i) Wir führen Induktion nach n . Sei n = 0 . Dann gilt: p0q−1 − p−1q 0 = a 0 ⋅ 0 − 1 ⋅ 1 = −1 = ( −1 )0 +1 . Mit
Lemma 7.7 gilt
pnqn −1 − pn −1qn = ( an pn −1 + pn −2 )qn −1 − pn −1 ( anqn −1 + qn −2 ) = − ( pn −1qn −2 − pn −2qn −1 )
= ( −1 )( −1 )n = ( −1 )n +1 .
(ii) Klar nach Lemma 7.7. Die Rekursionsformeln liefern stets die gekürzten Brüche
pn
qn
.
Beispiel 7.9
Es gilt:
2 = 1+
1
1
1
1
= 1+
= 1+
= 1+
= 1;2, 2,...? .
1
1
1
2, 4142...
2+
2+
2+
1
1
2, 4142...
2+
2+
1
2, 4142...
2+
...
Definition 7.10
Sei α ∈ R \ Q . Wir setzen θ0 = α und definieren zwei Folgen ( an )n ∈N0 über N (bis auf a 0 ∈ Z ) und
( θn )n ∈N0 über R>1 (bis auf θ0 ∈ R ) rekursiv durch
1
an = [ θn ] und θn = an +
für n ≥ 0 .
θn +1
Dann heißt a 0 ; a1, a2 ,... (unendlicher) Kettenbruch von α .
Bemerkung 7.11
Für α ∈ Q bricht der obige Algorithmus dadurch ab, dass für ein n ∈ N gilt an = θn . Dann erhalten wir
genau den endlichen Kettenbruch a 0 ; a1,..., an von α .
Wir wollen nun zeigen, dass a 0 ; a1, a2 ,... = α gilt.
Satz 7.12 (7.6)
Sei a 0 ; a1, a2 ,... der (endliche oder unendliche) Kettenbruch von α ∈ R . Für
pn
:= a 0 ; a1,..., an für n ≥ 0
qn
gilt:
p
p
p
p
(i) n ≤ α ≤ n +1 oder n +1 ≤ α ≤ n ,
qn
qn +1
qn +1
qn
p
1
,
(ii) α − n ≤
qn
qnqn +1
p
(iii) lim n = α , d.h. α = a 0 ; a1, a2 ,... .
n →∞ qn
Beweis
(i) Nach Definition der θn ist
1
1
α = θ0 = a 0 + = a 0 ; θ1 = a 0 ; a1 +
= a 0 ; a1, θ2 = ... = a 0 ; a1,..., an , θn +1 .
θ1
θ2
Dabei gilt
1
1
≤
.
0<
θn +1 an +1
Also gilt:
1 ≤
1
pn
≤
= a 0 ; a1,..., an
a ; a ,..., an +
a ; a ,..., an +
≥ 0 1
θn +1 ≥ 0 1
qn
an +1
= a 0 ; a1,..., an , an +1 =
=α
(ii) Nach Korollar 7.8 (i) folgt
p
p
p
pq
− qn pn +1
− ( −1 )n +2
1
.
=
=
α − n ≤ n − n +1 = n n +1
qn
qn
qn +1
qnqn +1
qnqn +1
qnqn +1
(iii) Zunächst zeigen wir mit Induktion, dass qn ≥ 2
−21
q0 = 1 ≥ 2
n −1
2
gilt. Die Induktionsvoraussetzung ist mit
1
0
, q1 ≥ 1 = 2 , q2 = a2q1 + q 0 ≥ q1 + q 0 ≥ 2 > 2 2
50
pn +1
.
qn +1
Kapitel 7
Diophantische Approximation und Kettenbrüche
erfüllt. Für n ≥ 3 gilt nach Induktionsvoraussetzung
qn ≥ qn −1 + qn −2 ≥ 2
n −2
2
Aus (ii) folgt dann
lim α −
n →∞
+2
n −3
2
≥ 2⋅2
n −3
2
=2
n −1
2
.
pn
= 0.
qn
Definition 7.13
Sei α = a 0 ; a1, a2 ,... . Dann heißt
pn
:= a 0 ; a1,..., an
qn
n -ter Näherungsbruch von α .
Korollar 7.14 (7.7)
Sei α ∈ R . Jeder Näherungsbruch
p
q
von α erfüllt
α−
p
1
< 2.
q
q
Beweis
Aus Satz 7.12 (ii) folgt wegen qn +1 = an +1qn + qn −1 ≥ qn + qn −1 > qn für n ≥ 1
p
1
1
<
.
α− n <
qn
qnqn +1 qn2
Bemerkung 7.15
(i) Korollar 7.14 zeigt, wie die Dirichlet-Approximationen konstruiert werden können.
(ii) Wie im Satz 7.6 lässt sich zeigen, dass auch irrationale Zahlen eine eindeutige Darstellung als Kettenbruch besitzen. Wir haben also mit Satz 7.6 und Satz 7.12 eine Bijektion zwischen R und der Menge
aller normierten Kettenbrüche.
Auf den Kettenbrüchen lässt sich Dirichlets Approximationssatz noch ein wenig verschärfen:
Satz 7.16 (7.8) (Hurwitzscher Approximationssatz)
Sei α ∈ R \ Q . Dann gibt es unendlich viele
Beweis
Seien
p
q
∈ Q mit q > 0 , ( p, q ) = 1 und
p
1
.
α− <
q
5q 2
pn pn +1
p
,
und n +2
qn qn +1
q n +2
drei aufeinander folgende Kettenbruch-Näherungen von α . Wir zeigen: mindestens eine davon leistet das
Gewünschte. Annahme:
pj
1
für j = n, n + 1, n + 2 .
α−
≥
qj
5q 2j
p
Nach Satz 7.12 (i) haben α − qpnn und α − qnn ++11 verschiedene Vorzeichen. Also gilt:
1
1
p
p
p
p
pq
− qn pn +1
− ( −1 )n +2
1
.
≤ α − n + α − n +1 = n − n +1 = n n +1
=
=
2 +
2
qn
qn +1
qn
qn +1
qnqn +1
qnqn +1
qnqn +1
5qn
5qn +1
Es folgt
qn +1
q
+ n ≤ 5.
qn
qn +1
Sei λ :=
qn +1
qn
. Dann gilt wegen λ ∈ Q
Analog folgt für μ :=
qn + 2
qn +1
λ+
1
<
λ
5.
μ+
1
<
μ
5.
:
Lösen einer quadratischen Gleichung ergibt
51
Kapitel 7
Diophantische Approximation und Kettenbrüche
λ, μ <
Außerdem gilt
μ=
1+ 5
.
2
q n +2
a q
+ qn
q
q
1
= n +2 n +1
= an + 2 + n ≥ 1 + n = 1 + .
qn +1
qn +1
qn +1
qn +1
λ
Weiter gilt:
1+ 5
1
2
1+ 5
.
> μ ≥1+ >1+
=
λ
2
1+ 5
2
Somit erhalten wir den gewünschten Widerspruch.
Bemerkung 7.17
Wir werden später anhand
1+ 5
= 1;1,1,... =: 1
2
5 in Satz 7.16 bestmöglich ist, welche den hurwitzschen Approximationssatz
α=
sehen, dass die Konstante
bestmöglich macht.
Wir wollen zeigen, dass nur Kettenbrüche die Näherungen á la Dirichlet liefern:
Satz 7.18 (7.9)
Für α = a 0 ; a1, a2 ,... gilt:
p θ
+ pn −1
(i) α = n n +1
,
qn θn +1 + qn −1
1
,
(ii) αqn − pn =
qn θn +1 + qn −1
1
p
1
(iii)
< α− n <
,
qn
an +1qn2
( an +1 + 2 )qn2
(iv) für alle p, q mit 0 < q < qn +1 gilt αq − p ≥ αqn − pn .
Beweis
(i) Nach Definition von θn gilt:
α =: θ0 = a 0 +
p0 ( a1 + θ12 ) + p−1
a θ +1
p θ + p−1
1
( a p + p−1 ) θ2 + p0
= 0 1
= 0 1
=
= 1 0
q 0θ1 + q−1
q 0 ( a1 + θ12 ) + q−1
θ1
θ1
( a1q 0 + q−1 ) θ2 + q 0
p1θ2 + p0
= ...
q1θ2 + q 0
(ii) Aus (i) folgt mit Korollar 7.8:
q p θ
+ qn pn −1 pnqn θn +1 + pnqn −1
− pnqn −1
1
q p
αqn − pn = n n n +1
.
−
= n n −1
=
qn θn +1 + qn −1
qn θn +1 + qn −1
qn θn +1 + qn −1
qn θn +1 + qn −1
=
(iii) Nach Definition von θn gilt:
an +1qn < θn +1qn ≤ θn +1qn + qn −1 < ( an +1 + 1 )qn + qn −1 ≤ ( an +1 + 1 )qn + qn = ( an +1 + 2 )qn .
Dann folgt nach (ii):
⎧ < qn (an1+1qn ) ,
⎪
pn
1
1
⎪
=
αqn + pn =
⎨
qn
qn
qn ( qn θn +1 + qn −1 ) ⎪⎪> qn (an +11 +2 )qn .
⎩
p
(iv) Wir setzen für gegebenes q :
pqn +1 − qpn +1
pqn − qpn
u :=
, v :=
,
pnqn +1 − qn pn +1
qn pn +1 − pnqn +1
α−
wobei u, v ∈ Z gilt, da der Nenner ±1 ist. Wegen ( pn +1, qn +1 ) = 1 nach Korollar 7.8 und wegen
p
0 < q < qn +1 ist u ≠ 0 (sonst folgt aus pqn +1 = qpn +1 , dass qp = qnn ++11 gilt, woraus der Widerspruch mit
p = kpn +1 und q = kqn +1 folgt). Es folgt upn + vpn +1 = p und uqn + vqn +1 = q durch nachrechnen.
Wegen 0 < q < qn +1 folgt für v ≠ 0 , dass u und v entgegengesetzte Vorzeichen haben. Nach Satz 7.12
(i) haben auch αqn − pn und αqn +1 − pn +1 entgegengesetzte Vorzeichen. Also folgt:
αq − p = α ( uqn + vqn +1 ) − ( upn + vpn +1 ) = u ( αqn − pn ) + v ( αqn +1 − pn +1 )
≥ u ( αqn − pn ) ≥ αqn − pn .
Falls v = 0 , so gilt pqn − qpn = 0 und es folgt qp = qpnn , woraus p = pn und q = qn folgt.
52
Kapitel 7
Diophantische Approximation und Kettenbrüche
Korollar 7.19 (7.10)
Sei α ∈ R und
p
q
∈ Q . Gilt
α−
p
1
< 2,
q
2q
so ist qp ein Näherungsbruch von α .
Beweis
Wir wählen denjenigen Näherungsbruch qpnn von α mit qn ≤ q < qn +1 . Dann gilt
p pn
p
p
p
p
1
1
1
1
−
=
− α + α − n ≤ α − + α − n = αq − p +
αqn − pn ≤
+
q qn
q
qn
q
qn
q
qn
q qn
nach Satz 7.18 (iv). Nach Voraussetzung und wegen q ≥ qn erhalten wir
p pn
2 1
1
−
<
=
.
q qn
qn 2q
qqn
Also bleibt nur
p
p
= n.
q
qn
(
) (
(
)
) αq − p
Definition 7.20
Eine reelle (oder komplexe) Zahl α heißt algebraisch vom Grad n , falls es ein Polynom
P ( x ) := an x n + ... + a1x + a 0 ∈ Z [ x ] \ { 0 } ,
aber keine solchen Polynome geringeren Grades gibt mit P ( α ) = 0 . Setzen wir voraus, dass an > 0 und
( an ,..., a 0 ) = 1 , so ist P ( x ) eindeutig bestimmt und heißt Minimalpolynom von α . Ist α nicht algebraisch,
so heißt α transzendent.
Bemerkung 7.21
(i) Fast alle Zahlen sind transzendent, denn die Menge A ⊆ R aller algebraischen Zahlen ist abzählbar:
Denn sei n ∈ N fest. Dann gibt es nur abzählbar viele Polynome P ( x ) = an x n + ... + a1x + a 0 ∈ Z [ x ] .
Da jedes P ( x ) höchstens n Nullstellen hat, folgt, dass die Menge der algebraischen Zahlen vom Grad
n abzählbar ist. Da die Menge der natürlichen Zahlen abzählbar ist, ist A als abzählbare Vereinigung
von abzählbaren Mengen wieder abzählbar.
(ii) Es ist schwer zu zeigen, dass gegebene Zahlen (wie z.B. e oder π ) transzendent sind.
Seit Lagrange ist bekannt, dass ein unendlicher Kettenbruch genau dann periodisch ist, wenn die durch ihn
dargestellte Zahl quadratisch irrational ist.
Satz 7.22 (7.11) (Lagrange)
Wird α ∈ R von einem Kettenbruch dargestellt, d.h.
α = a 0 ; a1, a2 ,..., am −1, am ,..., am +l −1
(der Oberstrich zeigt die Periode),
so ist α eine quadratische Irrationalzahl (d.h. α ist Nullstelle eines quadratischen Polynoms über Z ) und
umgekehrt.
Beweis
Nach Satz 7.18 (i) gilt für β := am ; am +1,..., am +l −1
p θ + pm −2
p β + pm −2
,
= m −1
α = m −1 m
qm −1θm + qm −2
qm −1β + qm −2
und wegen der Periodizität von β auch
p ′ β + pl′−2
,
β = l −1
ql′−1β + ql′−2
′
wobei hier qpi′ die Näherungsbrüche von β bezeichnen. Es folgt sofort, dass β eine quadratische Gleichung
i
über Z erfüllt, also auch α . Die Unendlichkeit des Kettenbruches von α impliziert schließlich dessen Irrationalität.
Um 1844 bewies Louville das folgende Ergebnis, das erstmalig die Existenz transzendenter Zahlen zeigte.
53
Kapitel 7
Diophantische Approximation und Kettenbrüche
Satz 7.23 (7.12) (Liouville)
Sei α ∈ C algebraisch vom Grad n ≥ 2 (d.h. irrational). Dann existiert eine positive Konstante c ( α ) derart,
dass für alle qp ∈ Q , q > 0 , gilt:
p
c (α)
α− ≥ n .
q
q
Beweis
Offenbar gilt:
p
p
α − ≥ Re α − .
q
q
Sei also o.B.d.A. α ∈ R . Weiter sei P ( x ) ∈ Z [ x ] das Minimalpolynom von α , d.h. deg P = n und P ( α ) = 0 .
Es gibt (nach dem Mittelwertsatz) ein ξ zwischen α und qp mit
P ( α ) − P ( qp )
= P′(ξ ) .
(7.1)
α − qp
Da α irrational ist, gilt P ( qp ) ≠ 0 (sonst folgt aus P ( qp ) = 0 , dass P ( x ) = ( x − qp ) R ( x ) gilt mit R ( α ) = 0
und deg R < n , was zu einem Widerspruch zum Minimalpolynom führt). Offenbar gilt:
q n P ( qp ) = q n ( an ( qp ) + an −1 ( qp )
n −1
n
+ ... + a 0 ) ∈ Z .
Damit folgt
q n P ( qp ) ≥ 1 , also P ( qp ) ≥
Mit (7.1) und P ( α ) = 0 folgt
P ( α ) − P ( qp ) = α −
und damit
α−
1
.
qn
p
P′(ξ )
q
P ( α ) − P ( qp )
P ( qp )
1
p
=
=
≥ n
q
P′(ξ )
P′(ξ )
q P′(ξ )
Abschätzung von P ′ ( ξ ) liefert zwei Fälle. Da für
α−
p
≥1
q
α−
p
<1
q
der Satz trivialerweise gilt, setzen wir
voraus. Es gilt:
ξ −α ≤ α−
(7.2)
p
< 1.
q
Also folgt
ξ < α +1.
Setze a = max 0≤ j ≤n a j (Koeffizient von P ). Damit hängen ξ und a j nur von α ab. Es gilt:
1
P ′ ( ξ ) = nan ξ n −1 + ... + 2a2ξ + a1 ≤ n ( na ( α + 1 )n −1 ) =:
.
c (α)
Mit (7.2) folgt die Behauptung.
Bemerkung 7.24
Der Beweis zum Satz von Liouville (7.23) ist konstruktiv, d.h. c ( α ) lässt sich explizit bestimmen. Wir tun
dies für ein konkretes α , nämlich α := α0 := 1+2 5 = 1 , um zu zeigen, dass der Satz von Hurwitz (7.16)
bestmöglich ist. Aus
1+ 5
1− 5
P0 ( x ) = x −
x−
= x2 − x − 1
2
2
folgt P0′ ( x ) = 2x − 1 . Sei qp ∈ Q beliebig. Setze δ := α0 − qp . Nach dem Mittelwertsatz existiert ein ξ zwischen α0 und qp mit P0 ( qp ) = δ P0′ ( ξ ) . Wegen
p
ξ = α0 + ( ξ − α0 ) ≤ α0 + ξ − α0 ≤ α0 + − α0 = α0 + δ
q
folgt
P0′ ( ξ ) ≤ 2 ( α0 + δ ) − 1 = 2δ + 5 .
(
)(
Wie im vorigen Beweis gilt:
54
)
Kapitel 7
Diophantische Approximation und Kettenbrüche
p
1
1
1
≥ 2
≥
>
.
( 2δ + 5 )q 2
( 5 + ε )q 2
q
q P0′ ( ξ )
Die letzte Ungleichheit folgt für jedes ε > 0 , sofern q hinreichend groß ist und Nenner eines Näherungsbruchs von α ist, denn nach Korollar 7.14 und für hinreichend großes q gilt:
p
1
ε
δ = α0 − < 2 < .
2
q
q
′
Für beliebige Brüche qp′ ∈ Q wissen wir:
p′
p
α0 −
für q ′ ≤ q
≥ α0 −
q
q′
nach Satz 7.18 (iv).
α0 −
Korollar 7.25 (7.13)
Die Zahl
∞
α = 0.1100010000000000000000010...010... =
∑ 10−k !
k =1
ist transzendent.
Beweis
Setze
d
pd = 10d ! ∑ 10−k ! ∈ Z und qd = 10d ! ∈ Z .
k =1
Offenbar gilt:
α−
pd
=
qd
∞
∑
10−k ! = 10−(d +1 ) ! ( 1 + 10(d +1 )!−(d +2 )! + 10(d +1 )!−(d + 3 ) ! + ... )
k =d +1
≤ 10−(d +1 )! ( 1 + 10−1 + 10−2 + ... ) =
10
⋅ 10−(d +1 )! < 10−(d +1 ) !+1 < 10−d ⋅d ! = ( qd )−d .
9
Also gilt für jedes d :
pd
1
< d .
qd
qd
Damit kann α nicht algebraisch vom Grad n ∈ N sein, denn sonst müsste gelten:
p
c (α)
p
α − ≥ n für alle .
q
q
q
Für p := pd und q := qd folgt
c (α)
1
≤ d
qdn
qd
und somit mit
1
qdd −n ≤
c (α)
der Widerspruch.
α−
Bemerkung 7.26
Der vorige Beweis führt allgemeiner immer dann zur Transzendenz einer gegebenen Zahl α , sofern eine Folge
verschiedener Zahlen qpnn ∈ Q existiert derart, dass
p
1
α − n < ωn mit lim ωn = ∞
n →∞
qn
qn
gilt.
55
56
Kapitel 8
Diophantische Gleichungen
Eine bedeutende quadratische diophantische Gleichung ist die pellsche Gleichung ( P )
x 2 − Dy 2 = 1 mit D ∈ N und D keine Quadratzahl.
Für D < 0 folgt für D ′ := −D aus x 2 + D ′y 2 = 1 : ( x , y ) = ( ±1, 0 ) und ( x , y ) = ( 0, ±1 ) für D ′ = 1 .
Für D = d 2 gilt offenbar
1 = x 2 − d 2y 2 = x 2 − ( dy )2 = ( x − dy )( x + dy ) ,
woraus x − dy = x + dy = ±1 und damit ( x , y ) = ( ±1, 0 ) folgt.
Existieren für ( P ) mit D ∈ N weitere Punkte zu ( x , y ) = ( ±1, 0 ) auf der Hyperbel?
O.B.d.A. seien x , y ∈ N . Das folgende Resultat wurde von Fermat vermutet und von Lagrange bewiesen.
Satz 8.1 (8.1)
Ist D ∈ N mit D ∉ N , so hat ( P ) unendlich viele Lösungen.
Beweis
(Nicht konstruktiv mittels unendlichem Schubfachschluss.)
2
Aus D ∉ N folgt D ∉ Q (sonst gilt D = ab mit (a, b ) = 1 , woraus D = ab2 , Db 2 = a 2 und damit D|a 2
2 2
folgt. Für alle p|D gilt p |a und es folgt: ep ( D ) ist gerade oder p|b und wir erhalten einen Widerspruch.
Es folgt D = m 2 und damit mit D ∈ N der Widerspruch.). Nach Dirichlets Approximationssatz (7.2) existieren unendlich viele x i , yi ∈ N , i = 1, 2,... , mit
x
1
D− i < 2,
yi
yi
was äquivalent zu
xi
1
− D < 2
yi
yi
Kapitel 8
Diophantische Gleichungen
ist. Also erhalten wir
0<
xi
+ D =
yi
xi
(y
)
− D +2 D ≤
i
xi
1
− D +2 D < 2 +2 D.
yi
yi
Es folgt
0 < x i2 − Dyi2 = x i − Dyi ⋅ x i + Dyi = yi2
2
xi
x
1⎛1
⎞
− D ⋅ i + D ≤ yi2 2 ⎜⎜ 2 + 2 D ⎟⎟ ≤ 2 D + 1 .
yi
yi
⎝
⎠
yi yi
Die erste Ungleichheit gilt, da sonst D = xyi2 ∈ Q gelten würde. Nun gilt:
i
− ( 2 D + 1 ) ≤ x i2 − Dyi2 ≤ 2 D + 1 für alle x i , yi , i = 1,2,... .
≠0
Der unendliche Schubfachschluss besagt: Werden unendlich viele Objekte auf endlich viele Schubladen verteilt, so befinden sich in mindestens einer Schublade unendlich viele Objekte.
Aus dem unendlichen Schubfachschluss folgt, dass es ein ganzzahliges k ∈ [ −2 D − 1, 2 D + 1 ] mit k ≠ 0
gibt, d.h. 0 < k ≤ 2 D + 1 , so dass
x i2 − Dyi2 = k
für unendlich viele i ∈ N gilt.
Eine weitere Anwendung des unendlichen Schubfachschlusses liefert: Für unendlich viele i ∈ N gilt:
x i2 − Dyi2 = k und x i ≡ x j mod k , yi ≡ y j mod k .
Insbesondere gibt es zwei solcher Indizes, d.h. es gibt ( x1, y1 ) ≠ ( x 2 , y2 ) mit
x i2 − Dyi2 = k , i = 1, 2 und x1 ≡ x 2 mod k , y1 ≡ y2 mod k .
Es folgt x1 ≠ x 2 und y1 ≠ y2 , sonst folgt aus x1 = x 2
x12 − Dy12 = k = x 22 − Dy22 = x12 − Dy22
und damit y12 = y22 , woraus mit y1 = y2 der Widerspruch folgt. Damit erhalten wir
x1x 2 − y1y2D ≡ x12 − Dy12 ≡ 0 mod k ,
woraus
x1x 2 − y1y2D = ku
für ein passendes u ∈ Z und
x1y2 − x 2y1 ≡ x1y1 − x1y1 = 0 mod k
folgt, woraus
x1y2 − x 2y1 = kv
für ein passendes v ∈ Z folgt. Nun folgt
( x1 − Dy1 )( x 2 + Dy2 ) = x1x 2 − y1y2D + ( x1y2 − x 2y1 ) D = ku + kv D = k ( u + v D )
und
( x1 + Dy1 )( x 2 − Dy2 ) = x1x 2 − y1y2D − ( x1y2 − x 2y1 ) D = ku − kv D = k ( u − v D ) .
Weiter folgt aus
k 2 = ( x12 − Dy12 )( x 22 − Dy22 ) = k ( u + v D ) k ( u − v D ) = k 2 ( u 2 − Dv 2 ) ,
dass u 2 − v 2D = 1 gilt, d.h. u, v ist eine Lösung von ( P ) .
Behauptung: v ≠ 0 . Denn: Wäre v = 0 , dann wäre u = ±1 und es würde folgen:
( x1 − y1 D ) k = ( x1 − y1 D )( x 22 − y22D ) = ( x1 − y1 D )( x 2 + y2 D )( x 2 − y2 D ) = ku ( x 2 − y2 D )
= ±k ( x 2 − y2 D ).
Also würden wir x1 − y1 D = ± ( x 2 − y2 D ) erhalten und wegen D ∉ Q und x1, x 2, y1, y2 ∈ N würde mit
x1 = x 2 und y1 = y2 ein Widerspruch folgen.
Also ist u, v eine nichttriviale Lösung von ( P ) . O.B.d.A. seien u, v ∈ N . Wir ordnen jeder Lösung x , y ∈ N
von ( P ) die Irrationalzahl x + y D zu. Diese Zahl heißt die pellsche Einheit. Wir definieren zu gegebenen
x1 := u und y1 := v die Folgen ( x n )n ∈N und ( yn )n ∈N durch:
n
x n + yn D := ( x1 + y1 D ) .
Dann sind x n , yn Lösungen von ( P ) für alle n ∈ N , denn nach dem binomischen Lehrsatz gilt:
n
n
n
n
n
n
( x1 ± y1 D )n = ∑
( ±y1 D )i x1n −i = ∑
( y1 D )i x1n −i ± ∑
( y1 D )i x1n −i =: x n ± yn D .
i
i
i
i =0
i =0
i =0
( )
( )
( )
2|i
2?i
=:x n ∈Z
Also folgt:
57
=:yn D ∈Z D
Kapitel 8
Diophantische Gleichungen
n
n
n
1 = ( x12 − Dy12 ) = ( x 1 − y1 D ) ( x 1 + y1 D ) = ( x n − yn D )( x n + yn D ) = x n2 − Dyn2 .
Dies sind unendlich viele verschiedene Lösungen, denn:
( x1 + y1 D )n → ∞ für n → ∞ .
Das kleinste Paar ( x1, y1 ) ≠ ( 1, 0 ) , das ( P ) löst, heißt Grundlösung, die zugehörige pellsche Einheit x1 + y1 D
heißt pellsche Grundeinheit.
Satz 8.2 (8.2)
Die pellschen Einheiten sind genau die Potenzen der Grundeinheit. Mit anderen Worten:
n
Alle Lösungen von ( P ) ergeben sich als ±x n , ±yn und ±x n , ∓yn mit x n + yn D := ( x1 + y1 D ) für n ≥ 1
zusammen mit der trivialen Lösung x 0 = ±1 , y 0 = 0 .
Beweis
Sei x + y D eine beliebige pellsche Einheit, die keine Potenz von x1 + y1 D ist. Also existiert ein n ∈ N
mit
( x1 + y1 D )n < x + y D < ( x1 + y1 D )n +1 .
n
Hieraus folgt durch Multiplikation mit ( x1 − y1 D )
n
n
1 = ( x 12 − y12D ) < ( x + y D ) ( x 1 − y1 D ) < x 1 + y1 D .
n
n
Wir setzen a + b D = ( x1 − y1 D ) und es folgt ( x1 + y1 D ) = a − b D . Weiter gilt:
n
c + d D := ( x + y D )( x 1 − y1 D ) = ax + byD + ( ay + bx ) D .
Dann folgt:
n
c − d D = ( x − y D ) ( x1 + y1 D ) .
Insgesamt folgt:
( c + d D )( c − d D ) = c 2 − Dd 2 = 1 ⋅ 1n = 1 .
Also ist c, d Lösung von ( P ) . Das widerspricht aber der Minimalität der Grundlösung.
Bemerkung 8.3
(i) Der Beweis von Satz 8.1 impliziert, dass als Lösungen der pellschen Gleichung x 2 − Dy 2 = 1 die Näherungsbrüche qpnn von D in der Form x = pn , y = qn in Frage kommen. In der Tat ergibt eine genauere Analyse des Kettenbruchs von D , dass dieser immer die Gestalt
D = a 0 ; a1,..., al
besitzt. Die Periodenlänge l erweist sich als markante Größe, denn als Grundlösung stellt sich gerade
das Paar x1 = pl −1, y1 = ql −1 mit 2|l bzw. x1 = p2l −1, y1 = q2l −1 mit 2 ? l heraus. Alle Lösungen sind
x n = pnl −1, yn = qnl −1 mit n ∈ N für 2|l oder n ∈ 2N für 2 ? l .
(ii) Nahe verwandt mit der pellschen Gleichung ist
x 2 − Dy 2 = −1 .
Wie in (i) hängt das Lösungsverhalten eng mit der Kettenbruchentwicklung von D zusammen: Bei
gerader Periodenlänge l existiert überhaupt keine Lösung, bei ungeradem l sind die positiven Lösungen
x n = pnl −1, yn = qnl −1 mit n = 1, 3, 5,... .
Die vielleicht berühmteste diophantische Gleichung ist die fermatsche Gleichung
x n + y n = z n für n ∈ N , n ≥ 2 .
Der Spezialfall n = 2 geht auf die Griechen der Antike zurück, die entsprechenden Lösungen x , y, z ∈ N
heißen pythagoräische Tripel. Wegen der Homogenität der fermatschen Gleichung können wir o.B.d.A. stets
( x , y, z ) = 1 voraussetzen.
Satz 8.4 (8.3)
Die Lösungen von x 2 + y 2 = z 2 mit ( x , y, z ) = 1 sind genau gegeben durch
x = a 2 − b 2 , y = 2ab , z = a 2 + b 2
mit a, b ∈ N , (a, b ) = 1 , 2|ab .
58
Kapitel 8
Diophantische Gleichungen
Beweis
Offenbar sind die gegebenen Tripel Lösungen, denn
x 2 + y 2 = ( a 4 − 2a 2b 2 + b 4 ) + 4a 2b 2 = a 4 + 2a 2b 2 + b 4 = z 2 .
Sei nun umgekehrt x , y, z ein Pythagoräisches Tripel mit ( x , y, z ) = 1 . O.B.d.A. sei y gerade, denn für
x ≡ y ≡ 1 mod 2 wäre x 2 + y 2 ≡ 2 mod 4 , aber z 2 ≡ 0,1 mod 4 . Es folgt 2 ? x und 2 ? z . Wir schreiben die
Ausgangsgleichung in der Form
y 2 = z 2 − x 2 = ( z + x )( z − x ) .
Wegen ( z + x , z − x ) = ( z + x , 2z ) = 2 erhalten wir
z + x = 2a 2 , z − x = 2b 2 , y = 2ab
für gewisse a, b mit (a, b ) = 1 . Wegen z ≡ 1, 3 mod 4 und x ≡ 1, 3 mod 4 folgt
z + x ≡ 2 mod 4 und z − x ≡ 0 mod 4 oder z + x ≡ 0 mod 4 und z − x ≡ 2 mod 4 ,
also ist eine der Zahlen a, b gerade, d.h. 2|ab .
Bekanntlich besagt die fermatsche Vermutung, die kürzlich von Wiles und Taylor in erheblich größerer Allgemeinheit bewiesen wurde, dass die fermatsche Gleichung für n ≥ 3 keine Lösung x , y, z ∈ N besitzt. Der
einfachste Fall ist n = 4 , der bereits von Fermat selbst geklärt wurde.
Satz 8.5 (8.4)
Die Gleichung x 4 + y 4 = z 2 besitzt keine Lösung x , y, z ∈ N .
Beweis (Fermatscher Abstieg)
Wir nehmen an, es gäbe eine Lösung x , y, z ∈ N , wobei z minimal sei. Dann gilt selbstverständlich ( x , y, z ) = 1 .
Außerdem ist – wie bei x 2 + y 2 = z 2 – eine der Zahlen x , y gerade, o.B.d.A. gelte 2|y . Damit ist x 2, y 2, z
ein Pythagoräisches Tripel, d.h. nach Satz 8.4 existieren a, b ∈ N mit
x 2 = a 2 − b 2 , y 2 = 2ab , z = a 2 + b 2
und ( a, b ) = 1 , 2|ab . Würde 2|a und 2 ? b gelten, so hätten wir x 2 ≡ 4 − 1 = 3 mod 4 , was unmöglich ist.
Also gilt 2|b und 2 ? a . Mit x 2 + b 2 = a 2 ist weiterhin auch x , b, a ein Pythagoräisches Tripel. Daher existieren nach Satz 8.4 Zahlen c, d ∈ N , (c, d ) = 1 , mit
x = c 2 − d 2 , b = 2cd , a = c 2 + d 2 .
Es folgt
y 2 = 2ab = 4cd ( c 2 + d 2 ) .
Wegen (c, d ) = 1 ist ( c, c 2 + d 2 ) = ( d, c 2 + d 2 ) = 1 , d.h.
c = e2 , d = f 2 , c2 + d 2 = g 2
für gewisse e, f , g ∈ N . Damit gilt
e4 + f 4 = g2 ,
wir haben also eine weitere Lösung unserer Ausgangsgleichung konstruiert. Dabei gilt aber
g ≤ g 2 = c2 + d 2 = a ≤ a 2 < a 2 + b2 = z ,
im Widerspruch zur Annahme der Minimalität von z .
Korollar 8.6 (8.5)
Die Gleichung x 4 + y 4 = z 4 besitzt keine Lösung x , y, z ∈ N .
Wir wollen noch ein anderes Beispiel einer Gleichung vierten Grades angeben, die ebenfalls auf Fermat zurückgeht.
Satz 8.7 (8.6)
Die Gleichung
x 4 − x 2y 2 + y 4 = z 2 , ( x , y ) = 1 ,
hat nur die Lösungen x 2 = 1 , y = 0 und x = 0 , y 2 = 1 und x 2 = y 2 = 1 .
Beweis
O.B.d.A. seien x , y, z ∈ N . Wir schreiben die Gleichung als
( x 2 − y 2 )2 + ( xy )2 = z 2 .
1. Fall: 2|xy . Es folgt 2 ? x 2 − y 2 , also nach Satz 8.4
59
Kapitel 8
Diophantische Gleichungen
x 2 − y 2 = a 2 − b 2 , xy = 2ab
für gewisse a, b mit (a, b ) = 1 . Wir setzen d1 = ( x , b ) , d2 = ( y, a ) und x =: d1X , b =: d1B , y =: d2Y ,
a =: d2A , also XY = 2AB . Selbstverständlich ist ( X , B ) = (Y , A ) = 1 und daher X |2A , A | X und Y |2B ,
B |Y . Es folgt X = 2A , Y = B oder X = A , Y = 2B .
1a. Fall: X = 2A , Y = B . Es ergibt sich
x = 2Ad1 , b = d1B , y = d2B , a = d2A .
Einsetzen in x 2 − y 2 = a 2 − b 2 liefert
4A2d12 − d22B 2 = d22A2 − d12B 2 ,
also
d12 ( 4A2 + B 2 ) = d22 ( A2 + B 2 ) .
Es ist ( A + B , 4A + B ) = ( A + B 2 , 3A2 ) = ( A2 + B 2 , A2 ) , denn wegen (a, b ) = 1 ist auch ( A, B ) = 1
und somit A2 + B 2 ≡ 0 mod 3 . Also haben wir
( A2 + B 2, A2 ) = ( B 2, A2 ) = 1 .
2
2
2
2
2
Damit folgt für gewisse C und D
A2 + B 2 = C 2 und 4A2 + B 2 = D 2 .
O.B.d.A. sei B ungerade, denn für 2|B ergäbe sich ein entsprechendes Paar von Gleichungen mit vertauschten Variablen. Die zweite der Gleichungen ( 2A )2 + B 2 = D 2 hat nach Satz 8.4 eine Lösung mit B = p 2 − q 2
und A = pq für gewisse ( p, q ) = 1 mit 2|pq . Einsetzen in die erste Gleichung liefert
p 4 − p 2q 2 + q 4 = C 2 .
Wegen pq = A ≤ a ≤ xy2 lässt sich die Methode des fermatschen Abstiegs anwenden, zumal p 2 = q 2 = 1
wegen B ≠ 0 auch nicht eintreten kann. Die Ausgangsgleichung ist in diesem Fall unlösbar.
1b. Fall: X = A , Y = 2B . Es folgt analog zu 1a. Fall
d12 ( A2 + B 2 ) = d22 ( A2 + 4B 2 )
und die obige Argumentation lässt erneut einen fermatschen Abstieg zu und auch hier existiert keine Lösung.
2. Fall: 2 ? xy .
Mit Satz 8.4 folgt sofort
xy = a 2 − b 2 , x 2 − y 2 = 2ab
für gewisse (a, b ) = 1 und es gilt 2|ab . Wir haben daher
(x
2
) (
)
2
2
− y2
x 2 + y2
=
.
2
2
Nach 1. Fall lässt dies nur die triviale Lösung ab = 0 zu, d.h. wegen ( x , y ) = 1 bleibt nur x 2 = y 2 = 1 .
a 4 − a 2b 2 + b 4 = ( xy )2 +
Korollar 8.8 (8.7)
Es gibt keine vier Quadrate in arithmetischer Progression, d.h. es gibt kein c ∈ N für Quadrate x12 , x 22 , x 32 , x 42
mit x i2+1 − x i2 = c , i = 1, 2, 3 .
Beweis
Wir nehmen an, es gäbe eine Konstante c ∈ N mit x i2+1 − x i2 = c für i = 1, 2, 3 . Es folgt
x12 + x 32 = ( x12 + c ) + ( x 32 − c ) = 2x 22 und x 22 + x 42 = 2x 32 .
Also ergibt sich
x12 ( 2x 32 − x 22 ) = x12x 42 = x 42 ( 2x 22 − x 32 )
und daraus
2 ( x12x 32 − x 22x 42 ) = x12x 22 − x 32x 42 .
1
1
2 ( x 1x 2 + x 3x 4 ) , d = 2 ( x 1x 2 − x 3x 4 )
2
2
Wir setzen a = x1x 3 , b = x 2x 4 , c =
x 2 ≡ x 4 mod 2 , also sind c, d ganz. Damit kommt neben ab = c − d
. Offenbar gilt x1 ≡ x 3 mod 2 und
aus obiger Identität
a 2 − b 2 = 2cd
und daraus
2
2
a 4 − a 2b 2 + b 4 = 4c 2d 2 + ( c 2 − d 2 ) = ( c 2 + d 2 ) .
Nach Satz 8.7 bleiben nur a = 0 oder b = 0 (d.h. eines der x i ist gleich 0), oder a 2 = b 2 , d.h. ( x1x 3 )2 = ( x 2x 4 )2 .
Beides ist offensichtlich unmöglich.
60
Kapitel 8
Diophantische Gleichungen
Bemerkung 8.9
Drei Quadrate in arithmetischer Progression existieren, z.B. 12 = 1 , 52 = 25 und 72 = 49 und daraus allgemein
x1 = k , x 2 = 5k , x 3 = 7k für k ∈ N ,
denn x 22 − x12 = 24k 2 = x 32 − x 22 .
Wir kommen nun auf eine diophantische Gleichung zu sprechen, die – anders als die bisher behandelten –
nicht polynomial ist, nämlich die so genannte catalansche Gleichung
x n − ym = 1 .
Hierbei sind x , y, n, m als Variablen aufzufassen und wir sprechen deshalb von einer diophantischen Exponential-Gleichung. Catalan vermutete 1844, dass für x , y, n, m ∈ N>1 die einzige Lösung gegeben ist durch x = 3 ,
y = 2 , n = 2 , m = 3 . Nach Tijdeman (1976) ist bekannt, dass die catalansche Gleichung nur endlich viele
Lösungen besitzt und diese lassen sich effektiv beschränken. 2002 konnte die catalansche Vermutung durch
den rumänischen Mathematiker Preda Mihailescu (1955 geboren) bewiesen werden. Wir behandeln ganz elementar einen Spezialfall.
Satz 8.10 (8.8)
Die Gleichung 3n − 2m = 1 hat über N nur die Lösungen n = m = 1 und n = 2 , m = 3 .
Beweis
1. Fall: n = 2k + 1 , k ∈ N 0 . Es ist 3n ≡ 32k +1 ≡ 9k ⋅ 3 ≡ 1k ⋅ 3 ≡ 3 mod 4 , also haben wir 3n − 1 ≡ 2 mod 4 .
Für m ≥ 2 gilt 4|2m , also ist 3n − 1 = 2m nur für m = 1 , daher n = 1 möglich.
2. Fall: n = 2k , k ∈ N . Wir haben 3n − 1 = ( 3k − 1 )( 3k + 1 ) = 2m , also 3k − 1 = 2s und 3k + 1 = 2t für
gewisse s, t ∈ N 0 . Es folgt 2s + 1 = 3k = 2t − 1 , d.h. 2t − 2s = 2 , mit anderen Worten: t = 2 , s = 1 , also
k = 1 . Es ergibt sich n = 2k = 2 und daraus m = 3
61
62
Kapitel 9
p-adische Zahlen
Der gewöhnliche Absolutbetrag x für x ∈ Q ist eine Norm auf Q und induziert die Metrik d ( x , y ) := x − y
für x , y ∈ Q . Wir werden andere Normen und Metriken auf Q kennen lernen.
Für p ∈ P und x = ± ab ∈ Q definieren wir (in Erweiterung der Ordnung ep ( n ) für n ∈ N )
für x ≠ 0,
⎧
⎪ep ( a ) − ep (b ) ,
ep ( x ) = ⎪
⎨ 0,
für x = 0.
⎪
⎪
⎩
Dabei hängt ep ( x ) nicht von der speziellen Bruchdarstellung von x ab, denn
ac
a
ep
= ep ( ac ) − ep (bc ) = ep ( a ) + ep ( c ) − ep (b ) − ep ( c ) = ep
.
bc
b
( )
( )
Lemma 9.1 (9.1)
Sei p ∈ P . Die Abbildung ⋅ p : Q → R≥0 , definiert durch
⎧⎪ p −ep x , für x ≠ 0,
x p = ⎪⎨
für x = 0,
⎪⎪⎩ 0,
ist eine Norm auf Q , d.h. es gilt
(i) x p = 0 ⇐⇒ x = 0 ,
(ii) x ⋅ y p = x p ⋅ y p ,
(iii) x + y p ≤ x p + y p
für alle x , y ∈ Q . Anstelle der Dreiecksungleichung (iii) gilt sogar die schärfere Ungleichung
x + y p ≤ max { x p , y p } ,
)
(
wobei < nur für x p = y p auftreten kann.
Beweis
(i) ist klar. Für xy = 0 ist auch (ii) klar. Für xy ≠ 0 haben wir
xy p = p−ep ( xy ) = p−ep x ⋅ p −ep ( y ) = x
(
)
p
⋅ yp.
Für x = 0 und y = 0 oder x + y = 0 ist die verschärfte Dreiecksungleichung klar. Sei also xy ≠ 0 und
x + y ≠ 0 . Mit gekürzten Brüchen x = ab , y = dc haben wir
ad + bc
ep ( x + y ) = ep
= ep (ad + bc ) − ep (b ) − ep ( d )
bd
≥ min {ep ( ad ) , ep (bc ) } − ep (b ) − ep ( d )
= min {ep (a ) + ep ( d ), ep (b ) + ep ( c ) } − ep (b ) − ep ( d )
= min {ep (a ) − ep (b ), ep (c ) − ep (d ) } = min {ep ( x ) , ep ( y )} .
(
)
Also gilt
x +y
p
= p −ep ( x +y ) ≤ max { p −ep
(
, p −ep ( y ) } = max { x p , y p } .
x)
Definition 9.2
Eine Norm heißt nicht-archimedisch, falls stets x + y ≤ max { x , y } gilt. Eine Metrik heißt nicht-archimedisch, falls d ( x , y ) ≤ max {d ( x , z ), d ( z, y ) } für alle x , y, z gilt. Gewöhnliche Normen und Metriken heißen
archimedisch.
Bemerkung 9.3
Nach Lemma 9.1 ist jede ⋅ p eine nicht-archimedische Norm auf Q . Nicht-archimedische Normen induzieren
nicht-archimedische Metriken, denn
d ( x , y ) := x − y = ( x − z ) + ( z − y ) ≤ max { x − z , z − y } = max {d ( x , z ), d ( z, y ) } .
Der gewöhnliche Absolutbetrag ist eine archimedische Norm auf Q , z.B.
p -adische Zahlen
Kapitel 9
1 + 1 > max { 1 , 1 } .
Bezüglich des gewöhnlichen Absolutbetrags bzw. der davon induzierten Metrik ist Q nicht vollständig, d.h. es
gibt rationale Cauchy-Folgen, die in Q keinen Grenzwert besitzen. Durch Vervollständigung entsteht R . Das
folgende Beispiel zeigt, dass Q auch bezüglich ⋅ p (jedenfalls für ein hinreichend großes p ) nicht vollständig ist.
Beispiel 9.4 (9.2)
Sei p ≠ 2 und r quadratischer Rest mod p . Wir definieren rekursiv eine Folge ( an )n ∈N über Z mit der
Bedingung
an2 ≡ r mod pn für n ∈ N .
Dies ist möglich (und nicht eindeutig), denn a12 ≡ r mod p ist lösbar, wobei p ? a1 . Wir setzen a2 = a1 + kp
mit k derart, dass gilt
a22 = ( a1 + kp )2 ≡ a12 + 2a1k ≡ r mod p 2 (lösbar wegen p = ( 2a1p, p 2 )|( r − a12 ) ).
Allgemein setzen wir an +1 := an + kn ⋅ pn mit an2 +1 ≡ r mod pn +1 . Die Folge ( an )n ∈N ist Cauchy-Folge bezüglich ⋅ p , denn für n > m gilt
an − am = ( an − an −1 ) + ( an −1 − an −2 ) + ... + ( am +1 − am )
= kn −1pn −1 + kn −2 pn −2 + ... + km pm ,
also an − am ≤ p−m .
Angenommen, ( an )n ∈N besitzt den Grenzwert a ∈ Q . Wir nehmen zunächst außerdem an, dass a 2 ≠ r . Dann
sei a 2 − r p =: p−e0 . Wegen limn →∞ an = a ist an − a p < p−(e0 +1) für alle großen n ∈ N . Mit a = dc , wobei (c, d ) = 1 , ist
(a d − c )
an − a = n
,
d
wobei p ? d gilt (sonst gilt p |c wegen pe0 +1 |( an − a ) ). Es folgt
a d +c
an + a p = n
≤ 1.
d
Also
an2 − a 2 p = an − a p ⋅ an + a p < p−(e0 +1) .
Wir erhalten
a2 − r
p
= ( a 2 − an2 ) + ( an2 − r ) p ≤ max { a 2 − an2
p
, an2 − r
p
}
< max { p
,p } = p
für n ≥ e0 + 1 . Dies führt zu einem Widerspruch.
1
Es bleibt nur die Möglichkeit r = a 2 , d.h. r ist Quadratzahl in Z . Zwischen 0 und p gibt es p −
2 quadrati1
sche Reste, aber nur [ p ] Quadratzahlen. Wegen p < p −
für p ≥ 7 können wir also einen quadratischen
2
Rest r mod p wählen, der keine Quadratzahl ist. Die damit konstruierte Folge ( an )n ∈N ist eine CauchyFolge ohne Grenzwert in Q .
−(e0 +1 )
−n
−(e0 +1 )
Vervollständigung von Q bezüglich ⋅ p
Wir nennen zwei Cauchy-Folgen ( an )n ∈N , (bn )n ∈N bezüglich ⋅ p äquivalent, falls an − bn p → 0 für
n → ∞ gilt. Wir definieren Qp als die Menge der Äquivalenzklassen von Cauchy-Folgen über Q . Für
x ∈ Q bezeichne ( x ) die konstante Cauchy-Folge ( x , x , x ,... ) . Offenbar gilt:
(x ) ∼ (x ′ ) ⇐
⇒ x = x′ .
Die Norm eines Elements a ∈ Qp (d.h. einer Äquivalenzklasse) sei erklärt durch
a p := lim an p ,
n →∞
wobei ( an )n ∈N ein beliebiger Repräsentant von a sei (wir schreiben: a = ( an ) ).
Lemma 9.5 (9.3)
Für a ∈ Qp ist a p existent und wohl definiert.
Beweis
Existenz: Falls a ∼ ( 0 ) , so gilt nach Definition der Äquivalenz an p = an − 0 p → 0 . Sei also a X ( 0 ) .
Dann gilt
∃ ε > 0 : ∀ N ∈ N : ∃ n 0 > N : an 0 p > ε .
63
p -adische Zahlen
Kapitel 9
Mit anderen Worten: Es gibt ein ε > 0 , so dass für alle N ∈ N ein n 0 > N existiert mit an0 p > ε . Wir
wählen N so groß, dass an − an ′ p < ε für alle n, n ′ > N . Insbesondere haben wir an − an0 p < ε für alle
n, n ′ ∈ N . Außerdem gilt
an − an0 p ≤ max { an p , an0 p } ,
wobei an0
p
> ε . Nach Lemma 9.1 bleibt nur die Möglichkeit an
lim an p = an0 p .
p
= an 0
p
für alle n > N , also
n →∞
Wohldefiniertheit: Für a ∼ b ∼ ( 0 ) folgt wieder aus der Definition der Äquivalenz an p → 0 und bn p → 0 .
Für a ∼ b X ( 0 ) erhalten wir aufgrund des Arguments im ersten Teil des Beweises, dass a p = p α und
b p = p β für gewisse α, β ∈ Z . Die Annahme α ≠ β stände im Widerspruch zu an − bn p → 0 .
Bemerkung 9.6
Das vorstehende Lemma 9.5 zeigt, dass gilt
{ a p | a ∈ Qp } = { r
p
| r ∈ Q } = { 0 } ∪ { pt | t ∈ Z } ,
d.h. der Wertebereich der Norm bleibt bei der Vervollständigung von Q nach Q p unverändert. Im Gegensatz dazu erweitert sich beim Übergang von Q zu R der Normwertebereich von Q≥0 zu Q≥0 .
Satz 9.7 (9.4)
Jedes Q p ist ein vollständiger Körper bezüglich ⋅ p mit Unterkörper Q (also char Qp = 0 ).
Beweis
Seien a = ( an ) = ( an′ ) , b = (bn ) = (bn′ ) ∈ Qp . Wir setzen
a + b := ( an + bn ) , a ⋅ b := ( an ⋅ bn ) .
Die Wohldefiniertheit folgt aus
( an + bn ) − ( an′ + bn′ ) p = ( an − an′ ) + (bn − bn′ ) p ≤ max { an − an′
p
, bn − bn′
p
}→ 0
bzw.
anbn − an′ bn′
p
= an (bn − bn′ ) + bn′ ( an − an′ ) p ≤ max { an
p
⋅ bn − bn′
p
, bn′
p
⋅ an − an′
p
}→ 0.
Additive Inverse werden in offensichtlicher Weise erklärt. Bei multiplikativen Inversen berücksichtigen wir,
dass jede Cauchy-Folge ( an ) äquivalent ist zu ( an ) mit
für an ≠ 0,
⎧⎪an ,
an = ⎪⎨ n
p
,
für an = 0.
⎪⎪⎩
Damit ist auch ( an )−1 := a1n eine Cauchy-Folge außer für an p → 0 , d.h. ( an ) = 0 . Die Rechengesetze für
Körper folgen für Q p direkt aus denen für Q . Wegen Q ≅ {( r ) | r ∈ Q } ⊆ Qp haben wir gezeigt, dass Qp
ein Oberkörper von Q ist. Zu zeigen bleibt die Vollständigkeit von Q p . Sei also ( αn )n ∈N eine Cauchy-Folge
über Qp , d.h. für ε > 0 gilt αi − α j p < ε , sofern i, j > N ( ε ) .
Wir zeigen zunächst, dass zu jedem αi ein ai ∈ Q existiert mit αi − ai p < 2i : Sei etwa αi = ( αij )j ∈N .
Dann gilt αij − αij ′ p < 2−i für hinreichend große j, j ′ . Mit ai := αij ′ haben wir gemäß Definition der Form
auf Q p
αi − ai p = lim αij − αij ′ p < 2−i .
j →∞
Die Folge ( an )n ∈N ist eine Cauchy-Folge (über Q ), denn für hinreichend große i, j gilt
ai − a j p = ( ai − αi ) + ( αi − α j ) + ( α j − a j ) p ≤ ai − αi p + αi − α j p + α j − a j
< 2−i + ε + 2− j < 2ε.
Wir setzen α = ( an )n ∈N und haben für ein hinreichend großes i
αi − α p = lim αi − a j p ≤ lim { αi − ai p + ai − a j
j →∞
j →∞
d.h. ( αn )n ∈N konvergiert gegen α .
p
p
−i
} ≤ 2 + 2ε < 3ε ,
Bemerkung 9.8
Der oben durchgeführte Vervollständigungsprozess kann ganz allgemein benutzt werden bei beliebigen Körpern, auf denen eine Norm erklärt ist.
Satz 9.9 (9.5)
Für p1 ≠ p2 sind die Körper Qp1 und Qp2 nicht isomorph.
64
p -adische Zahlen
Kapitel 9
Beweis
Es gibt eine natürliche Zahl r , die quadratischer Rest mod p1 und quadratischer Nichtrest mod p2 ist, denn:
Seien a 0 quadratischer Rest mod p1 und b0 quadratischer Nichtrest mod p2 . Damit ist a 0 + p1s für s ∈ Z
quadratischer Rest mod p1 und b0 + p2t für t ∈ Z quadratischer Nichtrest mod p2 . Die diophantische Gleichung
p1s − p2t = b0 − a 0
ist wegen ( p1, p2 ) = 1 lösbar und liefert das gewünschte r .
In Beispiel 9.4 hatten wir gezeigt, dass für den quadratischen Rest r mod p1 , die dort konstruierte CauchyFolge den Grenzwert a mit a 2 = r besitzt. Wegen der Vollständigkeit von Qp1 ist a ∈ Qp1 , d.h. x 2 = r ist
lösbar über Qp1 . Da r quadratischer Nichtrest mod p2 ist, kann x 2 = r über Qp2 nicht lösbar sein, da
nicht einmal x 2 ≡ r mod p2 lösbar ist. Daher können Qp1 und Qp2 nicht isomorph sein.
Bemerkung 9.10
Nach einem Satz von Ostrowski sind der gewöhnliche Absolutbetrag ⋅ und die p -adischen Beträge ⋅ p für
p ∈ P „bis auf Äquivalenz“ alle möglichen Normen auf Q . Da äquivalente Normen zu isomorphen Vervollständigungen führen, sind somit R und die p -adischen Körper Qp für p ∈ P bis auf Isomorphie alle möglichen Vervollständigungskörper von Q .
Die reellen Zahlen lassen sich (statt als Äquivalenzklassen von Cauchy-Folgen) konkret als Dezimalzahlen
schreiben. Eine ähnlich konkrete Darstellung existiert auch für p -adische Zahlen.
Wir definieren den Ring der ganzen p -adischen Zahlen
Z p := { x ∈ Qp | x p ≤ 1 } .
Offenbar gilt Z ⊆ Z p . Es gilt sogar
{ ab | (a,b ) = 1, p ? b } ⊆ Z
p
.
Damit lässt sich Q p auch auffassen als Quotientenkörper von Z p (so wie Q ) als Quotientenkörper von Z .
Lemma 9.11 (9.6)
Z liegt dicht in Z p .
Beweis
Sei α ∈ Z p . Gemäß Beweis zu Satz 9.7 existiert zu jedem i ∈ N ein ri ∈ Q mit α − ri p ≤ p−i . Wegen
α p ≤ 1 folgt ri p ≤ 1 (sonst α − ri p = max { α p , ri p } > 1 und damit Widerspruch). Also ist ri = abii für
gewisse ( ai , bi ) = 1 mit p ? bi . Gesucht ist ein ni ∈ Z mit ri − ni p ≤ p−i . Wir haben
1
a
ni − i ≤ p−i ⇐⇒ bi ni − ai p = bi ni − ai p
≤ p−i , da p ? bi ⇐⇒ bi ni ≡ ai mod pi .
bi p
bi p
Wegen (bi , pi ) = 1 ist die Kongruenz lösbar und es folgt
α − ni p = ( α − ri ) + ( ri − ni ) p ≤ α − ri
p
+ ri − ni
p
≤ 2p−i .
Satz 9.12 (9.7)
Sei α ∈ Z p . Dann existiert eine eindeutige Cauchy-Folge ( an )n ∈N mit ( an )n ∈N = α derart, dass
an ∈ Z , 0 ≤ an < p n , an +1 ≡ an mod pn für n ∈ N .
Beweis
Nach Lemma 9.11 existiert ein an ∈ Z mit α − an p ≤ p−n . Wegen
α − ( an + tpn ) p ≤ max { α − an p , t ⋅ pn p } ≤ max { p−n , p−n } = p−n
können wir o.B.d.A. 0 ≤ an < pn voraussetzen. Aus
an +1 − an p ≤ max { an +1 − α p , an − α p } ≤ p−n
folgt an +1 ≡ an mod pn . Insbesondere ist ( an )n ∈N eine Cauchy-Folge mit ( an )n ∈N = α . Zum Beweis der
Eindeutigkeit sei eine Folge (am′ )m ∈N mit denselben Eigenschaften gegeben. Wir haben
am′ − α p ≤ p−n für m ≥ m0 ( n ) ≥ n .
Nach Voraussetzung gilt
am′ ≡ am′ −1 ≡ ... ≡ an′ mod pn ,
also
am′ − an′
p
≤ max { am′ − am′ −1 p ,..., an′ +1 − an′
65
p
} ≤ p−n .
p -adische Zahlen
Kapitel 9
Mit an − α
p
≤ p−n folgt
an′ − an
p
≤ max { an′ − am′
p
, am′ − α p , α − an
p
} ≤ p−n ,
d.h. an ≡ an′ mod pn . Wegen 0 ≤ an , an′ < pn folgt an = an′ .
Satz 9.13 (9.8)
Für p ∈ P gilt
{bp
}
b−k +1
b
+ ... + −1 + b0 + b1p + b2 p 2 + ... | k ∈ Z, b−k ≠ 0, 0 ≤ bi < p
k −1
p
p
(d.h. jede p -adische Zahl wird durch eine Laurent-Reihe dargestellt), wobei ganz kanonisch mit Überträgen
gerechnet wird.
Beweis
Sei zunächst α ∈ Z p und sei ( an )n ∈N die eindeutige Folge aus Satz 9.12. Wir setzen
a1 =: b0 , a2 = a1 + pb1 , a 3 = a2 + p 2b2 , …,
Qp ≅
−k
k
+
also 0 ≤ bn < p für alle n ≥ 0 . Einsetzen liefert
a1 = b0 , a2 = b0 + b1p , a 3 = b0 + b1p + b2 p 2 , …,
also
α = lim an = b0 + b1p + b2 p 2 + ...
n →∞
Sei nun α ∈ Qp , α ≠ 0 . Dann existiert ein k ∈ Z mit α p = pk . Es folgt
α ⋅ p k p = α p ⋅ p k p = p k ⋅ p −k = 1 ,
d.h. α ⋅ pk ∈ Z p und
α ⋅ pk = b−k + b−k +1p + b−k +2 p 2 + ... ,
wobei b−k ≠ 0 (sonst α ⋅ p k < 1 ). Es ergibt sich
b
b
α = −kk + −kk−+11 + ... + b0 + b1p + ...
p
p
Die Richtigkeit des Rechnens mit Überträgen ergibt sich leicht, indem zuerst Teilsummen dieser unendlichen
Reihen verwendet werden und anschließend Grenzwerte bezüglich ⋅ p gebildet werden.
Beispiel 9.14 (vergleiche Beispiel 9.4)
Wir berechnen näherungsweise 6 in Q5 , d.h. wir lösen x 2 = 6 in Q5 . Gesucht werden a 0 , a1, a2 ,... mit
2
0 ≤ ai ≤ 4 und ( a 0 + a1 ⋅ 5 + a2 ⋅ 52 + ... ) = 6 = 1 + 1 ⋅ 5 .
2
Koeffizientenvergleich mod 5 , mod 5 , mod 53 , … liefert
a 0 = 1 , a1 = 3 , a2 = 0 , a 3 = 4 , … oder a 0 = 4 , a1 = 1 , a2 = 4 , a 3 = 0 , …
Die Gleichung x 2 = 6 hat also in Q5 die beiden Lösungen
a = 1 + 3 ⋅ 5 + 0 ⋅ 52 + 4 ⋅ 53 + ...
und
−a = 4 + 1 ⋅ 5 + 4 ⋅ 52 + 0 ⋅ 53 + ...
Ein Beispiel für die ungewohnten Konsequenzen des nicht-archimedischen Absolutbetrags gibt
Satz 9.15 (9.9)
Sei ( cn )n ∈N0 eine Folge über Qp . Dann gilt
∞
∑ cn
ist konvergent ⇐⇒ lim cn = 0 .
n →∞
n =0
Beweis
„ ⇒ “ Sei S N :=
N
∑ n =0 cn , also existiert
limN →∞ SN . Es folgt
lim cn = lim ( Sn − Sn −1 ) = lim Sn − lim Sn = 0 .
n →∞
n →∞
„ ⇐ “ Sei m > n . Dann gilt
Sm − Sn p = cn +1 + cn +2 + ... + cm
n →∞
p
n →∞
≤ max { cn +1 p ,..., cm
Also ist ( Sn )n ∈N eine Cauchy-Folge und somit konvergent in Qp .
66
p
} → 0 für n → ∞ .
Literaturverzeichnis
Literatur zur Zahlentheorie
•
Baker, Theory of Numbers, Cambridge University Press 1984
Knappe Darstellung der Grundlagen inklusive Bemerkungen zu aktuellen Entwicklungen.
•
P. Bundschuh, Einführung in die Zahlentheorie, Springer 2002
Ausführliche Darstellung inklusive geschichtlicher Entwicklung, reichlich diophantische Approximation.
•
G.H. Hardy, E.M. Wright, Einführung in die Zahlentheorie, Oldenbourg 1958
Umfangreiches Standardwerk der elementaren Zahlentheorie.
•
K.-H. Indlekofer, Zahlentheorie, Birkhäuser 1978
Kurze Einführung mit Übungsaufgaben.
•
I. Niven, H.S. Zuckermann, Einführung in die Zahlentheorie (B.I. Wissenschaftsverlag, Band 46 und 47)
Umfassende Darstellung mit Weiterführung.
Empfehlenswert und interessant ist auch das Buch von G.H. Hardy „Divergente Reihen“, in dem auch andere
Konvergenzbegriffe beschrieben werden, die z.B. Euler für seine Beweise benutzt hat und die schwächer sind
als die meistbekannten, d.h. dass eine Reihe, die so eine schwächere Konvergenzeigenschaft hat, z.B. nach
Cauchy divergent ist.
67
68
Index
a 0 ; a1,..., an z03a
#
Na
a 0 ; a1, a2 ,... z04a
3
Zb
48, 51
3
50
A z04b
53
n
n
n
58
2
2
2
x + y = z z05a
Qc
3
x + y = z z05b
58
a |b d
4
x 4 + y 4 = z 2 z05c
59
ggT e1
( a, b ) e2
5
5
4
4
4
n
m
= 1 z05e
61
n
m
= 1 z06
61
x + y = z z05d
x −y
59
P e3
6
3 −2
ep ( n ) f
7
ep ( n ) z07
62
kgV g
7
⋅ p z08
62
x h
9
Z p z09
65
x}i
n
j1
k
f ( n ) j2
9
10
g ( n ) j3
10
ϕ (n ) k
[
]
{
( )
A
10
abelsche Teilsummation (Lemma)
45
abundant
15
11
additiv
46
μ (n ) l
12
algebraisch
τ (n ) m
14
σ (n ) n
14
Approximationssatz
O (⋅) o
16
dirichletscher ~
47
o (⋅) p
16
hurwitzscher ~
51
a ≡ b mod m q
19
äquivalent
35
19
äquivalent (Cauchy-Folgen)
63
25
archimedisch
62
28
arithmetische Progression
60
asymptotisch gleich ( ∼ )
42
[ 0 ]m
r
ordm ( a ) s ( om (a ) )
a
u1
p
( )
p
(
a)
u2
a
v
n
h (d ) w
( )
29
~ vom Grad n ( α )
53
B
32
37
befreundet
15
2x
39
Binomialkoeffizient
10
π (x ) y
42
Bruchteil
∼ z00
42, 63
)
ϑ x z01
42
ω ( n ) z02
46
(
9
C
catalansche
Index
~ Gleichung
61
arithmetische ~
10
~ Vermutung
61
Eulersche ϕ -~
11
Chebyshev (Satz)
43
Möbiussche μ -~
12
chinesische Restsatz, der
21
summatorische ~
10
zahlentheoretische ~
10
D
G
definit
in~
35
Ganzteil
positiv ~, negativ ~
35
Gauß
defizient
15
Lemma von ~
diophantische
~ Exponential-Gleichung
~ Gleichung
61
5
Diskriminante
~ von f (bin. quad. Form)
Division mit Rest (Satz)
9
gemeinsamer Teiler
4
größter gemeinsamer Teiler ( ggT )
5
größtes Ganzes
9
Grundlösung
35
Hauptterm
eigentlich dargestellt
38
Erdõs & Shapiro (Beweis)
43
Erdõs, Paul (Beweis)
43
1. ~ (Satz)
29
2. ~ (Korollar)
30
inkongruent
Jacobi-Symbol
5
Kettenbruch
28
Existenz eines größten gemeinsamen Teilers
4
16
Fermat
23
fermatsche
~ Vermutung
~r Abstieg
endlicher ~
48
unendlicher ~
50
Klassenzahl
F
der kleine Satz von ~
32
K
6
Fehlerterm
19
J
Euklid
(Lemma)
16
I
Ergänzungsgesetz
eulersches Kriterium (Satz)
58
H
E
euklidischer Algorithmus
29
Gaußklammer
4
Satz von ~
9
59
37
kleinstes gemeinsames Vielfaches ( kgV )
7
Komplementärteiler
4
kongruent
19
Kongruenz
19
lineare ~
20
Kongruenzsystem
21
Körper
40, 59
endlicher ~
21
Fermatsche
~ Gleichung
58
~r Abstieg
60
Fundamentalsatz der Zahlentheorie
L
Lagrange
6
Satz von ~
Funktion
69
24, 53
Index
Landau-Symbol
16
~ Gleichung
56
Legendre-Symbol
28
~ Grundeinheit
58
Liouville
perfekt
Satz von ~
lösbar (lineare Kongruenz)
54
Primfaktorzerlegung
20
kanonische ~
Lösbarkeit linearer diophantischer Gleichungen
(Satz)
6
Primitivwurzel
5
25
Primzahl
6
~zählfunktion
M
Menge A aller algebraischen Zahlen, die
15
42
die Menge aller ~en
53
Mertens
Satz von ~
44
Mihailescu, Preda
61
Minimalpolynom
53
6
mersennesche ~
15
Primzahlsatz (PZS)
42
pythagoräische Tripel
58
Q
Möbius-Umkehrung
Quadrate-Satz
1. ~ (Satz)
13
Vier-~
40
2. ~ (Satz)
14
Zwei-~
39
Modul
19
quadratfrei
12
modulo
19
quadratfreier Kern
40
multiplikativ
10
quadratische Ergänzung
28
10
quadratische Form
total ~
binäre ~
N
Näherungsbruch
n -ter ~
51
nicht-archimedisch
62
normiert (Kettenbruch)
48
Nullteiler
21
quadratischer Nichtrest mod m
28
quadratischer Rest mod m
28
quadratisches Reziprozitätsgesetz (Satz)
31
R
O
Omega von n ( ω ( n ) )
Reduktionssatz
36
reduziert
37
Rest
46
Anzahl der betragskleinsten ~e (
Ordnung
p
betragskleinster ~
~ von a mod m ( ordm ( a ) )
~ von p in n ( ep ( n ) )
25
7, 62
Ostrowski
(
a))
29
29
Restklasse
19
Restklassenring
19
Restsystem
Satz von ~
65
P
primes ~
22
volles ~
19
p-adischen Zahlen
Ring der ganzen ~, der
S
65
Satz von Euklid-Euler
pellsche
~ Einheit
35
15
Schranken für einige zahlentheoretische
57
Funktionen (Satz)
70
15
Index
Schubfachprinzip, Dirichlets
teilt
47
Schubfachschluss
unendlicher ~
57
Theta von x ( ϑ ( x ) )
42
transzendent (reelle oder komplexe Zahl)
53
Stirling-Formel
sehr schwache ~ (Lemma)
U
42
unimodular
T
Taubenschlagprinzip
47
Taylor
59
Teiler
4
trivialer ~
Vielfaches
6
15
W
teilerfremd
5
Wiles
5
Wilson
Satz von ~
14
Teilnehmer
j -ter ~ des Kettenbruchs
4
vollkommen
14
Teilersumme
35
V
Teileranzahl
paarweise ~
4
48
71
59
23
Herunterladen